Download as pdf or txt
Download as pdf or txt
You are on page 1of 77

1

INDEX

1. Stone Age / Indus Valley Civilization – Bronze Age ........................................................................... 2


2. Early Vedic Age (1500-900 BC) / Later Vedic Age (900-600 BCE) Mahajanapadas / Age of Buddha 11
3. Jainism & Buddhism ....................................................................................................................... 21
4. Mauryan Empire, Post Mauryan Age, Megaliths & Sangam Age .................................................... 32
5. Gupta Age, Post Guptas .................................................................................................................. 41
6. Pallavas & Chalukyas in South, Chola Empire (9th -12th Century) .................................................. 48
7. Ghaznavid & Turkish Invasions, Delhi Sultanate (1200 - 1400 CE) .................................................. 57
8. Vijayanagara & Bahmini kingdom .................................................................................................. 63
9. Mughal Empire, Rise of Marathas................................................................................................... 67
10. Religious & Cultural Development in India Bhakti Sufi movements etc .......................................... 70
2

Stone Age / Indus Valley Civilization – Bronze Age


Q.1) Answer: C  Option (a) is correct: Menhir and Dolmen are
 Option (c) is correct: The term 'Teri’ signifies a the forms of different burial
coastal landscape caused by sand dunes. practices followed by the Megalithic builders
 These soils may have originated in the of South India.
Pleistocene epoch of the Quaternary period  Many archaeologists place the Megaliths
Mesolithic sites in India are found in found in south India around (1500 BC to 500
the Coastal areas Mumbai , teri sites of BC), i.e. the Iron age.
Thoothukudi in Tamil Nadu and  Megaliths refer to large stone structures that
Vishakapatnam. These sites have yielded were constructed either as burial sites or as
microlithic tools which is a marked feature of commemorative site
the Mesolithic age.  A Menhir is a stone Monolithic standing
vertically. It could also exist as part of a group
Q.2) Answer: B of similar stones
 Statement 1 is incorrect: The Chalcolithic  Dolmen: This is a type of megalith which is
people were not acquainted with burnt bricks made in a single chamber tomb, usually
and rarely used in the settlements. Their consisting of three or more upright stones
houses were made of mud bricks but mostly supporting a large flat horizontal capstone.
these were constructed with wattle and daub Menhir
and seemed to have been thatched houses.
 Statement 2 is correct: The Chalco-lithic Dolmen
people used copper as well stone tools. The
stone tools found at the Chalcolithic sites
indicate a large number of stone tools smaller
in size known as Microliths.
 Statement 3 is correct: The Chalcolithic sites in
Western Maharashtra have yielded large
numbers of burials of Children. This indicates
that the rate of infant mortality in Chalcolithic
settlements was very high.

Q.5) Answer: C
Q.3) Answer: C Statement 1,2 and 4 are correct:
 Both statements are correct: The painted grey  Animals represented on seals and terracotta
ware (PGW) is dated too. 1100 to 800 BCE. The art of the Harappan Culture – Unicorn (Bull),
pottery was fine grey in colour with painted Antelope, Crocodile ,Rhinoceros, Tiger.
geometric designs. The painted grey ware laid  Animals NOT represented on seals and
the foundation of the early political
formations. The Iron Age in North India
coincides with the painted Grey Ware
culture.
 The Painted Grey Ware cultural phase is
followed by Northern Black Polished Ware
culture (NBPW), which is associated with the
Mahajanapada and Mauryan periods.
Source: Tamil Nadu State Board
terracotta art of the Harappan Culture - Camel
, Horse , Lion.

Q.4) Answer: A
3

Q.7) Ans: A
Extra Edge by Only IAS
Exp:
About Mesolithic culture
● Pashupati (lord of shiva) seal is a steatite seal
 Mesolithic sites are found in most parts of
with a depiction of a three headed figure
India. They occur in all eco-zones from the
of Mahadeva surrounded by four animals
coasts to the hills: sand dunes, rock shelters,
Elephant,TIger, Rhino and Buffalo.
deltaic regions, lake areas, forested territories,
● Pashupati is represented as a yogi or a proto
hilly and mountainous areas, rocky terrains
shiva figure seated in a traditional cross leg
and coastal environments.
pose.
 Statement 1 is correct: Hunting wild animals
and gathering plant food and fishing were
people’s main occupation during this age.
Q.6) Answer: D
Agriculture was not practised in the early
 Statement 1 is Incorrect : Harappan people
stages. At the end of the Mesolithic period,
Engaged in inter-regional as well as foreign
humans domesticated animals and paved the
trade. Sumerian texts refer trade relation
way for the Neolithic way of life. The rock
with Meluha ie. ancient name given to Indus
paintings of Central India depict hunting,
region & mentions 2 intermediate trading
trapping, fishing and plant food collection.
stations- Dilmun (Bahrain) & Makan (Makran
 Statement 2 is correct: The faunal evidence
coast). Gold, copper, tin and several semi
from this period shows that people belonging
precious stones were imported. Main exports
to this period hunted cattle, gaur, buffalo,
were several agricultural products such as
barasingha, porcupines, sambar, chital,
wheat, oilseeds, barley, cotton goods,
gazelle, hog deer, nilgai, jackal, turtle, fish, wild
terracotta figurines, beads etc.
hare, lizard fox and monitor lizard.
Bones of rhinoceros and elephant have also
been found. They used spears, bow and arrow
and traps. The paintings of Bhimbetka show
that various animals were hunted and for this
men and women went together. The people
used fire and perhaps roasted food.
 Statement 3 is incorrect: The Mesolithic
people buried the dead, which suggests their
beliefs and humane relationships. Human
skeletons have been found in Mahadaha,
 Statement 2 is Incorrect : The Harappans
Damdama and Sarai Nahar Rai in Uttar
got copper from the present day Rajasthan and
Pradesh. At Mahadaha, a man and a woman
Baluchistan But they were not able to produce
were buried together. One burial had an ivory
enough copper to export it to its contemporary
pendant as the grave good.
Mesopotamian civilization.
 Statement 3 is Incorrect : There is evidence of
Q.8) Ans: B
use of gold, silver, nickel and precious metal in
Exp:
the civilisation that was sourced from Southern
Bhimbetka Rock Paintings:
India but there is no evidence of use of iron in
 The tools show well thought-out design and
civilization. Although Harappan Culture
physical symmetry, and convey high-quality
belongs to the Bronze age, they used bronze
cognitive (perception) skills and capabilities of
on a limited scale. Bronze is an alloy of copper
prehistoric humans. They used the tools for
and tin.
hunting, butchering and skinning the animals,
breaking the bones for bone marrow and to
recover tubers and plant foods, and for
processing food.
4

 Art is an integral part of human existence. cultural developments of this period are called
While evidence of art is found in Europe in the Neolithic revolution.
large volume, they are found only at a few sites  The Neolithic cultures of India are divided into
in India. A chert stone used as a core had various regional cultures and they flourished
geometric engravings from Chandravati in in different time periods. In the north-western
Rajasthan, bone objects from Bhimbetka and part of India and Pakistan, it began at a very
human tooth engraved with geometric design. early date. In north-eastern India, Neolithic
cultures appeared at a very late date, around
the early historic time

Q.10) Ans: D
Exp:
 Mature Harappan culture developed in some
of the areas occupied by the Early Harappan
cultures. These cultures also shared certain
common elements including subsistence
strategies.
 Rock paintings are found in the rock shelters of
 Statement 1 is correct: The Harappans ate a
Madhya Pradesh and Central India. They show
wide range of plant and animal products,
people hunting, trapping animals and fishing
including fish. Archaeologists have been able
and dancing. Bhimbetka near Bhopal, Raisen
to reconstruct dietary practices from finds of
and Pachmarhi in Madhya Pradesh and South
charred grains and seeds.
Mirzapur in Uttar Pradesh are some of the
o These are studied by archaeo-
sites. Haematite, an iron-rich stone with
botanists, who are specialists in
traces of rubbing, has been found. These
ancient plant remains. Grains found at
people might have decorated themselves with
Harappan sites include wheat, barley,
flowers and leaves.
lentil, chickpea and sesame. Millets
are found from sites in Gujarat. Finds
Q.9) Ans: C of rice are relatively rare.
Exp:
 Statement 2 is incorrect: Animal bones found
 Statement 1 is correct: The Neolithic period
at Harappan sites include those of cattle,
marked the beginning of agriculture and
sheep, goat, buffalo and pig. Studies done by
animal domestication. It is an important phase
archaeo-zoologists or zoo- archaeologists
in Indian history. Early evidence of Neolithic
indicate that these animals were
culture is found in the Fertile Crescent region
domesticated. Bones of wild species such as
of Egypt and Mesopotamia, the Indus region,
boar, deer and gharial are also found.
the Ganges valley of India and also in China.
 Statement 3 is correct: We do not know
Between 10,000 BCE to 5000 BCE, agriculture
whether the Harappans hunted these animals
emerged in these regions, which led to several
themselves or obtained meat from other
cultural developments.
hunting communities. The Harappans ate a
 Statement 2 is correct: The introduction of
wide range of plant and animal products,
domestication of animals and plants resulted
including fish. Bones of fish and fowl are also
in the production and supply of a large
found.
quantity of grains and animal food. The fertile
soil deposited by the rivers enhanced the
Q.11) Answer: C
growth of agriculture, generating a surplus of
grains.  Statement 1 is correct: The people of the
Surplus food production played a major role in Harappan civilization practiced agriculture as
the rise of early civilisations. Large villages evidence of grains have been recovered which
came to exist and pottery developed. includes wheat, barley, lentil, rice, millets, etc.
Permanent residences were built. Hence, the However, they were mostly located on semi-
arid lands and therefore required extensive
5

irrigation for agriculture. The evidence of a depicts a naked girl covered with ornaments
canal found from Shortughai (trading post) in and standing in a tribhanga position.
Afghanistan also indicates that irrigation might
have been done through this water.

 Statement 3 is correct: Bearded priest is


another figure obtained from Mohenjo-
daro and made of Steatite and is draped in a
shawl.
 Statement 2 is correct: Dholavira in Gujarat
 Statement 4 is incorrect: Male torso is
also contains evidence of a water reservoir
obtained from Harappa and is made up of red
which was used for the storage of water for the
sandstone. It depicts only the frontal posture
purpose of irrigation needs. It is the latest
with holes in places of head and arms.
Harappan city discovered and provides
evidence of a stadium, dams, embankments,
and a large sized advertisement board.
Q.14) Answer: C
Q.12) Answer: B  Statement 1 is correct: Mesopotamian texts
refer to Oman as Magan/Makan. There has
 Option B is correct: The word Shaman is used
been evidence of trade between the Oman and
to indicate those men and women that have
the Harappan people. The trade was of barter
healing and magical powers. They are known
type wherein the Harappans exchanged a Jar
to establish communication with the other
coated with the thick clay for preventing the
worlds. It is regarded as a religious practice
percolation of the stored liquid with that of the
that is associated with the tribal people. It is
Omani copper. Presence of Nickel traces in
different from the Shramana tradition which
both the Harappan and Omani objects
deals with the practices of Jainism and
indicates trade relations between the two.
Buddhism. The Shramana tradition was
patronized by the Ashoka. It was initiated by  Statement 2 is correct: Inland
the wandering ascetics who renounced transportation was highly developed for the
worldly existence for the search of the purpose of trade. Mesopotamian texts refer to
meaning of life and the universe. the people of the Harappan civilization
as Meluhha or the land of
Q.13) Answer: B seafarers. Also, boats and ships are indicated
 Statement 1 is incorrect: It is obtained in some of the Harappan seals.
from Mohenjo-daro and is made of steatite. It
Q.15) Answer: C
depicts a human figure of a deity referred to
as Pashupati with three horned headgear and  Statement 1 is correct: Terracotta figures of
several female goddesses have been found
surrounded by animals like elephants, tiger,
rhinoceros, and from the Harappan seals which indicate the
buffalo. Two antelopes also appear at the worship of Mother Goddess. Also, a plant
emerging from an embryo indicates
bottom of the figure.
that Mother Earth was also worshipped. There
 Statement 2 is incorrect: It is found
has been evidence of Phallus worship which
in Mohenjo-daro and is regarded as
later developed into Hinduism as Shivlinga.
the world’s oldest bronze sculpture that
Hence, a number of symbols of Phallus and
6

female sex organs have been found which on the River Indus. Harappa and Mohenjo-daro
are made of stone. – the two major sites of this civilisation – are
among the earliest and finest examples of
urban civic planning. The towns were laid out
in a rectangular grid pattern. Important
findings - the citadel, the great bath, the great
granary, post cremation burial, sculpture of
bearded priest, the famous bronze statue of
the Dancing Girl and Pashupati Seal.
 Pair 3 is correct: Lothal & Chanhudaro were
famous Bead making factory sites in IVC. Many
 Statement 2 is correct: The people of
seals were found in a storehouse found in
Harappan civilization wore amulets to ward off
Lothal.
any evil spirit. The Bronze statue of the Dancing
The Harappans used a large variety of
girl from Mohenjo-daro also carries an amulet
materials, from precious metals and
in her arms. The use of amulets for warding off
gemstones to bones and even baked clay, to
spirits also finds mention in the Atharva
make ornaments. Both men and women wore
Veda. Atharva Veda is one of the four Vedas
ornaments like necklaces, fillets, armlets and
that deals with the cure of several diseases. It
finger rings. Girdles, earrings and anklets were
also contains the details of several rituals.
worn only by women. Beads made from
 Statement 3 is incorrect: Animals like one
carnelian, amethyst, quartz, steatite, etc. were
horned rhinoceros and the humped bull are
quite popular and were produced on a large
also worshipped by the Harappan people. They
scale, as is evident from the factories
believed in nature worship. Trees are also
discovered in Chanhudaro and Lothal.
depicted in the seals. In one of the seals, a deity
Lothal is situated near the Ahmadabad in
is depicted amidst the branches of a Pipal tree.
Gujarat. Excavations have revealed the world’s
oldest known artificial dock, which was
Q.16) Answer: A connected to an old course of the Sabarmati
 Pair 1 is correct: At Dholavira a signboard River.
containing ten letters of the Indus script was  Pair 4 is incorrect: The famous Pashupati seal
found. The inscription is the one of the longest is a steatite seal discovered at Mohenjo daro
in the Indus script. However until the Indus depicts a deity sitting cross legged. The figure
script is deciphered, what the sign is saying still wears a 3 horned headgear & is surrounded by
remains a mystery. Dholavira is 5th largest city animals Elephant, tiger, rhinoceros, buffalo & 2
of Indus civilization and it was discovered in antelopes.
1968 by archaeologist Jagat Pati Joshi. It is
recently added in the UNESCO world heritage
Q.17) Answer: C
sites. It is 40th site from India and the first site
● Statement 1 is correct: Terracotta figures of
of the ancient Indus Valley Civilisation (IVC) to
women from the Chalcolithic period suggest
get the tag. Other findings at Dholavira - giant
that the Chalcolithic people venerated the
water reservoir, unique water harnessing mother goddess. Similar terracotta figures of
system, stadium, dams and embankments the mother goddess have also been found
 Pair 2 is correct: The figure of a bearded man from the Indus Valley Civilisation.
or the Priest king was found at Mohenjo Daro. ● Statement 2 is incorrect: The people of the
Fillet with circular inlay ornament on the Chalcolithic period used to deposit pots and
forehead & smaller ornament on the Right some copper objects in the graves for the use
upper arm. Mohenjo-Daro is the largest of all of the dead in the next world. This shows their
the Indus cities. It is located in present Pakistan belief in the afterlife. Similarly, Harappan
7

people also buried pottery and ornaments,


indicating a belief that these could be used in
the afterlife.
● Statement 3 is correct: Both the settlement
pattern and burial practices suggest the
beginning of social inequalities in the
chalcolithic society. Some settlements in Jorwe
are as large as twenty hectares but others are
only five hectares and even less. This suggests The findings
that the large settlements dominated the from Mohenjodaro include a great bath,
smaller ones. In the Indus Valley Civilisation, the great granary, post cremation burial,
the citadel and the lower town were depiction sculpture of a bearded priest, the
of social inequalities. famous bronze statue of the Dancing Girl and
Pashupati Seal.
Q.18) Answer: C
 Pair 1 is correct: At Lothal, a brick structure has
been identified as a dockyard meant for
Q.19) Answer: D
berthing ships and handling cargo. This
 Statement 1 is correct: Perhaps the
suggests that Lothal was an important port
most unique feature of the Harappan
and trading centre of the Harappan people.
Civilisation was the development of urban
 Pair 2 is correct: One of the most exciting centres. The settlement is divided into two
discoveries at Dholavira is a large wooden sections, one smaller but higher and the other
"signboard" just outside the north entrance much larger but lower.
to the citadel. This is actually one of ● Archaeologists designate these as the Citadel
the longest Indus inscriptions known. There and the Lower Town respectively. The Citadel
are 10 symbols in the panel, each one is about owes its height to the fact that buildings were
37 centimetres high and the board on which constructed on mud brick platforms. It was
the letters were inscribed appears to have walled, which meant that it was physically
been about 3 meters long. However until the separated from the Lower Town.
Indus script is deciphered, what the sign is
saying still remains a mystery.
 Pair 3 is correct: The furrows discovered in
the pre-Harappan phase
at Kalibangan indicate that the fields
were ploughed in Rajasthan during the
Harappan period. The Harappans probably
used the wooden plough drawn by oxen,
and camels may also have been used for this
● Statement 2 is correct: One of the
purpose.
most distinctive features of Harappan
 Pair 4 is incorrect: The findings cities was the carefully planned drainage
from Harappa include 2 rows of six system. If you look at the plan of the Lower
granaries with big platforms, stone Town you will notice that roads and streets
symbols of lingam and yoni, mother goddess were laid out along an approximate “grid”
sculpture, wheat and barley in wooden pattern, intersecting at right angles. It seems
mortar, dice, copper scale and mirror. that streets with drains were laid out first and
Moreover, a sculpture of a dog chasing a then houses built along them. If domestic
deer in bronze metal, and a red sandstone wastewater had to flow into the street drains,
male torso have been excavated. every house needed to have at least one wall
along a street.
8

● Statement 3 is correct: The Lower Town  Mature Harappan sites and location:
at Mohenjodaro provides examples o Manda -- India
of residential buildings. Many were centered o Harappa --- Pakistan
on a courtyard, with rooms on all sides. The o Banawali ---- India
courtyard was probably the centre of o Rakhigarhi---India
activities such as cooking and o Mitathal----India
weaving, particularly during hot and dry o Ganweriwala----Pakistan
weather. What is also interesting is o Kot Diji ----Pakistan
an apparent concern for privacy: there are no o Amri ----Pakistan
windows in the walls along the ground level. o Chanhudaro---Pakistan
Besides, the main entrance does not give a o Balakot----Pakistan
direct view of the interior or the courtyard. o Mohenjodaro---Pakistan
o Sutkagendor--- pakistan
o Dholavira--- India
Q.20) Answer: D o Lothal --- India
 Mature Harappan culture developed in some o Nageshwara -- india
of the areas occupied by the Early Harappan
cultures. These cultures also shared
certain common elements including Q.22) Answer: C
subsistence strategies. The Harappans ate  Statement 1 is correct: The Indus people
a wide range of plant and animal sowed seed in the floodplains in November,
products, including fish. when the flood water receded, and reaped
 Archaeologists have been able to reconstruct their harvest of wheat and barley in April,
dietary practices from finds of charred before the advent of the next flood. They
grains and seeds. These are studied produced wheat, barley, rice, peas,
by archaeo-botanists, who are specialists sesame, mustard and rice.
in ancient plant remains.  Statement 2 is correct: Foodgrains were
 Grains found at Harappan sites include wheat, stored in huge granaries in both Mohenjodaro
barley, lentil, chickpea and Harappa. Probably, cereals were received
and sesame. Millets are found from sites in as taxes from peasants and stored in
Gujarat. Finds of rice are relatively rare. granaries for the payment of wages as well as
Animal bones found at Harappan sites include use during emergencies.
those of cattle, sheep, goat, buffalo and  No hoe or ploughshare has been discovered,
pig. Studies done by archaeo-zoologists but the furrows discovered in the pre-
or zoo-archaeologists indicate that these Harappan phase at Kalibangan show that the
animals were domesticated. fields were ploughed in Rajasthan in
 Bones of wild species such as boar, deer and the Harappan period. The Harappans probably
gharial are also found. We do not know used the wooden ploughshare. We do not
whether the Harappans hunted these know whether the plough was drawn by men
animals themselves or obtained meat from or oxen. Stone sickles may have been used for
other hunting communities. harvesting the crops.

Q.21) Answer: B Q.23) Answer: A


 The Indus Valley was home to the largest of  Statement 1 is correct: Neolithic Age (6000 B.C
the four ancient urban civilizations of to 4000 B.C.) was the last and third part of the
Egypt, Mesopotamia, India and China. In the Stone Age. The Neolithic Age saw a man
1920s, the Archaeological Department of turning into a food producer from a food
India carried out excavations in the Indus gatherer. It also witnessed the use of pottery
valley wherein the ruins of the two old cities, for the first time. People used microlithic
viz. Mohenjodaro and Harappa were blades in addition to tools made of polished
unearthed. stone. The use of metal was unknown.
9

 Statement 2 is correct: The Neolithic settlers  It is a nude female figure, with a right arm on
were the earliest farming communities. They the hip and left arm hanging in a dancing pose.
broke the ground with stone hoes and digging She is wearing a large number of bangles.
sticks at the end of which ring stones weighing  Pottery-making was also an important industry
one to half a kilogram were fixed. The people in the Harappan period. These were
of the Neolithic Age cultivated ragi, horse chiefly wheel-made and were treated with
gram, cotton, rice, wheat, and barley and a red coating and had decorations in black.
hence were termed as food producers.  The painted motifs, generally noticed on the
 Statement 3 is correct: They led a settled life. pottery, are pipal leaves, fish-scale design,
The people of the Neolithic Age lived in intersecting circles, zigzag lines, horizontal
rectangular or circular houses which were bands and geometrical motifs with floral and
made of mud and reed. The people of faunal patterns. The Harappan pottery is well
Mehrgarh lived in mud-brick houses while pit- baked and fine in decorations. Hence
dwelling is reported from Burzahom, statement 2 is correct.
the Neolithic site found in Kashmir.
Q.26) Answer: A
SOME MAJOR STRUCTURAL REMAINS OF THE
Q.24) Answer: B
HARAPPAN TOWNS-
 Statements 1 is incorrect: Archaeologists have
 At Mohenjodaro the ‘Great Bath’ is the most
discovered thousands of seals, mostly made
important structure.
of steatite, and occasionally of agate, chert,
copper, faience, and terracotta, with beautiful
figures of animals, such as unicorn bull,
rhinoceros, tiger, elephant, bison, goat,
buffalo, etc. Some seals have also been found
in ivory.
 Statement 2 is correct: The realistic
rendering of these animals in various moods is
remarkable. The purpose of producing
seals was mainly commercial. It appears that
the seals were also used as amulets, carried on
the persons of their owners, perhaps as It is surrounded by corridors on all sides and
modern-day identity cards. The standard is approached at either end by a flight of steps
Harappan seal was a square plaque 2×2 square in north and south. A thin layer of
inches and rectangular made from steatite. bitumen was applied to the bed of the Bath to
Every seal is engraved in a pictographic script ensure that water did not seep in. Scholars
which is yet to be deciphered. believe that the ‘Great Bath’ was used for
ritual bathing. Hence pair 1 is correctly
matched.
Q.25) Answer: A  At Lothal, a brick structure has been identified
 The Harappan people were aware of almost all as a dockyard meant for berthing
the metals except iron. They manufactured ships and handling cargo. This suggests that
gold and silver objects. Lothal was an important port and trading
 A number of copper tools and weapons have center of the Harappan people.
also been discovered. The common tools Archaeologists have also found evidence of
included an axe, saws, chisels, knives, a ploughed field at Kalibangan associated with
spearheads and arrowheads. Early Harappan levels.
 There is evidence of the use of the
bronze though in a limited manner. The
most famous specimen in this regard is the
bronze ‘dancing girl’ figurine discovered at
Mohenjodaro. Hence statement 1 is incorrect.
10

 Statement 2 is incorrect: The Harappan


civilization was marked by the absence of the
The field had two sets of furrows at right temples and favored the worship of trees,
angles to each other, suggesting that two animals, and followed idol worship. There
different crops were grown together. Hence were no temples, and it was largely
pair 2 is incorrectly matched. a secular civilization. Yet, a number of
 In Dholavira, The remains show an terracotta figurines of the female goddess, the
imposing citadel in the centre, with a middle Pashupati seal, and the phallus worship
and lower town, each fortified separately, built indicate that idol worship was practiced.
with pleasingly smoothed structures of sun-
dried brick and stone masonry, and with
Q.28) Answer: A
remarkable town planning. Well laid out lanes
lead outward systematically from the citadel,  Statement 1 is correct: Harappans greatly
with a well-constructed underground drainage specialized in the making of beads and
system for sanitation. There is a made jewellery of gold, silver, carnelian, and
large stadium with a complex structure and other precious stones. At a site
seating arrangement in Dholavira. Hence pair named Allahdino, near Karachi, a lot of
3 is incorrectly matched. jewellery made up of gold, silver, copper,
carnelian, and other precious stones have been
Q.27) Answer: D unearthed.
 Statement 1 is incorrect: The Harappan  Statement 2 is incorrect: Kot Bala is a
civilization or the Indus Valley Civilization can Harappan site located in Pakistan from where
be classified into three phases: Early Harappan the earliest evidence for the use
Phase (3200-2600 B.C.); Mature Harappan of furnaces have been found. Kerala-no-
Phase (2600-1900 B.C.); and Late Harappan dhoro in Gujarat is another Harappan site
Phase (1900-1300 B.C.). There exists slight from where evidence of salt production has
variation in each of the phases, yet it is marked been obtained.
by continuity in each of the phases. The  Statement 3 is correct: Balu is a Harappan site
Harappan civilization was known for its located in the state of Haryana from where the
extensive town planning and well designed evidence for the production of Garlic has been
drainage pattern. Besides, there was a huge found. It is also famous for the various plant
degree of craft specialization. The Harappan remains obtained from this site.
people were well equipped with potter’s
wheel and made red pottery with black Students Note:
designs. However, they were poor in stone
art.
11

Early Vedic Age (1500-900 BC) / Later Vedic Age (900-600 BCE) Mahajanapadas /
Age of Buddha
associated with it were known as karmar. The
Q.1) Answer: A Aryans were engaged in two types of conflicts.
 Statement 1 is correct - Cattle were
considered to be the most important wealth
and among them cows were the most
precious. Wealth was determined by the
number of cows one possessed.
 Statement 2 is correct - Cows were given as
gifts to the priests which suggests its sanctity First they fought with pre-Aryans/indigenous
as a sacred thing.Cows were given to the people and secondly they fought among
officiating priest in the rajasuya sacrifice to pay themselves. Wars were fought mainly to gain
tribute. supremacy of cattle wealth and in the later
period for territorial expansion.

Q.3) Answer: C
 Option A is Incorrect: The later Vedic text
draws a line between the three higher varnas
on the one hand and the Shudras on the other.
The worst position was reserved for the
 Statement 3 is incorrect - Early Aryans were Shudras. They were made servants of another.
pastoralists in nature and practiced agriculture  Option B is Incorrect:This type of sacrifice is
as a secondary activity. They possessed better known as Rajasuya sacrifice rituals
knowledge of agriculture and were acquainted strengthened the influence of the king.It was
with sowing, harvesting and threshing, and the consecration ceremony/kings inaugural
knew about various seasons also. sacrifice ritual.The king’s influence was
strengthened by this ritual and it was supposed
Q.2) Answer: B
to confer supreme power on him.
 Statement 1 is Incorrect - The king did not
 Option C is Correct: Vish word was used to
maintain any standing army, but in times of
describe the people or the community as a
war he mustered a militia whose military
whole. It was used with the word jana (same
functions were performed by different tribal
meaning). The king received a voluntary
groups called vrata, gana, sardha. Tribal units
present called bali from them.
continued to be assembled in times of war. By
 Option D is Incorrect: Tribal clan based
and large, it was a tribal system of government
assemblies are mentioned as sabha, samiti,
in which the military element was strong.
vidatha and gana are mentioned in the Rig
 Statement 2 is Correct - The rig Vedic people
Veda. Vidhata seems to be the oldest. Sabha
used chariots for warfare.
had few chiefs while Samiti was
The term ayas is used for copper or bronze
larger body. These assemblies were
shows that metal-working was known to them.
conferred with decision making associated
They possessed better arms. The smelting of
with military, religious and political.
metals was known and the population
12

Q.4) Answer: D  Sacrifices became an important and common


 Statement 1 is correct - The two outstanding aspect in the later vedic period. They acquired
Rig Vedic Gods, Indra and Agni, lost their a holistic character and performed both
former importance. Prajapati, the creator publicly and at households.The king along with
came to occupy the supreme position during the entire community took part in it.
the later vedic age. Vishnu came to be  Individuals offered obligations to Agni and
regarded as the preserver and protector of the each of these took the form of a ritual or
people. Pushan was regarded as the God of sacrifice.Sacrifices involved the killing of
shudras who looked after the cattle. animals on a large scale and especially the
 Statement 2 is correct - People worshipped destruction of cattle wealth.The guest was
Gods for the same material reasons in this known as goghna or one who was fed on
period as they did in earlier times.However,the cattle.
mode of worship changed
considerably.Sacrifices and rituals have taken Q.6) Ans: A
the forefront.It involved killing of animals in Exp:
 Statement 1 is correct: The Vedas (Vid = to
public and domestic spaces.Led to the
know, Vidya) are one of the earliest known
destruction of cattle wealth.The gowing cult of
texts to have been composed in India. The
sacrifices enormously added to the power of language of the Vedas is described as Vedic
Brahmanas and the later vedic society got Sanskrit. The Vedas are four: Rig, Yajur, Sama
divided into 4 varnas:Brahmins-Kshatriyas- and Atharva. Among the four Rig Veda is the
Vaishyas-Shudras. oldest. The Vedic texts were memorized and
orally transmitted by Brahmins from
 Statement 3 is correct - Institution of gotra
generation to generation. They were written
appeared in the later Vedic times.It meant the
down in the later time period, with the
place where cattle belonging to the whole clan introduction of writing. The earliest known
are kept,but in course of time it signified written manuscripts of the Vedas date to the
descent from a common ancestor.People 10-11th century CE. They contain information
began to practice gotra exogamy and no about the polity, society, religion and
marriage can take place between same gotra. philosophy, and hence they are a source for
writing history.
Wars fought not only for the possession of
cattle but also for the territory.The famous
battle of Mahabharata is attributed to this
period.
 Statement 4 is correct - The tribal pastoralists
transformed into peasants.The tribal chiefs
started to become dominant at the expense of
the tribal peasant. Crops produced include  Statement 2 is incorrect: The main collections
rice(Vrihi),wheat(Godhuma), sugarcane, lentil, of Vedic hymns are called samhitas. The Rig
millet, barley. Cattle rearing continued to be Vedic- samhita is the earliest text. The Rig
Veda is dated to between 1500 and 1000 BCE.
practiced. Iron was used in the later vedic age
The Rig Veda contains 10 books among
along with bronze and copper it Books 2 to 7 are the earliest and the Books
1, 8, 9 and 10 are assigned to a later period.
Samhitas are ritualistic texts, and they explain
Q.5) Answer: D the social and religious importance of rituals.
Option D is correct:  Each samhita has added texts called
brahmanas, which have commentaries on the
13

hymns and rituals. Each brahmana has an text. The Rig Veda is dated to between 1500
aranyaka (forest text) and an upanishad. The and 1000 BCE. The Rig Veda contains 10 books.
aranyakas contain mystical ritual instructions Books 2 to 7 are the earliest and the Books 1,
to be undertaken in secret by the sages who 8, 9 and 10 are assigned to a later period.
live in the forests. Upanishads deal with  Samhitas are ritualistic texts, and they explain
philosophical enquiries. the social and religious importance of rituals.
Each samhita has added texts called
Q.7) Ans: C brahmanas, which have commentaries on the
Exp: hymns and rituals. Each brahmana has an
 Statement 1 is correct: Samhitas are ritualistic aranyaka (forest text) and an upanishad. The
texts, and they explain the social and religious aranyakas contain mystical ritual instructions
importance of rituals. Each samhita has added to be undertaken in secret by the sages who
texts called Brahmanas, which have live in the forests. Upanishads deal with
commentaries on the hymns and rituals. Each philosophical enquiries.
Brahmana has an aranyaka (forest text) and  The Yajur, Sama and Atharva Vedas are dated
an upanishad. The aranyakas contain mystical to a slightly later period.
ritual instructions to be undertaken in secret
by the sages who live in the forests.
Upanishads deal with philosophical enquiries
 Statement 2 is correct: Sama veda is an
ancient Vedic Sanskrit text, and part of the
scriptures of Hinduism. One of the four Vedas,
it is a liturgical text which consists of 1,875
verses. All but 75 verses have been taken from The
the Rigveda. The Yajur, Sama and Atharva samhitas of the Sama, Yajur and Atharva
Vedas are dated to a slightly later period. The Vedas, and the brahmanas, aranyakas and
samhitas of the Sama, Yajur and Atharva upanishads attached to the Vedas are the Late
Vedas, and the brahmanas, aranyakas and Vedic texts. The Sama Veda Was composed in
upanishads attached to the Vedas are the Late musical notes which are considered to
Vedic texts. The Sama Veda was composed in constitute the basis of Indian music. The Yajur
musical notes which are considered to Veda has rituals and hymns. The Atharva Veda
constitute the basis of Indian music. The Yajur contains charms and magical spells.
Veda has rituals and hymns. The Atharva Veda
contains charms and magical spells.
Q.9) Ans: A
Q.8) Ans: D Exp
Exp:  The burial system followed by the people of
All options are correct: the Neolithic period continued into the
 The Later Vedic culture is dated to the period Megalithic period. A circular tomb using
between 1000 BCE and 700–600 BCE. The o big stone slabs built upon the place of
Painted Grey Ware Culture of the Iron Age, burial is known as a megalith. Such
which has been identified by archaeologists at megaliths have been found in many parts
many excavated sites, is associated with the of Tamilnadu .
Later Vedic culture. This period witnessed  The urn burial system was another type of
political, social, economic complexity and practice and is evidenced in Adichanallur
developments. The Later Vedic texts were (present Thoothukudi district). Black-ware is
composed after the Rig Veda Samhitas. The peculiar to burial sites in Tamilnadu.
Yajur, Samaand Atharva Vedas were Interestingly, black-ware is found mostly in
composed after the Rig Veda. burial mounds and not in human habitations.
 The main collections of Vedic hymns are called
samhitas. The Rig Vedic-samhita is the earliest
14

the divine nature of kingship gets mentioned


in the literature of this period.
 Statement 2 is incorrect: In the Later-Vedic
period, Vidatha completely disappeared.
The sabha and samiti continued to hold their
presence but they lost their popular
character and came to be dominated by chief
and rich nobles. Women were no longer
permitted to sit in the Sabha.
 In a majority of urn burials, the use of stone is  Statement 3 is incorrect: There was no
almost non-existent. However, urn burials are standing army in the Vedic period and in time
grouped under megalithic because the
of war military functions were performed by
materials - the pottery, iron objects, beads of
different tribal groups called Vratagana,
semi-precious stones kept in them - are
identical to those found in the stone burials. grama, sardha. Even in later Vedic times the
king did not have a standing army. Tribal
Q.10) Ans: D units were mustered in times of war and,
Exp: according to one ritual, for success in war, the
 Statement 1 is correct: The Vedic people king had to eat along with his people (vis) from
distinguished themselves from the non-Aryan the same plate.
people. Varna was the term used by Aryans to
refer to colour and category. The Rig Veda Q.12) Ans: B
refers to Arya varna and Dasa varna. The Dasas Exp:
and Dasyus were conquered and treated as  Statement 1 is correct: Later Vedic Culture
slaves. They came to be considered sudras in
and Civilization (C. 1000-500 BCE) : The Later
the later period. Social classes were classified
as warriors, priests and common people. Vedic Age is characterised by more
 Statement 2 and 3 are correct: Sudras as a complexities in social, political, and economic
category of people appeared at the end of the life. In terms of the political context, the tiny
Rig Vedic period. Slavery was common and tribal settlements of the Vedic period were
slaves were given as gifts to the priests, but replaced by comparatively stronger kingdoms,
there is no reference to wage labour. Horse-
while royal power increased. One of the most
drawn chariots and bronze objects were
possessed by a few, suggesting social important changes from the Rig Vedic society
distinction. Vedic society was largely was the rise and growth of social
egalitarian initially, and social distinctions differentiation in the form of the varna
emerged later. system.Caste exogamy was extensively
 According to the Purusha Sukta of the Rig Veda practiced and a rigid social hierarchy
the various varnas emerged thus: Brahmanas developed, which restricted the social mobility
from the mouth, the kshatriya from the arms,
of the earlier period. The increasing cult of
the vaisya from the thighs and the sudra from
the feet of Purusha, when he was sacrificed. sacrifices added enormously to the power of
the Brahmanas.
 Statement 2 is incorrect : In the Later Vedic
Q.11) Answer: A
family, a patrimonial (authority of father)
 Statement 1 is correct: In the Rig Vedic
system developed and women were generally
period the post of the tribal chief was not
confined to household chores and subordinate
hereditary and the tribe generally elected him.
positions. As compared to the Rig Vedic age,
In the Later-Vedic Period the post of tribal
chief had become hereditary. The idea of women lost importance in the society and
there have been references to rare instances of
15

sati and child marriage.In one text, women alliances, friendship with strong rulers, and
have been counted as a vice along with dice conquest of weak neighbours to expand the
and wine. In another text, a daughter has been empire.
said to be the source of all sorrows.  Statement 2 is incorrect : Udayin (c. 460-444
 Statement 3 is correct : Pottery plays an BCE) also known as Udayabhadra was a king of
important role in studying culture and Magadha in ancient India. According to the
reconstructing the past. Historically with Buddhist and Jain accounts, he was the son
distinct culture, the style of pottery changed. It and successor of the Haryanka king
reflects the social, economic and Ajatashatru. Udayin laid the foundation of the
environmental conditions a culture thrived in, city of Pataliputra at the confluence of two
which helps the archaeologists and historians rivers, the Son and the Ganges. He shifted his
in understanding our past . The Later Vedic capital from Rajgriha to Patliputra due to the
culture is also called as PGW–Iron Phase latter's central location in the empire.
culture, as a particular kind of pottery (painted  Statement 3 is correct : Forests in southern
grey ware) was used during that phase. areas of Magadhan Empire provided timber
and elephants, which gave a special military
advantage to Magadha as all the other
Mahajanapads effectively used horses and
chariots in warfare. Magadha was the first
state to use elephants on a large scale against
its neighbours. Elephants could be used in
storming fortresses and in marching over areas
 Statement 4 is correct: Assemblies were then lacking roads or other means of
dominated by nobles and affluent men and
communication.
women were no longer permitted to attend
assemblies. Sabha and Samities continued to Q.14) Answer: C
hold ground, though not like the Rig Vedic Age,  Statement 1 is correct : Aryan society was
but the vidatha completely disappeared. male dominated and patriarchal in nature. The
birth of daughters was not desired, but once
Q.13) Ans: C they were born, they were treated with
Exp: kindness and consideration. Women had a
 Statement 1 is incorrect : The political conflict position of honour in the household. Women
among the Mahajanapadas ultimately led to were given equal opportunities as men for
the emergence of Magadha as the most their spiritual and intellectual development.
powerful state and the centre of a vast empire. There were women poets like Apala,
Magadha’s rise to political supremacy began Viswavara, Ghosa and Lopamudra during the
with Bimbisara, who belonged to the Haryanka Rig Vedic period. Women could even attend
dynasty, and ruled for 52 years, from 544 to the popular assemblies. There was no child
marriage and the practice of sati was absent.
492 BCE. In the Mahavamsa, it is stated that he
 Statement 2 is incorrect : The earliest life of
was anointed as the king by his father at the
the Aryans seems to have been mainly
age of 15, which suggests that he was not the
pastoral, agriculture being the secondary
founder of his dynasty. He was a
occupation. They owed their prosperity to
contemporary of both Buddha and Mahavira, cattle wealth. Fought wars to gain supremacy
and paid equal respect to them.He pursued a of cattle wealth. They were well acquainted
three-pronged policy, namely, matrimonial with the sowing and harvesting procedures
16

(evident from the excavated wooden Brahmana, a daughter has been described as a
ploughshare), they had knowledge about source of misery.
various seasons also. Agriculture was mostly to
Q.16) Answer: A
produce fodder.
 Option A is correct: The Aryans were also
 Statement 3 is correct : The horses played the
known as manushi praja, who worshipped
most significant role in their lives.Its swiftness
Agni and who sometimes set fire to the house
enabled them to make successful ventures to
of black-skinned people. The Aryans and their
new places.The domesticated horse appears in
enemies can be identified by the colour of their
the 6th millennium B.C in the Black Sea and the
skin.
Ural mountains area.Archaeological evidence
 Option B is incorrect: Almost all the Vedic
of the horse and cremation appears in Swat
Gods are associated with horses, and this is
Valley,Pakistan.
particularly true of Indra and his fighting
Q.15) Answer: A companions, the maruts. The horse
 Statement 1 is correct - Gargi, Maitreyi and symbolizes strength and is generally employed
Katyayani composed hymns and had scholarly as a metaphor for might. The Vedic people
expertise. Early vedic society was male prayed for horses in addition to praja and pasu.
dominated. Birth of daughters was not desired  Option C is incorrect: Cattle sacrifices of the
during the early Rig Vedic period, but once Vedic period were called pasubandha, the
they were born, they were treated with cattle rearers offered meat to their Gods and
kindness and consideration. Their education in return asked for cattle wealth, so that their
was not neglected. Women attended the practice of sacrificing cattle and eating meat
meetings of the Vidatha. Girls were free to continued.
choose their life partners. There are no  Option D is incorrect: Wheat in the Vedic
instances of child marriage, sati, or purdah in period was known as godhuma. Other crops
the Rig Veda were barley (yava), beans, sesamum(tila)
 Statement 2 is correct - During the Early vedic and vrihi-rice.
period Girls enjoyed a lot more freedom than
the later vedic period. They were married long
Q.17) Answer: C
after they had reached puberty and there
seems to have been considerable freedom in  Statement 1 is correct. Only voluntary
the selection of husbands. There was no offerings (Bali) were taken in the Rig Vedic
purdah system. Period whereas the system for tax Collection
was established in the Later Vedic period. The
 Statement 3 is correct - Remarriage of widows
officers called sangrihitri were appointed for
was permitted and Niyoga or Levirate(a dead
that purpose. Vaishyas were the only tribute
man’s brother or next of kin marrying the
payers in Later Vedic times.
widow) was practiced by some. No instances of
child marriage or purdah system. Though  Statement 2 is correct. : In the later vedic
monogamy was the rule but polygamy was also period life became sedentary and
permitted. Instances (Maruts marriage with the domestication of animals and cultivation
Rodasi) of polyandry were also there. increased. The idea of private possession of
lands started taking shape. Trade was also
 Statement 4 is correct - Status of women
boosted. Thus the barter system in the early
declined in the later Vedic period and there
vedic period was replaced by the exchange of
was social stratification based on gender lines.
gold and silver in the later vedic period.
Their participation in ceremonies was
restricted. They were stripped of the right to  Statement 3 is incorrect : The Kings in Rig
attend assemblies. According to Aitareya vedic time did not possess any standing
army. In the later Vedic Period, Territorial
17

expansion led to certain rivalry among the Q.20) Answer: A


different groups, a still standing army was not  Statement 1 is correct: Rajgir was also called
possessed by the king in the Later Vedic as Girivraja as it was surrounded by five
period and gramas mustered in times of war. hills and any opening was closed by the stone
walls. Hence, it was entirely enclosed and
impregnable and was difficult to invade with
Q.18) Answer: A
the level of military technology present during
 Statement 1 is Correct: The Rig Vedic economy that time. However, with the use of cannons,
was clan based and pastoral in the city could be invaded and hence, its
nature. Evidence of trade and commerce is strategic importance began to decline.
meagre, and there was no concept of private
property based on land ownership. The clan as
a whole enjoyed rights over the resources.
 Statement 2 is Incorrect: Though agriculture
during the Rig Vedic period was primitive, the
Rig Vedic people were aware about the
concepts of soil fertility. The Rig Vedic hymns
refer to the levelling of fields for
cultivation, the desire for fertile fields
(urvara), and furrows (sita) drenched by rain,
producing rich harvests.
 Statement 2 is incorrect: The Magadha society
Q.19) Answer: A was unorthodox in character. It was mainly
 Statement 1 is incorrect: Anguttara Nikaya, a inhabited by the Magadhas and Kirtas who
buddhist text, mentions the presence of the were regarded lowly by the orthodox
16 Mahajanpadas or the large states. They Brahmins. The rulers of Magadha were more
extended to the North of the Vindhya range inclined towards military conquests and hence
from the North-west frontier to Bihar. Of it emerged as the first large kingdom by
these, Kosala and Magadha were the two defeating the other kingdoms.
powerful ones. Magadha emerged into power
during the rule of the Haryanka Dynasty and Q.21) Answer: A
its ruler Bimbisara. He was succeeded by his
 Statement 1 is correct: As per the Buddhist
son Ajatshatru. The capital of the Magadha text, Anguttara Nikaya, there was the
then was Rajgir. The last ruler of this existence of 16 Mahajanapadas. The period
dynasty, Udayin , was succeeded by
of 6th Century B. C. was regarded as the era
the Shishunaga dynasty. Avanti was the of Second Urbanisation in India.
biggest rival of Magadha and had capitals
at Ujjain and Mahishmati. This rivalry ended
with the defeat of the Avanti by the
Shaisungas. Hence, it was not Ajatshatru rather
Shaishungas that brought an end to Avanti.
 Statement 2 is correct: The importance of
Avanti lies in the presence of good quality iron Among
weapons and the extensive use of smelting and the 16 Mahajanapadas or the large
forging to smelt the iron ore. states, Kosala and Magadha emerged as the
most powerful ones. The idea of territorial
importance increased manifolds as the people
18

showed greater allegiance to (godhuma). Cattle rearing was an


the Mahajanapadas they belonged to as important economic activity for the Aryans,
against that of the Jana or tribe. although they practiced agriculture. There
 Statement 2 is incorrect: The record of this was no private property on land.
period was obtained from a number of Pali
and Sanskrit texts and hence, there was no Q.24) Answer: C
disappearance of the Pali texts. The Buddhists  Statement 1 is correct: Although people
texts, Sutta, Vinaya, and Abhidhamma during the Rig Vedic period practiced
pitaka were composed in Pali language. agriculture, Cattle rearing was an important
economic activity for them and Cattle were
The Jataka tales are written in both Pali and
considered wealth.
Sanskrit language.
 Statement 2 is correct: The Rig Veda
mentions artisans such as carpenters, chariot-
Q.22) Answer: D makers, weavers and leather-workers. Copper
 Statement 1 is incorrect: Rig Vedic Samhita is metallurgy was among the important
the earliest text that relates to the Early Vedic developments of this period. The term ‘ayas’
period. The Early Vedic culture is in the Rig Veda used refers to copper and
placed between 1500 BCE and 1000 BCE. bronze. Karmara, smith, is mentioned in
The political, social and economic aspects of the Rig Veda. Similarly, there are references
life of this period are reflected in the Rig Vedic to siri or yarn, indicating spinning which was
hymns. done by women and to carpenters, takshan.
 Kinship was the basis of the social Weaving of clothes of wool is also referred to
structure of Rig Vedic society. People were and obviously it was necessary in the cold
identified with specific clans and the clans weather. Some of the crafts were fulltime
formed the tribe or jana. The term jana occurs crafts, involving specialists.
in the Rig Veda 21 times but janapada does  Statement 3 is correct: Trading activities
not occur even once. The term vis, which refers were limited though traders were present
to the common people, occurs 170 times and during the Early Vedic period. Panis are
they lived in gramas (villages). referred to as traders and they were
 Statement 2 is incorrect: The family (griha) perhaps caravan traders. The word pan means
was the main social unit within the tribe. It barter, which was a mode of exchange. Nishka
was headed by the grihapati and his wife was was a gold or silver ornament used in barter.
known as sapatni. And the family at  The term for war in the Rig Veda was
that point of time was perhaps a joint family. gavishthi which means search for cows.
The donations to the priests were
mainly cows and women slaves but not land,
Q.23) Answer: D which reveals the importance of pastoralism.
 Archaeological evidence highlights There was no private property on land.
the development of agriculture among the Rig
Vedic people. The ploughshare is Q.25) Answer: D
also mentioned in the Rig Vedas. The field was  The Rig Vedic society was a tribal society.
known as kshetra and the term krishi referred The chief of the tribe was the main political
to ploughing. head and he was known as rajan. They
 The terms langla and sura referred to plough offered gifts of cattle, chariots, horse
and the term sita meant the furrow created ornaments and gold to the priests. Rajan was
by ploughing. Water for irrigation was a hereditary chief and was perhaps elected by
probably drawn from wells by cattle-driven the assembly called samiti.
water-lifts using pulleys.  Tributes and booty collected from war were
 People had knowledge of different seasons, redistributed by the king. There are also
sowing, harvesting and threshing. They references to gift of dasas or slaves. The
cultivated barley (yavam) and wheat king Trasadasyu, the chief of the Purus, gave
19

away 50 women as a gift. The chief was known settled life leading to formation of territorial
as gopa or gopati which means, chief of cattle. units. The term janapada, referring
to territory, is found in the Brahmanas dated
to ca. 800 BCE. There are more than 1000 sites
Q.26) Answer: C of painted Grey Ware culture in this
 Statement 1 is correct: During the Early Vedic area, suggesting that new settlements came
period trading activities were limited though up and the Upper Ganga Valley was densely
traders. Panis are referred to populated.
as traders and they were perhaps caravan  People lived either in mud-brick houses or
traders. The word pan means barter, which houses with wattle and daub walls. The
was a mode of exchange. Nishka was a gold or foundations for the towns must have emerged
silver ornament used in barter. The danas and during the later Vedic period. This was a period
dakshinas offered to people were means of of intense interactions. The term
redistributing resources. The dakshina was nagara, referring to commercial quarters,
both a fee for a specific service and also a is found in the later Vedic texts.
means of distributing wealth. The distribution  Statement 3 is correct: However, large towns
of cows helped spread pastoral appeared only at the end of the Vedic period.
activities and economic production. The sites of Hastinapura and Kausambi are
 Statement 2 is correct: The polity of the Rig considered proto urban (urban-like)
Vedic period was tribal in character. The chief settlements. The material culture of this
of the tribe was the main political head and he period shows more diversity and is
was known as rajan. They offered gifts of an improvement over the Early Vedic period.
cattle, chariots, horse ornaments and gold to It can be surmised that there was
the priests. Rajan was a hereditary chief. He surplus production to support various classes
was perhaps elected by the assembly called such as chiefs, princes and priests.
samiti. The main duty of the king was
to protect the tribe. He protected wealth,
fought wars, and offered prayers on behalf of Q.28) Answer: D
gods. The king had authority over the territory  The mahajanapadas on the Gangetic plains
and people. were all monarchies. Vedic orthodoxy was
an established practice in these kingdoms.
The priestly class enjoyed a preeminent
Q.27) Answer: B status in the mahajanapadas unlike in
 Statement 1 is correct: Iron was an important the gana-sanghas. The kingdoms
metal used for implements in this period. It were governed by kings and
was called syama-ayas or krishna-ayas or the administration was centralised. The
dark metal. Iron is believed to have played brahmin priests provided legitimacy to the
an important role in the conversion of the king through various rituals.
forests of the Ganga Valley into agricultural  The kingship was hereditary and the
lands. By the end of Vedic period, succession was in most cases based on the law
the knowledge of iron had reached eastern of primogeniture. The king was assisted by
Uttar Pradesh and Videha. councils called parishad and sabha.
 Earlier it was believed that iron originated The councils were advisory in nature. The king
around 700 BCE, but recent research dates the appropriated the agricultural
beginning of iron to around 1200 BCE or even surplus through land revenue apart from
earlier. The early views gave excessive a few other taxes.
emphasis to iron to the colonization of the  Bali was a tax imposed based on the area of
Ganga Valley, but new scholarship argues cultivable land. Bhaga was obtained as a share
that iron was not the only factor behind the of the produce. Kara and Shulka Were some of
expansion of the population. the other taxes collected during this period.
 Statement 2 is correct: With the intensification
of agriculture, the Later Vedic people led a
20

Q.29) Answer: C wealth. Gradually, some states acquired


 The sixth century BCE is often regarded as standing armies and maintained regular
a major turning point in early Indian history. It bureaucracies. Others continued to depend
is an era associated with early states, cities, on the militia, recruited, more often than
the growing use of iron, the development of not, from the peasantry.
coinage, etc.
 Statement 1 is correct. It also witnessed the
growth of diverse systems of thought, Students Note:
including Buddhism and Jainism. Early
Buddhist and Jaina texts mention, amongst
other things, sixteen states known as
mahajanapadas. Although the lists vary, some
names such as Vajji, Magadha, Koshala, Kuru,
Panchala, Gandhara, and Avanti occur
frequently. Clearly, these were amongst the
most important mahajanapadas. While most
mahajanapadas were ruled by kings,
some, known as ganas or sanghas, were
oligarchies, where power was shared by a
number of men, often collectively called rajas.

 Statement 2 is correct: Each mahajanapada


had a capital city, which was often
fortified. Maintaining these fortified cities as
well as providing for incipient
armies and bureaucracies required
resources. Brahmanas began composing
Sanskrit texts known as the Dharmasutras.
These laid down norms for rulers (as well as for
other social categories), who were ideally
expected to be Kshatriyas. Rulers were
advised to collect taxes and tribute from
cultivators, traders, and artisans.
Raids in neighboring states were recognized as
a legitimate means of acquiring
21

Jainism & Buddhism


Community’. Later it led to development of the
Q.1) Answer: B Mahayana form of Buddhism.
 Bhagavati Sutra: Vyakhyaprajnapati is  Sub-schools of Mahasanghikas -
commonly known as Bhagwati Sutra. It is an Lokottaravadins, Ekavyavaharikas, Kaukkutikas
important book of Jaina literature. It contains etc.
details about Questions answered by  Sub-Schools of Sthaviravada - Sarvastivadin,
Mahavira. It also gives a different list of sixteen Sammatiyas, Vatsiputriyas, Sautrantikas,
Mahajanapada. Theravadins etc.
 Anguttara Nikaya: The Buddhist literature
Anguttara Nikaya gives a list of sixteen great Q.3) Answer: C
kingdoms called ‘Sixteen Mahajanapadas’. The  Option A is correct: The primary cause for the
Sutta Pitaka, which contains the teachings of rise of Jainism and Buddhism was the religious
Buddha, is divided into five groups or Nikayas unrest in India in the 6 th century B.C.
(Digha, Majhim, Sanyukta, Anguttar and Superstitious beliefs and mantras confused the
Khuddak). The Mahavastu is a text of people.
the Lokottaravada school of Early Buddhism  Option B is correct: The complex rituals and
also mentioned about 16 mahajanapadas. sacrifices advocated in the Later Vedic period
 Indica: Megasthenes was the Greek were not acceptable to the common people.
ambassador in Chandragupta Mauryans court. The sacrificial ceremonies were also found to
His account Indica gives details about the be too expensive.
Mauryan administration, particularly the  Option C is incorrect: The growth of trade led
administration of the capital city of to the improvement in the economic
Pataliputra,the military organization and conditions of the Vaisyas. They wanted to
contemporary social life. enhance their social status and hence they
 Arthashastra: Authored by Chanakya. It is started supporting Buddhism and Jainism.
divided into 3 parts: First deals with the king  Option D is correct: People wanted a simple,
and his council and the departments of short and intelligible way to salvation for all
government; the second with civil and criminal people. Such religious teaching should also be
law; and the third with diplomacy and war. in a language known to people. This need was
fulfilled by the teachings of Buddha and
Q.2) Answer: A Mahavira.
 Buddhism:
 During the Second Buddhist Council at Vaishali Q.4) Answer: D
the Buddhist Order split into two sects  Pair 1 is Incorrect - First Buddhist Council was
namely Sthaviravada or ‘Believers in the held in Rajgriha (483 BC). It was convened
Teachings of the Elders’ and the other known by Ajathsatru (Haryanka Dynasty) and headed
as Mahasanghikas or ‘Members of the Great by Mahakassapa. In this council, Upali recited
the Vinaya Pitaka. Ananda recited Sutta
Pitaka. Council was held soon after the death
of the Buddha.
 Pair 2 is Incorrect - Second Buddhist
Council: Vaishali (383BC) was convened by
Kalashoka (Shisunaga Dynasty) under the
Chairmanship of Sabakami. The Buddhist
Order split into two sects Sthaviravadin or
‘Believers in the Teachings of the Elders’ and
the other known as Mahasanghika or
‘Members of the Great Community’.
 Pair 3 is Correct - Third Buddhist
Council: Pataliputra (250 BC). Chairman of this
22

council was Mogaliputta Tissa which was the brahmanas and kshatriyas. So they sought
convened under the king Ashoka (Mauryan a religion that would improve their position.
Dynasty). Sthaviravadins gained hold over  Statement 4 is correct: There was also a strong
buddhism. The last section called “Kathavatthu reaction against various forms of private
'' was added to Abhidhamma Pitaka. property. Old-fashioned people did not like the
 Pair 4 is Correct - Fourth Buddhist use and accumulation of coins of gold, silver.
Council: Kashmir (98AD) Chairman of this They disliked the new dwellings and clothes,
council was Vasumitra and was convened new luxurious systems of transport, and
during the reign of Kanishka disliked war and violence. The new forms of
(Kushana Dynasty). Buddhism is divided into property created social inequalities and
Hinayana and Mahayana. caused misery and suffering to the mass of
ordinary people. Therefore the common
Q.5) Ans: D people yearned to return to a primitive
Exp: lifestyle, both Jainism and Buddhism
 Statement 1 is correct: Numerous religious propounded simple, puritan, ascetic living.
sects arose in mid-Gangetic plains in sixth-fifth
centuries BC and it is believed they were 62 in Q.6) Ans: B
number. Jainism and Buddhism were the most Exp:
important. About Pavarana:
o Post Vedic society was clearly divided into  Pavarana is a Buddhist holy day celebrated on
four varnas: Brahmans, the full moon of the eleventh lunar month. It
Kshatriyas,Vaishyas and Shudras, which marks the end of the month of Vassa,
was based on birth and the two higher sometimes called "Buddhist Lent."
varnas(Brahmanas and Kshatriyas)  This day marks the end of the rainy season in
captured power, prestige and privileges at some Asian countries like Thailand, where
the cost of the lower varnas. Theravada Buddhism is practised. On this day,
o Varna divided society seems to have each monk must come before the community
generated tensions. The kshatriya of monks and atone for an offence he may
reaction against the domination of the have committed during the Vassa.
brahmanas, who claimed various
privileges,was one of the causes of the Q.7) Ans: A
origin of new religions. Vardhamana Exp:
Mahavira, who founded Jainism and  Statement 1 is correct: Jainism recognized the
Gautama Buddha who founded Buddhism, existence of Gods but placed them lower than
belonged to the kshatriya clan, and both the jina.
disputed the authority of Brahmanas.  Statement 2 is correct: Jainism did not
 Statement 2 is correct: The spread of the new condemn the varna system as Buddhism did.
agricultural economy in the north-eastern According to Mahavira, a person is born in
India, also led to the rise of new religions. The high or in a lower varna as a consequence of
iron tools began to be used in this area. The his sins committed or virtues acquired by him
use of iron tools made possible clearance of in his previous birth. In his opinion, by leading
forests, agriculture and large settlements. The a pure and meritorious life, members of lower
iron ploughshare required the use of bullocks. castes can achieve liberation. Jainism
However the Vedic practise of killing cattle particularly aims at the attainment of freedom
indiscriminately in sacrifice hampered the from worldly bonds.
progress of new agriculture.  Statement 3 is incorrect: Jainism made the
 Statement 3 is correct: The use of punch first serious attempt to mitigate the evils of
marked coins in the fifth century BC facilitated the varna order and the ritualistic Vedic
trade and commerce, which added to the religion. The Early Jainas discarded the Sanskrit
importance of Vaishyas. In the Brahmanical language principally patronised by the
society the vaishyas were ranked third after Brahmanas. They adopted Prakrit, the
23

language of the common people, to preach


their doctrines. Their religious literature was
written in Ardhamagadhi and the texts were
eventually compiled in 6th century AD in
Gujarat at Valabhi.

Q.8) Ans: B
Exp:
 Statement 1 is incorrect: Vajrayana is a form
of Buddhism probably originated from  Statement 2 is correct: According to Jainism,
Mahayana Buddhism. It is based on a complex Asceticism and penance are required to free
philosophical and ritual system meant to oneself from the cycle of karma. This can be
provide a path towards enlightenment. The achieved only by renouncing the world;
term Vajrayana roughly means “the way of the therefore, monastic existence is a very
diamond”. The word Vajra refers to the necessary condition of salvation.
diamond-hard thunderbolt that was used as a  Statement 3 is correct: According to Mahavira,
weapon by the Hindu god of thunder and rain, a person is born in a higher or lower varna as a
Indra. Yana refers to the way or the spiritual consequence of his sins committed or virtues
vehicle for achieving enlightenment. acquired by him in the previous birth and the
o Vajrayana Buddhism developed in India cycle of birth and rebirth is shaped through
around the 6th or 7 century CE. By the these karma’s.
seventh century CE, the Buddhist o In his opinion, by leading pure and
monasteries had come to be dominated by meritorious life, members of the lower
ease-loving people and became centres of castes can achieve liberation. No ritual is
corrupt practices which had been necessary for such liberation.
prohibited by Gautama Buddha.
 Statement 2 is correct: Tantric ideas began to Q.10) Ans: B
be incorporated into the teachings of Buddhist Exp:
universities in Northern India, generally About Buddha’s Sangha:
leading to the development of this new age  Those who followed the Buddha came to be
tradition. Vajrayana Buddhism is full of called the Sangha—the community of monks
symbolism and rites, mostly focused on the (bhikkhus) and nuns (bhikkhunis). Gautama
search of enlightenment. The rites are often Buddha organised the sangha or the religious
meditation that includes tantric practices, order, whose doors were open to all
combining the physical and spiritual worlds. irrespective of caste, creed and sex. Those
who became monks and nuns underwent an
Q.9) Ans: B ordination ceremony of shaving their heads
Exp: and donning robes to symbolise world-
 Statement 1 is incorrect: The Jainas believed renunciation. They entered into the homeless
that their most important religious teacher life of wandering monastics who settled down
Mahavira had twenty-three predecessors in monasteries only during the months of the
who were called Tirthankaras. Some Jainas rainy season.
believe that Rishabhadev was the first  The Monks were required to observe the rules
tirthankara. The earliest important teachings and regulations of the Sangha faithfully. Once
of Jainism are attributed to Parshvanatha, the they were enrolled as members of the
twenty-third tirthankara. BUddhist church, they had to take vow of
o So the basic philosophy of the Jainas was continence, poverty and faith.
already in existence in north India before  The doors of Sangha were not opened to
the birth of Vardhamana Mahavira in the slaves, soldiers, and debtors. Women were
sixth century BC. admitted to the Sangha and thus brought on
par with men.
24

Q.11) Ans: A enlightened mind, is made of four sections:


Exp: the square base, the dome, the cone, and the
Great events of Buddha life: canopy. They represent the four elements:
 Avakranti: Buddha’s conception or descent, earth, water, fire and air, completed by space,
known as Avakranti, is symbolised by a white signification given to the total volume of the
elephant. This symbol is found in a variety of stupa.
Aśoka’s remains, along with other animals:
bull, horse and lion. A narrative at Bharhut Q.12) Answer: C
showing Queen Mayadavi’s(mother of  The third Buddhist Council was held in 240
Siddhartha Gautam) dream, a descending BC at Pataliputra. Moggaliputta Tissa was its
elephantis shown. The queen is reclining on chairman and Asoka was its royal patron. Its
bed whereas an elephant is shown on the top objective was to reconcile the different schools
heading towards the womb of Queen of Buddhism and to purify the Buddhist
Mahadevi.
movement, particularly from opportunistic
 Jati (Birth): Buddha’s Jati, or birth, is usually
factions which had been attracted by the royal
represented by a lotus. This element
patronage. The final version of Tripitakas was
symbolises Buddha’s purity, his diving birth,
or his spiritual ascent. It is sometimes used to completed in this council.
represent a “tree of life and fortune”. The  Menander was a Indo-Greek ruler, who ruled
Lotus flower is occasionally laden with fruits over the North-western part of India
and jewels, symbols of countless blessings. between 165 BCE to 145 BCE. He was
 Mahabhinishkramana(Great Renunciation): converted to Buddhism by Nagasena.
Buddha left his home at the age of 29 years Menander asked Nagasena many questions
to live a life of an ascetic which is known as relating to Buddhism. These questions and
Mahabhinishkramana (the great Nagasena’s answers are recorded in the form
departure).He leaved his palace with his of a book known as Milinda Panho or the
charioteer named Channa and his horse called
question of Milinda.
Kanthaka. This event is symbolised by the
 The Shaka Era (Shakar Samvat) was started by
Horse.
 Nirvana/ Sambodhi (enlightenment): Kushan Ruler Kanishka in 78AD, after
Buddha’s Sambodhi, or enlightenment, is defeating a Shaka king. In 1957, the Shaka
represented by the Bodhi tree. This Pipal tree Samvat was adopted as the national calendar
(Indian fig) is used to represent his of India.
illumination because it is under one of them  Vijayalaya Chola was the founder of the
that he attained Nirvana, in the locality now medieval chola empire in the 8th century by
known as Bodhgaya. The Asvatha tree defeating Pallavas. He was a feudatory of
symbolises an abode of folk deities or spirits. Pallavas. He captured Tanjore in 850. By the
 Dharmachakra Parivarthana (first Sermon): end of the 9th century the Cholas had defeated
The Dharmachakrapravartana, or “Turning of both the Pallavas of Kanchi and weakened the
the wheel of law”, his first sermon, is
Pandyas, bringing the southern Tamil
represented by the Dharma Wheel (“Wheel of
(Tondamandala) country under their control.
Law”). He delivered his first sermon known as
Dhammacakkappavattana Sutta (turning of the
Wheel of Law) at Sarnath in which he dealt
with the Four Noble Truths and the Noble Q.13) Ans: B
Eightfold Path, the very base of Buddhist Exp:
teaching.  The Tipitaka (in Pali) and Tripitaka (in
 Mahaparinirvana (Death): Buddha’s Sanskrit) meaning ‘The Three
Mahaparinibbāna or passing away, is usually Baskets/Collections’ is one of the earliest
symbolised by a stūpa, where his remains compilations of Buddhist teachings written
were kept. This construction, a sign of the
(originally) on long, narrow leaves. All
25

branches of Buddhism have the Tripitaka as  Statement 2 is correct: A large number


part of their core scriptures, which comprises of nucleated settlements was observed during
three books — the Sutta (conventional the time of the Buddha in the rural areas.
teaching), the Vinaya (disciplinary code), and The Pali literature talks of the nucleated
the Abhidhamma (moral psychology). settlements in which people along with their
land and property lived away from the
main settlements in the Gangetic plains. The
text also talks of the three types of villages
namely, the typical kind, the craft villages, and
the border villages.
 Statement 3 is incorrect: During this time, the
tax collection was done mainly by the royal
agents directly and hence there were no
 Statement 1 is correct : The Sutta Pitaka intermediaries on a large scale. One-sixth of
(Sutra/Basket of Discourses): This contains the the total produce was collected as taxes from
Buddha’s discourses on various doctrinal the peasants directly.
issues in dialogue form. These texts are also  Statement 4 is correct: In the eastern Uttar
Pradesh and Bihar, rice served as the main
known as ‘Buddhavacana’ or ‘the word of the
cereal and was grown on a large scale. A
Buddha’, as it refers to texts that are supposed
number of paddy fields have been described in
to contain what the Buddha himself said.
the Pali and Sanskrit texts. Wet paddy
 Statement 2 is incorrect : The Abhidhamma production or paddy
Pitaka (Basket of Higher Teachings): This transplantation emerged on a large scale
contains a thorough study and systemisation of during this time.
the teachings of the Sutta Pitaka through
summaries, questions and answers, lists, etc. Q.15) Answer: A
 Statement 3 is incorrect : The Vinaya Pitaka  Statement 1 is incorrect: The people during
(Discipline Basket): This contains rules for this time were divided into four
monks and nuns of the monastic order varnas- Brahmanas, Kshatriyas, Vaishyas, and
(Sangha). It includes the Patimokka – a list of Shudras. The Dharmasutra provides a clear
transgressions against monastic discipline and mention of the duties and the civil and criminal
atonements for these. Apart from monastic laws relating to each varna. The crime
against Brahmin and Kshatriyas was treated
rules, the Vinaya texts also include doctrinal
with heavy punishment while the punishment
expositions, ritual texts,biographical stories,
for the upper varnas was lighter.
and some elements of the ‘Jatakas’ or ‘birth
The Shudras were accorded the lowest status
stories’.
with no legal or religious rights. They were
meant to serve the upper varnas. They were
Q.14) Answer: B denied the Upanayana or the holding of the
 Statement 1 is incorrect: The trade during sacred thread.
the vedic times was largely barter in  Statement 2 is correct: The laws and rules as
nature and it was during the time of Buddha compiled in the Dharmasutra varied with
that a large number of silver and some
each Varna. Shudra were considered to have
copper coins came into circulation. These
emerged from the feet of the creator and
coins were used for the trade and were punch
marked with the images of hills, bulls, trees, hence were accorded the lowest position with
etc. no legal or religious rights.
26

Q.16) Answer: C  Statement 2 is correct: Theravadins is


 Option A is correct: Several interpretations a subsect under Hinayana Buddhism. It deals
of Chaitya have been found. In some, it is with the cessation of suffering and attainment
considered as a site for those surrounded by of Nirvana. It does not believe in the existence
nature with a shrine meant for the purpose of of a Savior like the Mahayana Buddhism which
worship. believes in Bodhisattvas that would liberate all
human beings.
Vaishnavism believes in the presence of a
Savior. The incarnations of Lord Vishnu were
meant to save the world whenever it was in
danger. The last avatar, Kalki, to end the
demon Kali, is yet to come as per the
mythologies.

Q.18) Answer: B
 Buddha, Dhamma, and Sangha are
 Option B is correct: The word Chaitya has considered as the three jewels or triratnas of
been derived from the word Chita which Buddhism.
means the funeral pyre and hence is also
regarded as the extension of the funeral
mound.
 Option C is incorrect/ least appropriate: A
place meant for debates and discussion has not
been regarded as a chaitya.
 Option D is correct: In Buddhist
literature, Chaitya also refers to the sacred
places associated with the life of Gautam
Buddha, i.e., Lumbini, where he was
born, Bodh Gaya, where he received  Buddha refers to as the Enlightened
Enlightenment, Sarnath, where he delivered being, Dhamma comprises his teachings and
his first Sermon, and Kushinagar where he doctrines, while Sangha denotes the monastic
attained Mahaparinirvana. order.

Q.17) Answer: C Q.19) Answer: A


 Statement 1 is correct: After  Statement 1 is correct: Sthaviravada also
the Mahaparinirvana of Gautam Buddha, referred to as the Theravada is a subsect
Buddhism was divided into Mahayana and under Hinayana Buddhism. It focuses on the
Hinayana at the fourth Buddhist Council in teachings of Buddha as its central
Kashmir. theme. Theravada refers to the school of older
o The Hinayana is an orthodox form of monks. Their ultimate aim is to end suffering
Buddhism and believes in the original and attain Nirvana. Practices
preaching of Buddha. It does not believe in like Samatha and Vipassana form an
idol worship. Mahayana Buddhism on the important part of Theravada tradition. Like the
other hand, deals with idol worship and Hinayana tradition, they used Pali as their
believes in the concept of main language.
Bodhisattvas (one that imbibes the Statement 2 is incorrect: Lotus Sutra is an
Buddhist nature). important text under the Mahayana
27

Buddhism. It deals with six perfections  Statement 4 is incorrect: Gautam Buddha was
or paramitas to be followed by an a contemporary of Bimbisara and Ajatshatru.
individual: Dana, Sila, Ksanti, Virya, He was not a contemporary of Ashoka.
Dhyana, and Prajana. Mahavamsa is another
important text related to Mahayana Extra Edge By PW Only IAS
Buddhism. It deals with the history of
Buddhism in Sri Lanka. ● In Mahayana, Buddhism Bodhisattva is a
person who can reach nirvana but delays
Q.20) Answer: C doing so through compassion for suffering
 Statement 1 is correct: The Third Buddhist beings.
● It is similar to the concept of Incarnations in
Council was held at Pataliputra under the
Hindu Mythology. Bodhisattvas are common
patronage of King Ashoka and the
figures in Buddhist literature and art.
chairmanship of Moggaliputta Tissa. It was in
this Buddhist Council that the compilation
of Abhidhamma Pitaka took place.
 Statement 2 is correct: The fourth Buddhist Q.22) Answer: A
council was held in Kashmir under the
 Statement 1 is correct: Buddhism believes that
patronage of King Kanishka and the
the world is transient and soulless and is
chairmanship of Vasumitra. The division of impermanent. The world is full of sorrows and
Buddhism into Hinayana and Mahayana took sufferings caused due to desire. It prescribes
place in this Buddhist council. the path of moderation to
o The first Buddhist council was held attain nirvana unlike the Jain philosophy which
at Rajgir under the patronage of King believes in severe penance.
Ajatshatru and the chairmanship  Statement 2 is incorrect: According to
of Mahakasyapa. Sutta Pitaka and Vinaya Buddhism, social relations are the creation of
Pitaka came up during this council. human beings and not divine in origin and
The second Buddhist Council was held hence it is the duty of the human beings to be
at Vaishali under the patronage of King humane and kind to other human beings.
Kalasoka under the chairmanship Buddha emphasized that the people must
of Sabakami. work out their own salvation by way of right
actions and burn their ego and desire.
Q.21) Answer: B
 Statement 1 is incorrect: Padmapani is one of Q.23) Answer: A
the protective deities around Buddha in the  Option A is correct: A number of sculptures are
Ajanta Caves, who is regarded as depicted at Sanchi which are directly or
the Bodhisattva of Compassion. He is also indirectly related to Buddhism. The sculpture
known as Avalokitesvara and is holding a of a woman at the gateway is considered as a
Lotus flower in his hand. Vajrapani, sacred symbol and symbolizes the enrichment
of Buddhism with the cultural beliefs and the
also depicted in the Ajanta caves, is regarded
traditions of the non-buddhist people who
as the Bodhisattva of Power.
joined Buddhism. Shalabhanjika is found in
 Statement 2 is correct: Bodhisattva could be different religious sculptures in India, not
both male and female. Padampani is regarded necessarily Buddhism. It is a symbol of
as a female by some. a Yakshi (a benign deity associated with
 Statement 3 is correct: Vajrayana Buddhism fertility), holding a branch of a tree and is
has been influenced by Hinduism. It is also depicted in a sensuous form.
based on the Mahayana Buddhist philosophy
and believes in Tantra, Mantra, and Yantra for Q.24) Answer: B
the attainment of Nirvana.  Option B is correct: Kutagarashala was
28

referred to a place where travelling Q.27) Answer: B


mendicants/beggars halted. It was the place  Statement 1 is incorrect: Vardhana Mahavira
where debates and discussions regarding the is not regarded as the founder of Jainism. He
truth of life and its various philosophies were was 24th and the last Tirthankara of Jainism.
taken up. It is associated with Buddhism and is The first tirthankara Rishabhadev was born in
regarded as the Vihara where the Buddha
Ayodhya and is regarded as the one who laid
mostly halted while on his way to Vaishali. It is
the foundation of the ordered and organised
mentioned in the Pali Buddhist texts as a
monastery located in the middle country of society.
ancient India. It was a part of Vaishali.  Statement 2 is correct: Anekantavada or the
multiple dimensions of a truth is a core
Q.25) Answer: B doctrine of Jainism.
 The three principles of Jainism, also known as
Triratnas (three gems), are: 1. Right faith
2. Right knowledge 3. Right conduct.
 Right conduct refers to the observance of
the five great vows:
1. Not to injure life (Ahimsa)
2. Not to lie (Satya)
3. Not to steal (Asteya) According to this, a particular reality has
4. Not to acquire property (Aparigraha) multiple dimensions and there is no absolute
definition that can describe the ultimate truth.
5. Observe Chastity (Brahmacharya).
 Statement 3 is correct: The philosophy of
Eightfold Path (Ashtang Marg) given in Jainism deals with five constraints of life which
Buddhism includes: Ahimsa (nonviolence); Satya (truthf
(1) Right Views (2) Right Resolve ulness); Asteya (not to steal); Aparigraha (not
(3) Right Speech (4) Right Conduct to acquire property);
(5) Right Livelihood (6) Right Effort
and Brahmacharya (celibacy). Brahmacharya
(7) Right Recollection (8) Right Meditation.
was propounded by Mahavira. It
Q.26) Answer: B was Parshvanatha, the 23rd Tirthankara who
propounded the first four constraints.
 Statement 1 is incorrect: The second great
Jain Council was held at Valabhi during Gupta
period and the Jain Canon of the Swetambras
was written. The first Jain Council was
convened at Pataliputra by Sthulabahu, the Q.28) Answer: C
leader of the Digambaras, in the beginning of  Statement 1 is incorrect: Jainism was divided
the 3 rd century B.C. under two main
 Statement 2 is correct: The final compilation sects: Digambaras and Svetambaras.
of Jain literature called Twelve Angas Digambara school was propounded
(limbs) was completed in the second Jain by Bhadrabahu who moved to Karnataka with
council. his discipline predicting a long famine in
 Statement 3 is incorrect: Women were Magadha. Under the Digambara school of
admitted into the monastic order. However, as Jainism, the monks practice complete nudity
a woman one cannot attain salvation. But by unlike Svetambara who wear white clothes.
accumulating merit by good deeds, a woman Women monks are allowed in the Digambara
could be reborn as a man and then strive to sect and wear plain unstitched white sarees.
attain salvation. Women are not allowed to be Tirthankaras in
the case of Digambara.
29

 Statement 2 is incorrect: absolutism and vedic authority. According to


 Svetambaras were those who stayed back in Jains, there are many dimensions of the
the North. Sthulabhadra ultimate truth or reality and hence it focuses
was the exponent of this school. They follow on Anekantavada.
the teachings of Pashvanatha who gave the
Q.31) Answer: A
first constraints four:
 Statement 1 is incorrect: Buddhism does not
Ahimsa (nonviolence); Satya (truthfulness); As
acknowledge a supreme God or deity.
teya (not to steal); Aparigraha (not to acquire
Followers instead focus on achieving
property); while they do not follow
enlightenment. The path to enlightenment is
Brahmacharya. They wear simple white clothes
attained by
and believe that even women could be a
utilizing morality, meditation and wisdom.
Tirthankara. They believe that the 19th
Jainism recognised the existence of god but
Tirthankara, Mallinatha, was a female.
not as a creator and placed them lower than
Q.29) Answer: D Jina (Mahavira)
 Statement 1 is correct: Ajivika sect was  Statement 2 is incorrect: Division of
founded by Makkhali Gosala, who was the Buddhism into Hinayana and Mahayana
follower of Mahavira and later left him and happened in the fourth Buddhist Council
founded his own creed. Ajivikas deals (72AD) in reign of Kanishka, whereas
with Niyati or fate and believes that destiny is the division of Jainism into Digambara and
responsible for controlling every matter in the Shvetambara happened 200 years after the
universe. death of Mahavira (467 BCE at Pavapuri near
Patna) when there was a famine in Magadh.
 Statement 2 is correct: The concept
of Sunyata was given by Nagarjuna under Many Jainas migrated to the South under the
the Mahayana Buddhism. According to this, all leadership of Bhadrabahu, though the rest of
the phenomena and matter in existence are them stayed back in Magadha under the
devoid of nature, essence, and substance. leadership of Stahlbau. When the southerners
returned, differences emerged between both
 Statement 3 is correct: Jainism got divided into
factions. In order to sort out these differences
sects
and to compile the principal teachings of
namely: Digambara and Svetambara. The
Jainism, a council was convened in
Digambara sect was propounded
Pataliputra, but the southerners boycotted it
by Bhadrabahu while the Svetambara was
and refused to accept its decisions. From now
propounded by Sthulabhadra.
onwards, the southerners began to be called
digambaras and the Magadhans
Q.30) Answer: C svetambaras.
 Statement 3 is correct: Jainism did not
 Statement C is correct: Jainism believed that
all objects whether living or non-living possess condemn Varna, as Buddhism did, but
souls and must not be harmed. It believes attempted to mitigate the evils of the varna
in starvation, renunciation of the world, and order and the ritualistic Vedic religion.
strict asceticism. Hence, it rejects the practice According to Mahavira a person is born in a
of agriculture as it involves killing of insects and high or lower class as a consequence of the
other similar beings. Jainism hence believes sins or virtues acquired by him in the previous
in Starvation, Vedic authority, Existence of birth. Mahavira looks for human values even
soul, soul is the fundamental unit of Jain in a chandala. In his opinion, by leading a pure
philosophy. Buddhism on the other hand and meritorious life, members of the lower
focuses on the middle path and Jainism focuses castes can achieve liberation.
on the extremities. Jainism does not believe in
30

satisfy their religious cravings. The monks


Q.32) Answer: D and nuns could not afford to lose the cash
 Statement 1 is correct: Both Buddha (563BC- donations from the growing body of
483BC) and Mahavira (540BC-468BC) were Central Asian traders and artisans and thus
contemporary. Other contemporaries of now started accepting gold and silver,
Buddha took to non-vegetarian food and wore
include Bimbisar, Jivaka, Ajatshatru, Vasakar elaborate robes. Kanishka was the biggest
a, Amrapali, Prasanjit, Vatsaraja Udayan, Puk proponent of Mahayana Buddhism.
kusati, Chandrapradyot, Vasavadutta.  Option 3 is incorrect:
 Statement 2 is correct: In Jainism, o Even though Indian Astronomy and
‘Nirvana’ refers to the end of suffering and Astrology profited from contact with the
freedom from the cycle of life and Greeks, however, the Indians did not owe
birth. Enlightenment in Jainism is known as anything striking to the Greeks in
‘Kaivalya’. Whereas medicine, Botany and Chemistry. These
in Buddhism ‘Nirvana’ means attainment of three subjects were dealt with
‘enlightenment’ or ‘salvation’. Death is called by Charakha and Susruta.
‘Parinirvana’.  Option 4 is correct: The Central Asian rulers
 Statement 3 is correct: The Mathura school of also patronised and helped in the growth of
arts developed under the patronage of Sanskritliterature. The earliest specimen
Kushana rulers and was influenced of kavya style and that too in chaste Sanskrit is
by Buddhism, Jainism and Hinduism. The found in the Junagadh Inscription
Mathura School showed a striking use of of Rudradaman I. From now
symbolism in the images. The Hindu onwards, inscriptions began to be composed
Gods were represented using their avayudhas. in chaste Sanskrit although the use of Prakrit
For example, Shiva is shown through linga and continued for a couple of centuries.
mukhalinga. Buddha is shown to be
Q.34) Answer: A
surrounded by two
 Statement 1 is Incorrect: Two of the famous
Bodhisattavas – Padmapani holding a lotus
sculptures of the Mauryan period are: Yaksha
and Vajrapani holding a thunderbolt.
and Yakshi. The earliest reference of Yakshi
can be found in Silappadikaram, a Tamil text.
Q.33) Answer: C ● Statement 2 is Correct: Yaksha worship was
 Option 1 is correct: very popular before and after the advent of
o The Central Asian conquerors introduced Buddhism and it was assimilated in Buddhism
new styles in administration. For instance, and Jainism. Hence, all of the Jain
the Indo- Greeks introduced the practice of Tirthankars were associated with a
military governorship wherein they Yakshi. For instance, Yaksha Dharenendra and
appointed military governors called the Yakshi Padmavati are often shown near
strategos while the Kushanas introduced the Jain Tirthankar, Parshvanath.
the ‘Satrap’ system of government, under ● Statement 3 is Incorrect: Yaksha and Yakshi
which the empire was divided into were worshipped by ALL three religions:
Jainism, Hinduism and Buddhism.
numerous satrapis and each satrapi was
placed under the rule of a satrap.
Q.35) Answer: D
 Option 2 is correct:  Statement 1 is incorrect: Buddha
o The origin of Mahayana Buddhism owes a advocated avoiding the two
lot to the Central Asian influence. The extremes of indulgence in worldly
Central Asian Rulers did not appreciate pleasure and the practice of strict abstinence
the philosophical doctrines of Buddhism and asceticism. He ascribed instead of
and wanted something novel that could
31

the 'Madhyam Marg' or the middle Ardha Magadhi Prakrit script accompanied by
path which was to be followed. a Sanskrit.
 According to him, everyone was
responsible for their own happiness in life, Q.37) Answer: A
stressing upon the individualistic component  Statement 1 is correct: The famous Chinese
of Buddhism. pilgrim, Fahien visited India during the reign
 Statement 2 is correct: Buddhism believes in of Chandragupta II. Out of his nine years stay in
the transmigration of souls. The main India, he spent six years in the Gupta empire.
teachings of Buddhism are encapsulated in the He came to India by the land route through
basic concept of four noble truths or Khotan, Kashgar, Gandhara and Punjab.
ariyasachchani and eightfold path or He visited Peshawar, Mathura, Kanauj,
ashtangika marg. Sravasti, Kapilavastu, Kusinagara, Pataliputra,
 The essence of Buddhism is the attainment of Kasi and Bodh Gaya among other places. He
enlightenment. It points to a way of life that returned by the sea route, visiting Ceylon and
avoids self- indulgence and self-denial. There Java.
is no supreme god or deity in Buddhism.  Statement 2 is incorrect: The main purpose of
 Statement 3 is correct: The ultimate goal of his visit was to see the land of the Buddha and
Buddha’s teaching was the attainment of to collect Buddhist manuscripts from India. He
nirvana which was not a place but experience stayed in Pataliputra for three years studying
and could be attained in this life. Sanskrit and copying Buddhist texts. Fahien
provides valuable information on the
religious, social and economic condition of the
Q.36) Answer: A Gupta empire.
Some of the Major Buddhist texts are:
 Dipavamsa: Dipavaṃsa, (Pāli: “History of the
Island”), oldest extant historical record of Sri
Lanka, compiled in the 4th century. It is an
important early work in Buddhist and Pali
literature It details the tooth relic and Bodhi
Tree’s arrival in Sri Lanka. It also deals with the
arrival of Buddha’s teaching and preachers in
Sri Lanka. It mentions that Buddha visited
Kelaniya and Deegavapi in Sri Lanka.
 Therigatha: it is an anthology of poems in the  Statement 3 is incorrect: According to him,
Pali language by and about the first Buddhist Buddhism was in a flourishing condition in
women. It was composed more than two northwestern India but in the Gangetic
millennia ago. valley it was in a state of neglect. He refers to
 Ashokavadana: It is a Buddhist text related to the Gangetic valley as the land of
the legend of the Maurya Emperor Ashoka. It Brahmanism‟. Fahien mentions.
was written in the 2nd century CE. The
Ashokavadana is an account of the birth and
reign of King Ashoka. Students Note:
 Vessantara Jataka: Vessantara was a very
generous king who, in Buddhist mythology, is
considered to be a previous incarnation of the
Buddha Gotama.
Uttaradhyayana Sutta is a Jain text. It is one of
the four ‘Mulasutras’ of the canons of
Shvetambar Jain. The main text is written in
32

Mauryan Empire, Post Mauryan Age, Megaliths & Sangam Age

Q.1) Answer: C  Statement 4 is Incorrect - Chandragupta


Chandragupta Maurya was the founder of the Maurya defeated Selukas Niketar. Selukas
Mauryan Empire. For the first time, the political Nikator ceded some trans-Indus territories to
unity was achieved in India under the Mauryans. the Mauryan Empire and Megasthenes was
sent to the Mauryan court as Greek
ambassador.

Q.3) Answer: B
 Statement 1 is Incorrect- Ashoka’s policy
of Dhamma was a broad concept. It was a way
of life, a code of conduct and a set of principles
to be adopted by the people. Some ideas of
Ashoka’s Dhamma are similar to the teachings
of buddhism like non-violence etc but both of
Leaving Kerala, Tamil Nadu, and some parts of them are quite different from each other. The
north-eastern India, the Mauryas ruled over the concept of non-violence and other similar
whole of the sub-continent ideas of Asoka’s Dhamma are identical with the
The history writing has also become clear from this teachings of Buddha. But he did not equate
period due to accuracy in chronology and sources. Dhamma with Buddhist teachings. Buddhism
 The Mauryan Empire was divided into four remained his personal belief. His Dhamma
provinces with their capitals at Taxila, Ujjain, signifies a general code of conduct.
Suvarnagiri and Kalinga.
 The provincial governors were mostly
appointed from the members of the royal
family. They were responsible for the
maintenance of law and order and collection of
taxes for the empire.

Q.2) Answer: A
 Statement 1 is Incorrect - Chandragupta
Maurya was the founder of the Mauryan
Empire. He captured Pataliputra from  Statement 2 is Correct- Among many
Dhanananda, last ruler of the Nanda dynasty. principles of Dhamma, Tolerance among all
He was assisted by religious sects is one of the important
Kautilya/Chanakya/Vishnugupta. principles of Dhamma.
 Statement 2 is Correct - Bindusara supported  Statement 3 is Correct- He appointed officials
the Ajivikas sect. They are an ascetic sect that known as Dhamma Mahamatta who went
emerged in India about the same time as from place to place teaching people about his
Buddhism and Jainism. Its most important dhamma. Ashoka also got his messages
leader was Makkhali Goshala. The main inscribed on rocks and pillars, instructing his
philosophy of Ajivikas is that absolutely officials to read his message to those who
everything is predetermined by fate, or niyati, could not read it themselves.Ashoka also sent
and therefore, human action has no messengers to spread ideas about dhamma to
consequences. other lands, such as Syria, Egypt, Greece and
 Statement 3 is Correct - Bindusara had cordial Sri Lanka.
relations with the Syrian king Antiochus I. He
also received Deimachus as ambassador from
him.
33

Q.4) Answer: B Q.6) Ans: D


 Statement 1 is incorrect: The Mauryan Empire Exp:
was divided into four provinces with their  Agriculture flourished in the Gupta period due
capitals at Taxila, Ujjain, Suvarnagiri and to the establishment of irrigation works. Apart
Kalinga. The provincial governors were mostly from the state and individual cultivators,
appointed from the members of the royal Brahmins, Buddhists, and Jain sanghas
family. They were responsible for the brought waste lands under cultivation when
maintenance of law and order and collection of they were donated to them as religious
taxes for the empire. endowments.
 Statement 2 is correct: According to the  The Paharpur copper plate inscription
account of a Roman Writer Pliny, indicates that the king was the sole proprietor
Chandragupta Maurya maintained 600,000 of the land. Even when he made land grants,
foot soldiers and 30,000 cavalry. In order to he reserved his prerogatives over it. As stated
meet the expenses of such a huge army, the in Paharpur plates, an officer called ustapala
state controlled almost all the economic maintained records of all the land transactions
activities in the realm. The state brought new in the district and the village accountant
virgin land under cultivation which yielded preserved records of land in the village. During
handsome income to the state in the form of the Gupta period, the land was classified as
revenue collected from the newly settled detailed below.
peasants. o Kshetra: Cultivable land
 Apart from this, the state enjoyed a complete o Khila: Wasteland
monopoly in mining, sale of liquor, o Aprahata: Jungle or wasteland
manufacture of arms etc. This naturally o Vasti: Habitable land
brought money to the royal exchequer. o Gapata Saraha: Pastoral land
 The crops cultivated during the Gupta period
Q.5) Ans: C were rice, wheat, barley, peas, lentils, pulses,
Exp: sugarcane, and oilseeds. From Kalidasa, we
 Statement 1 is correct: Chandragupta II was a came to know that the south was famous for
great conqueror and an able administrator as pepper and cardamom.
well. His other names (as mentioned in coins)
include Vikrama, Devagupta, Devaraja,
Simhavikrama, Vikramaditya and Sakari. His Q.7) Ans: C
court had nine jewels or navaratnas, that is, Exp:
nine eminent people in various fields of art,  Statement 1 is incorrect: Samudragupta was
literature and science. the greatest of the rulers of the Gupta dynasty.
 This included the great Sanskrit poet Kalidasa, The Allahabad Pillar inscription provides a
the Sanskrit scholar Harisena, the detailed account of his reign. He was the
lexicographer Amarasimha, and the physician opposite of Ashoka. Unlike Ashoka, he
Dhanvantari. Fahien, the Buddhist scholar delighted in violence and conquest.
from China, visited India during his reign. He  Statement 2 is incorrect: Samudragupta was a
records the prosperity of the Gupta Empire. patron of many poets and scholars, one of
 Statement 2 is correct: Chandragupta II was whom was Harisena. Thus he must be credited
the first Gupta ruler to issue silver coins. His with a share in the promotion of Sanskrit
rule thus formed the peak period of Gupta’s literature and learning, characteristic of his
territorial expansion. The kingdom’s prosperity dynasty. Kalidasa is a court poet of
grew out of its trade links with the Roman Chandragupta II.
Empire. After establishing himself in eastern  Statement 3 is correct: Samudragupta
and western India, Chandragupta II defeated marched against the South Indian monarchs.
northern rulers like the Huns, Kambojas and The Allahabad Pillar inscription mentions that
Kiratas. Samudragupta defeated twelve rulers in his
South Indian Expedition (Dakshinapatha
34

expedition). He did not destroy and annex feudal traits are found in the administration of
those kingdoms. Instead, he defeated the the Satavahanas.
rulers but gave them back their kingdoms. He
only insisted on them to acknowledge his
suzerainty.

Q.8) Ans: D
Exp:
 Chandragupta-II (A.D. 380-412) was known for
his deep interest in art and culture and nine
gems or Navratna adorned his court. He
enjoyed the titles ‘Vikramaditya (the sun of
prowess) and ‘Sakari’. His reign is often  Statement 3 is correct : The lowest level of
remembered not for wars but for his administration was a grama (village), which
patronage of Art and Literature. The Court of
was under the charge of a gaulmika (village
Chandragupta was adorned by celebrated
headman), who was also the head of a military
scholars collectively known as ‘Navaratnas’.
Kalidasa, the great Sanskrit poet and regiment consisting of nine chariots, nine
playWright, flourished in his court. elephants, 25 horses and 45 foot soldiers.
 The two famous practitioners of Ayurveda in  Statement 4 is not correct : Unlike the gold
ancient India were Charaka (1st-2nd centuries coins of the Kushanas, Satavahanas mostly
C.E.) and Sushruta (c. 4th century C.E.) Charak issued lead coins (which they probably
Samhita, written by Charak is a remarkable
imported from the Romans) apart from copper
book on medicine. In his treatise, Susruta
Samhita, Sushruta speaks about elaborate and bronze coins. The Deccan developed a
surgical procedures. very advanced rural economy. The people
knew about the art of paddy transplantation
Q.9) Ans: A and especially the area on the confluence of
Exp: the rivers Krishna and Godavari formed a great
 Statement 1 is incorrect : In Deccan and in rice bowl.
central India, the Satavahanas succeeded the
Mauryas though after a gap of almost 100 Q.10) Ans: B
years. Simuka was the founder of the Exp:
Satavahana Dynasty and was immediately  Pulakeshin II (609 -642 A.D.) was one of the
active after Ashoka’s death. He is mentioned as greatest ruler of Chalukya dynasty who
the first king in a list of royals in a Satavahana converted the Chalukyan kingdom into an
inscription at Nanaghat.In the Puranas, the empire comprising most part of the Deccan
name of the first Andhra (Satavahana) king is plateau and extending its northern boundary
variously spelt as Shivmukha, Sishuka, till river Narmada. It assumed the titles of
Sindhuka, Chhismaka, Shipraka, Srimukha, etc "Satyashraya Sri Prithviballabh Maharaj",
"Dakshinapatheswara", "Parambhattaraka"
. He built Jain and Buddhist temples
and "Maharajadhiraja". Important information
 Statement 2 is correct : The Satavahanas
about it is obtained from the Aihole rock
retained some of the administrative units of
Inscription composed by Rajkavi
Ashokan times. The kingdom was divided into Ravikirti.Pulakeshin II was the first ruler to
subdivisions called aharas or rashtras, establish an empire in the south which covered
meaning districts. There were also officers the whole of south India. The title Shri-prithvi-
called amatyas/mahamatras who were vallabha means "the husband of the goddess of
perhaps ministers or advisors of the king. But, fortune and the Earth”, Dakshinapatheshwara
unlike Mauryan times, certain military and
35

means The Lord of the South and Q.13) Answer: A


Maharajadhiraja means 'great king of kings'.

Q.11) Answer: D
 Pair 1 is incorrect: The Iranian and
Macedonian invasions left an impact on the
culture and social history to the regions of the
North-west. The Achaemanian rulers of Iran
under Darius and Xerxes, conquered areas to
the west of Sindh including Punjab and
Sindh. The Kharosthi script was introduced by
the Iranian invasion. It is written from right to  Statement 1 is correct: The highest
left and is present in a number functionaries such as the minister (Mantrin),
of Ashokan inscriptions in the north-west. high priest (purohita) etc. were paid as much
 Pair 2 is correct: The monuments of Ashoka at as 48000 panas (silver coins currency). In
that time were influenced by the Iranian sharp contrast to them, the lowest officers
contacts. The presence of preamble on the were given 60 Panas in consolidated pay with
Ashokan edicts along with the bell shaped some officials getting as low as 20 panas.
Ashokan capital is influenced by the Iranian  Statement 2 is incorrect: Chandragupta
contact. Maurya maintained a body of spies who kept
 Pair 3 is correct: There was an extensive use of watch on events going on throughout the
elephants in the army as witnessed in the army country and regularly reported to the
of the Nandas. The Nandas were known to King. Strabo says that these spies were
have possessed as many as 6000 elephants appointed from the most faithful persons.
which made it easy for the crushing of cities There is also mention
and forts. of Pativedakas and Pulisani who functioned
 Pair 4 is correct: The horse chariots were as special reporters of the king and kept him
introduced by the vedic people. However, its informed of public opinion.
significance began to decline in the later times
with the introduction of cavalry, infantry, and Q.14) Answer: B
elephants. However, their use was not  A very important aspect of the Mauryan
completely stopped. Economy was the Agricultural developments
and modernization. The importance of
Q.12) Answer: B irrigation was fully realised and peasants had
Exp: to pay more tax on irrigated land known
 Option B is correct: It was during Vedic period as udaka-bhaga (water rate, and generally
that Sulvasutras were produced and dealt levied at one-fifth to one-third of the produce)
with measurement systems, especially the Share-cropping existed on both state-owned
measurement of fields and and privately owned land. Kautilya refers to
houses. Sulvasutras deals with the field of different kinds of share-croppers working on
geometry and mathematics. They are classified the state-owned land such as
into four with the oldest one Baudhayan the Ardhasitikas (kept half of the produce) and
sulvasutra dealing with the Pythagoras the Svaviryopajivins (kept one-fourth or one-
theorem or the calculation of the length of the fifth of the produce). It should be noted that
sides of a right angled triangle. They are the there is no reference in any text of taking away
reference of the Mathematical knowledge
the land of the farmers in case of non-payment
from the vedic people and formed part of the
of taxes. Kautilya also mentions
appendices of the Vedas.
certain emergency taxes (Pranaya) or
36

additional levies that the state could impose a Greek Ambassador who adopted
if the treasury got depleted. Vaishnavism. New terms which had the
influence of Greek terminology began to
Q.15) Answer: B be used in the subcontinent. For instance,
 Option A is correct: the term ‘horshastra’ used for astrology in
o Menander /Milinda/Minedra was one of Sanskrit is derived from Greek word
the most celebrated Indo-Greek rulers ‘horoscope’.
who not only stabilised Indo-Greek power,
Q.16) Answer: A
but also extended his empire’s frontiers in
 Statement 1 is Correct - Tolkappiyam is a work
India.
on Tamil grammar and it also provides
o He was converted to Buddhism by
information about the society, political and
Nagasena and has been identified with the
economic condition of the sangam age. It is a
King Milinda mentioned in the famous
treatise on grammar and poetics.
Buddhist text Milindapanha (Question of
 Statement 2 is Incorrect - Tolkappiyam is the
Milinda), which contains philosophical
only book surviving from the second sangam.
questions that Milinda asked Nagasena
Works from the first sangam didnt survive.it is
(the Buddhist author of the text). The text
believed that it was written by a single author
claims that impressed by the answers,
named Tholkappiyar, a disciple of Vedic sage
the king accepted Buddhism as his religion.
Agastya.
 Option B is incorrect:
o The Indo-Greeks were the first rulers in
India to issue coins (gold, silver, copper,
and nickel) which can be definitely
attributed to any dynasty and were also
the first to issue gold coins in India (which
increased in number under the Kushanas).
o Interestingly, the coins of the Shakas,  Statement 3 is Incorrect - Silappadikaram
Parthians and the Kshatrapas followed written by Ilango Adigal is about love affair of
the basic features of Indo-Greek coinage, Kovalan, Kanaggi and Madhavi. It also
including the bilingual and bi-script provides valuable information on the Sangam
legends. polity and society. The Silappathikaram gives a
o Indo- Greek coins were miniatures of detailed picture about the Tamil culture, its
aesthetic excellence, of precise economic varied religions, its town plans and city types,
value and also provided useful information the commingling of Greek, Arab, and Tamil
on emerging religious sects and cults peoples, and the arts of dance and music.
(specially Shaiva and Bhagvata sects) that o Manimekhalai was Written by Sittalai
were prominent in that area. Sathanar continues the story of
 Option C and D are correct: Silappadikaram in next generation
o They introduced new features of in which Manimekhalai is the daughter
Hellenistic art and literature such as of Madhavi and Kovalan.
Gandhara art in the north-west frontier of
India, which was not purely Greek, but Q.17) Answer: A
rather it was the outcome of  Statement 1 is Correct - Hereditary monarchy
the intermingling and influence of both was the form of government in the Sangam
Indian,central Asian and Hellenistic age. The king was assisted by a wide body of
contacts. officials who were categorised into five
o A striking instance of this mingling is the councils.They were ministers (amaichar),
Besnagar Pillar inscription of Heliodorus -
37

priests (anthanar), envoys (thuthar), military Sangam Age. The literature includes
commanders (senapathi), and spies (orrar). Tolkappiyam, Ettutogai, Pattuppattu,
 Statement 2 is Incorrect - Land revenue, Pathinenkilkanakku, and two epics named –
customs duty on foreign trade and booty Silappathikaram and Manimegalai. Among
captured in wars were the main sources of them, The Ettuthogai means the Eight
theincome.One sixth of the total produce was Anthologies of the sangam era. They are
collected as land tax. Tolls were also collected. Aingurunooru, Narrinai, Aganaooru,
The local chieftains paid tributes and gifts to Purananooru, Kuruntogai, Kalittogai, Paripadal
the king and this income constituted a and Padirruppattu.
significant part of the government revenue.
 Statement 3 is Incorrect - Sati was still
prevalent in the society and practised Q.19) Answer: B
especially by higher caste women.Widows  Pair 1 is correct - Silappadikaram – written by
position in the society was vulnerable. Women Ilango Adigal. It is about the love affair of
were not permitted to remarry and inherit Kovalan, Kanaggi and Madhavi. Later, a
property.Women’s education was also insisted Kannagi Cult developed in South India.
during the Sangam Age. There were few  Pair 2 is incorrect - Tirrukural deals with
women poets like Avvaiyar, Kakkai Padiniyar philosophy and wise maxims & was written by
and Nachchellaiyar, whose verses are found in Tiruvallur. Tirukkural comprises 133 sections of
the Sangam literature. 10 couplets each is divided into three books:
Aram (virtue), Porul (government and society),
and Kamam (love).
Q.18) Answer: D
 Pair 3 is incorrect - Madurai Kanji describes the
 Pair 1 is Incorrect - Kurinji was the hilly,
socio-economic condition of the Pandya
forested terrain/ tracts of land/ mountainous
country and is written by Mangudi
regions in the sangam age and its chief deity
Maruthanar.
was Murugan whose chief occupation was
hunting and honey collection. The other types
of land are as follows- Q.20) Answer: A
o Mullai was referred to the pastoral lands  Statement 1 is correct - According to the
and the chief Deity was Mayon, Sangam literature, the Tamil land had certain
o Marudam was the agricultural lands and commodities which were in great demand in
the chief deity was Indira, foreign markets. Black pepper, cardamom,
o Neydal was referred to as the coastal lands ginger, cardamom, cinnamon, turmeric, ivory
and the chief deity was Varunan products, pearls, precious stones. pearls and
o Palai was the desert lands and the chief
gemstones,etc. The main imports included
deity was Korravai.
horses, gold and glass.
 Pair 2 is Incorrect - Nadu Kal/ Hero Stone was
erected in memory of the bravery shown by  Statement 2 is incorrect - Tolkappiyam refers
to four castes namely arasar, anthanar,
the warrior in battle. They were erected to
honour valiant men killed in battle, or those vanigar and vellalar. The ruling class was
called arasar. Anthanars played a significant
who lost their lives while defending their
villages or womenfolk. The nadu kals' or hero role in the Sangam polity and religion. Vanigars
stones with inscriptions were planted as part carried on trade and commerce. The vellalas
were agriculturists.
of the burial and this has been described by
many Sangam poets too.  Statement 3 is incorrect - Caste system was
widely prevalent during the Chola period.The
 Pair 3 is correct - The Sangam literature
provides valuable information regarding the inscriptions of the later period of the Chola rule
mention about two major divisions among the
38

castes – Valangai and Idangai castes. However, o There is plenty of information in the
there was cooperation among various castes Sangam literature to trace the position of
and subcastes in social and religious life. women during the Sangam age. Women
poets like Avvaiyar, Nachchellaiyar, and
Q.21) Answer: A
Kakkaipadiniyar flourished in this period
 Pair 1 and 2 are correct:
and contributed to Tamil literature.
 Sangam Epic Tolkappiyam refers to the five-
fold division of lands - Kurinji (hilly tracks),
Mullai (pastoral), Marudam (agricultural),
Neydal (coastal) and Palai (desert). The people
living in these five divisions had their
respective chief occupations as well as gods for
worship.
 Kurinji – chief deity was Murugan – chief
occupation, hunting and honey collection.
 Mullai – chief deity Mayon (Vishnu) – chief o The courage of women was also
occupation,cattle-rearing and dealing with appreciated in many poems. Karpu or
dairy products. Chaste life was considered the highest
 Marudam – chief deity Indira – chief virtue of women. Love marriage was a
occupation, agriculture. common practice.
 Neydal – chief deity Varunan – chief o Women were allowed to choose their life
occupation fishing and salt manufacturing. partners. However, the life of widows was
 Palai – chief deity Korravai – chief miserable. The practice of Sati was also
occupation robbery. prevalent in the higher strata of society.
The class of dancers was patronized by the
kings and nobles.
Q.22) Answer: D
 Tolkappiyam refers to four castes
Q.24) Answer: B
namely arasar, anthanar, vanigar and vellalar.
The ruling class was called arasar. Anthanars  Options 1,2,3 and 6 are correct:
played a significant role in the o External trade was carried between South
Sangam polity and religion. Vanigars carried India and the Greek kingdoms. After
on trade and commerce. the ascendancy of the Roman Empire, the
The vellalas were agriculturists. Roman trade assumed importance. The
port city of Puhar became an emporium of
 Other tribal groups like Parathavar, Panar,
foreign trade, as big ships entered this port
Eyinar, Kadambar, Maravar and Pulaiyar were
with precious goods. Other ports of
also found in the Sangam society.
commercial activity include Tondi, Musiri,
Ancient primitive tribes like Thodas, Irulas,
Korkai, Arikkamedu and Marakkanam.
Nagas and Vedars lived in this period.
o The author of Periplus provides the most
 The primary deity of the Sangam period was
valuable information on foreign
Seyon or Murugan, who is hailed as Tamil God.
trade. Plenty of gold and silver coins
The worship of Murugan had an ancient origin
issued by the Roman Emperors like
and the festivals relating to God Murugan was
Augustus, Tiberius and Nero were found
mentioned in the Sangam literature. He was
in all parts of Tamil Nadu.
honoured with six abodes known as Arupadai
o They reveal the extent of the trade and the
Veedu.
presence of Roman traders in the Tamil
country. The main exports of the Sangam
Q.23) Answer: B
age were cotton fabrics, spices like
 Only Statement 2 is incorrect:
39

pepper, ginger, cardamom, cinnamon and  Statement 2 is correct: Prayag


turmeric, ivory products, pearls and Prashasti written by Harisena in Sanskrit gives
precious stones. an account of Samudragupta’s conquests.
 Statement 3 is incorrect: The Allahabad Pillar
inscriptions indicate that Samudragupta was
Q.25) Answer: C
entirely opposite to that of Ashoka.
Exp:
Samudragupta believed in military expeditions
Megasthenes mentions that the palace
and conquests while Ashoka was a peace
of Chandragupta Maurya, though very large and
loving ruler who renounced warfare post the
luxurious, was built of carved and gilded wood
Kalinga war.
models on wooden originals. However, this was
not because of unskilled craftsmen or because
Mauryan architecture was primitive. The mastery Q.27) Answer: C
in stone work is also evident in Mauryan monolithic  Pair 1 is correct: Brihadsamhita was written
columns. Thus, the use of wood over stone is by Varahamihira in Sanskrit and deals with a
mainly attributed to the fact that stone was multitude of domains ranging from astronomy,
comparatively scarce in the Gangetic plain and astrology, weather, marriage, animals. Other
timber was vastly abundant. The adoption of works by Varahamihira are: Pancha
stone in later periods as a building medium was Siddhantika, Brihadjataka. Pancha
partly due to foreign contacts, but also to the Siddhantika is an astronomical text which
gradual disappearance of timber forests from the comprises five earlier astronomical treatises -
more populous and civilized regions of Surya Siddhanta, Romaka Siddhanta, Paulisa
India. Hence, the assertion is correct but the Siddhanta, Vasishtha Siddhanta, and
reason is incorrect. Paitamaha Siddhanta.
 Pair 2 is incorrect: Vagbhata (600) was a
Q.26) Answer: C disciple of Charaka and a distinguished
 Statement 1 is incorrect: The Allahabad Pillar physician of Ayurveda. He
inscription gives an account of military wrote Ashtangasamgraha which is a work on
conquests of Samudragupta. It describes the medicine. He also wrote
circumstances of Samudragupta’s accession, Ashtangahridaya. Banbhatta lived in the
his military campaigns in north India and the Harsha’s (606-647 A.D.) court. He
Deccan, his relationship with other wrote Harshacharita and Kadambari
contemporary rulers, and his accomplishments  Pair 3 is correct: Kavyadarsha was written
as a poet and scholar. This inscription by Dandin. He was also the author
is engraved on an Asokan pillar. Hence, it is of Dasakumaracharita.
related to both Ashoka and Samudragupta.

Q.28) Answer: B
 Pair 1 is incorrect: The founder of
the Vardhana dynasty was Pushyabhuti who
ruled from Thaneswar. He served as a military
general under the imperial Guptas and rose to
power after the fall of the Guptas. With the
accession of Prabakara Vardhana (580–605
CE), the Pushyabhuti family became strong and
powerful. Prabakara Vardhana fought against
the Gurjaras and the Huns and established his
authority as far as Malwa and Gujarat.
40

 Pair 2 is correct: The Palas ruled eastern India  Bhaga, Hiranya and Bali were the three kinds
from Pataliputra between the ninth and the of tax collected during Harsha’s reign. Bhaga
early twelfth centuries. Sanskrit, Prakrit and was the land tax paid in kind. One-sixth of the
Pali were the languages in use in their produce was collected as land revenue.
kingdom. Palas followed Mahayana Hiranya was the tax paid by farmers and
Buddhism. merchants in cash. There is no reference to
 Pair 3 is incorrect: The Rashtrakutas were the tax Bali. The crown land was divided into
originally known to be the feudatories of the four parts.
Western Chalukyas of Vatapi. Rashtrakutas o Part I - for carrying out the affairs of the
were early rulers, the greatest ruler of state
the dynasty was Dandidurga. Ruling from a o Part II - for paying the ministers and
smaller part of Berar, Dandidurga built his officers of the crown
career of territorial conquest after the death o Part III - for rewarding men of letters.
of Vikramaditya II (733–746 CE), the Chalukya o Part IV - for charity to religious institutions.
ruler. He captured the Gurjara Kingdom
of Nandipur, Malwa and eastern Madhya
Pradesh and the whole of Berar. By 750 CE, he Students Note:
had become the master of central and
southern Gujarat and the whole of Madhya
Pradesh and Berar.

Q.29) Answer: B
 Statement 1 is incorrect: Information about
the Palas is also provided to us by Tibetan
chronicles, although these were written in
the seventeenth century. According to these,
the Pala rulers were great patrons of Buddhist
learning and religion. The Nalanda university
which had been famous all over the
eastern world was revived by Dharmapala,
and 200 villages were set apart to meet its
expenses. He also founded the Vikramasila
university which became second only to
Nalanda in fame. It was located on the top of a
hill, on the banks of the Ganga in Magadha,
amidst pleasant surroundings. The Palas built
many viharas in which a large number of
Buddhist monks lived. Hence, Statement 2 is
correct.
 Statement 2 is correct: The
directly administered territories in the Pala
and Pratihara empires were divided into bhukti
(provinces), and mandala or visaya (districts).
The governor of a province was called uparika
and the head of a district, vidyapati. The
uparika was expected to collect land revenue
and maintain law and order with the help of
the army. The visayapati was expected to do
the same within his jurisdiction.

Q.30) Answer: C
41

Gupta Age, Post Guptas


 Statement 2 is Correct - Brahmanism gained a
Q.1) Answer: D lot of popularity during the Gupta period and
 Statement 1 is Incorrect - Prashastis are as a result Buddhism and Jainism received very
glorifying praises of kings and rulers, and were less royal patronage than the earlier times.
written by the poets of that time. Similarly Fahien also mentions the decline of Buddhism.
Prayag Prashasti is an inscription composed by However, a few Buddhist scholars like
Harisena, a poet and minister in the royal court Vasubandhu were supported by Gupta kings.
of Samudragupta to depict his political and In western and southern India Jainism
military achievements to praise the king. It flourished. The great Jain Council was held at
contains historical evidence of the political Valabhi during this period and the Jain Canon
imperial power of Guptas. of the Swetambras was written.
 Statement 2 is Incorrect - The inscription was  Statement 3 is Correct - Women were
written in both verse and prose in classical prohibited from studying the religious texts
Sanskrit using Brahmi script . like the Puranas. The practice of Swyamvara
Prayag-Prashasti is the name given to the was no longer practiced. The Manusmriti
Allahabad Pillar. It was originally engraved on suggested early marriage for the girls. So, it can
the Ashokan Pillar. It mentions about the be concluded that the position of women had
conquests of Samudragupta and boundaries of become miserable during the Gupta period.
the Gupta Empire.
Q.4) Answer: B
Q.2) Answer: D  Statement 1 is Correct - The inscriptions of
 Pair 1 is Incorrect - Charak is considered to be Naneghat inscriptions mentions the
the father of Ayurveda. Charak Samhita is his Satavahanas especially about Gautamiputra
treatise on the subject. Satakarni. The Naneghat inscription suggests
 Pair 2 is Incorrect - Varahamihira wrote three that Satakarni performed horse sacrifices
important books-Pancha Siddhantika, the five (Aswamedha), to proclaim his sovereignty.
astronomical systems, Brihadsamhita- It deals  Statement 2 is Incorrect - Satavahanas
with a variety of subjects like astronomy, patronized both Buddhism and Brahmanism.
astrology, geography, architecture, weather, They built chaityas and viharas. They also made
animals, marriage and omens in the Sanskrit grants of villages and lands to Buddhist monks.
language, Brihat Jataka -work on astrology. The Karle inscription mentions that Satakarni
 Pair 3 is Incorrect - Amarasimha was one of the donated lands to the Buddhist monks. Also,
nine gems in the court of chandragupta II along Vashishtaputra Pulamayi had got the old
with Kalidasa, Varahamihira, Dhanvantri, etc. Amaravathi stupa repaired.
he is famous for his Sanskrit thesaurus  Statement 3 is Incorrect - The trade and
“Amarakosha”. commerce prospered during the Satavahana
rule. Guilds were organized by the merchants
Q.3) Answer: D to increase their activities. The Satavahana
 Statement 1 is Correct - The Brahmins were period also witnessed overseas commercial
given high status in the society. They were activity.
bestowed with plenty of gifts by the rulers.
Meanwhile, the practice of untouchability Q.5) Answer: B
slowly started during this period. Fahien  The Aihole inscription of Pulakesin
elaborated that Chandalas were segregated II mentions the defeat of Harsha by Pulakesin
from the society and their condition was II, who after this achievement assumed the
miserable. The untouchables including title Paramesvara. It was written by Ravikirti,
chandals lived away from the other groups the court poet of king Pulakesin-II. The
They were assigned the lowest status in the inscription is found at Aihole Meguti Temple in
Indian society. Karnataka state, India.
42

Q.7) Answer: D
 Statement 1 is incorrect: Intercaste marriage
was practiced by the Gupta rulers
to strengthen and consolidate their
powers. Chandragupta I married a Lichchhavi
princess from Nepal. Chandragupta II married
his daughter Prabhavati to the Brahmin rulers
of the Vakataka Dynasty.
 Mehrauli Inscription is related to  Statement 2 is incorrect: The accession
Chandragupta II- This mentions that of Chandragupta I in 319 A.D. marked the
Chandragupta conquered the Vanga beginning of the Gupta era. The Gupta Dynasty
Countries. It also mentions his conquest of was founded by Srigupta. Chandragupta I was
Vakatkas. This pillar was established by the third ruler of the Gupta dynasty and was
Chandragupta-II of Gupta dynasty as regarded as the founder of the Gupta era.
Vishnupada in the honor of Lord Vishnu.
 The Allahabad Pillar inscription of Q.8) Ans: B
Samudragupta, a prashasti composed by Exp:
Harishena, credits him with extensive military Chandragupta Maurya:
conquests. It suggests that he defeated several  Chandragupta Maurya established the
kings of northern India, and annexed their Mauryan Empire and became its first emperor
territories to his empire. in 321 BCE.He was the first ruler who tried to
 Udayagiri Cave Inscription - Mentions consolidate small fragmented kingdoms and
about Chandragupta II as well as combined them to form the first empire of the
Kumaragupta. Indian subcontinent. He was also called
'Sandrocottus' by the Greek scholar William
Q.6) Answer: D Jones. Hence, Statement 2 is correct.
 Option A is incorrect: After the end of Kushan
rule, a large part of Central India was ruled by
their kinsmen, Murundas till 250 A.D. Twenty-
five years later, the Guptas came to power in
275 A.D.
 Option B is incorrect: The use of elephants and
horse-chariots began to decline from
the Kushan rule itself. Hence, during the times
of Gupta rule, cavalry and horsemen played an
important part.  Ashoka was the first king in Indian history who
 Option C is incorrect: The Gupta Rule was has left his records engraved on stones. Many
primarily centered around the present-day edicts were built by Ashoka throughout India,
states of Uttar Pradesh and Bihar. Madhya including in modern-day Nepal and Hence,
Pradesh was also part of their Empire, but it Statement 1 is incorrect .
was Uttar Pradesh that was the dominant  Megasthenes visited India during the reign of
center. The important centers Chandragupta Maurya. He was a Greek envoy
included Anuganga (middle Gangetic of Seleucus I Nicator visited
basin), Prayag (Prayagraj), Saketa (Ayodhya), Pataliputra.He was a historian and diplomat
and Magadha. who served as an ambassador to
 Option D is correct: Saketa, present day India.Megasthenes assembled information
Ayodhya city was one of the important centers about India in his book called Indika. Hence,
of the Gupta. Statement 3 is correct.
43

Q.9) Ans: A  The account of Megasthenes does not survive


Exp: in full but quotation occur in the works of the
 Samaharta was the chief revenue collector of several subsequent Greek writers.these
the Mauryan period. Samaharta was a very fragments have been collected and published
responsible post in the administration of the in the form of a book called Indika which
Mauryan government of Emperor throws valuable light on the administration,
Chandragupta Maurya.The key task of the society and economy of Maurya Times.
collection of revenue from various parts of the  The Mauryan Empire was finally destroyed by
kingdom was the prime responsibility of a pushyamitra shunga in 185 BC. Although a
samaharta. bramhana, he was a general of the last Maurya
 Sannidhata was the chief custodian of the king called Brihadrata. He is said to have killed
state treasury and store house during the Brihadrata in public and forcibly usurped the
Mauryan period. Navadhyaksha was the throne of Patliputra. Statement 3 is correct.
Superintendent of ships. Pairs 1 ,2 and 3 are
correct . Q.11) Ans: D
 The 'Sitadhyaksha' was the officer in charge of Exp:
agriculture. Internal policy of Ashoka:
 Akaradhyaksha was the Superintendent of  Ashoka was converted to Buddhism as a result
mines. Samsthadhyaksha was the of Kalinga war. According to tradition he
Superintendent of the market. Pair 4 is became a monk, made huge gifts to Buddhist
incorrect. and undertook pilgrimages to the Buddhist
shrines. The fact of his visiting the Buddhist
Q.10) Ans: B shrines is also suggested by the Dhamma
Exp: Yatras mentioned in his inscriptions. Ashoka
 The Mudrarakshasa is a Sanskrit-language play set a very high ideal for himself and this was
by Vishakhadatta that narrates the ascent of the ideal of paternal kingship. He repeatedly
the king Chandragupta Maurya to power in asked his official to tell the subjects that the
India.Vishakhadatta was the court poet of king looked upon them as his children.
Chandragupta II. Hence statement 1 is  He disapproved of rituals especially those
incorrect . observed by women. He forbade killing certain
 Chandragupta II (c. 380 – c. 415 CE), also birds and animals and completely prohibited
known by his title Vikramaditya as well as the slaughter of animals in the capital
Chandragupta Vikramaditya, was the third Pataliputra. Statement 1 is correct.
ruler of the Gupta Empire in India, and one of  He interadicted gay social functions in which
the most powerful emperors of the dynasty. people indulged in revalaries. Statement 2 is
 Chandragupta liberated northwestern India correct.
from the thraldom of Seleucus, who ruled over  Ashoka brought about the political unification
the area west of the Indus. In the war with the of the country. He bound it further by one
Greek viceroy, Chandragupta seems to have Dharma, one language and practically one
come out victorious. Eventually, there was script called Brahmi which was used in most of
peace between the two and in return for 500 his inscriptions. Statement 3 is correct. In
elephants, Seleucus gave him eastern unifying country he respected such scripts as
Afghanistan, Baluchistan and the area west of Brahmi, kharoshthi, Aramaic and Greek.
Indus. Evidently he also accommodated such
 Megasthenes was a Greek ambassador sent languages as Greek, Prakrit and Sanskrit and
by seleucus to the court of Chandragupta various religious sects. Ashoka followed a
Maurya. Hence statement 2 is correct. tolerant religious policy.
 He lived in the Maurya capital of Patliputra and
wrote an account not only of the Q.12) Answer: A
administration of the city of Patliputra but also  Statement 1 is correct: Fa-hsien (Fa-
of the Maurya Empire as a whole. Xian) visited Indian during the reign of Gupta
44

king Chandragupta-II. His purpose was to visit It records the historical events of his reign in a
holy Buddhis places such as Gaya, Lumbini, chronological order.
Sarnath, Sanchi and to collect holy Buddhist  Pair 3 is correct: The achievements
scriptures. On the other hand, Hsuan Tsang of Gautamiputra Satakarni were recorded in
(Xuan Zang) came to India between 631- Nasik inscriptions by his mother. In this
645AD. At that time Harsha was ruling over inscription Gautamiputra Satakarni is called
North India and Pallava King Narsimha “Ekabrahmana” which means either
Verman was ruling over southern India. Xuan ‘unrivalled Brahmin’ or ‘the only protector of
Zang had visited Pallava capital the Brahmins’
Kanchipuram during his visit to India According  Pair 4 is correct: Harsha with an ambition to
to his account the people of Kanchi esteemed extend his kingdom south of the Narmada river
great learning and the Ghatika at Kanchi served marched against the Chalukya ruler. But
as a great centre of learning. the Aihole inscription of Pulakesin II mentions
Statement 2 is incorrect: Fa-Xian came to India the defeat of Harsha by Pulakesin, who after
through Central Asia and returned to China this achievement assumed the title
from Bengal in a merchant ship. From there he Paramesvara. The inscription was composed
reached Java, where he halted for five months, by Ravikirti.
before boarding another merchant ship that
took him to China. On the other hand, Xuan- Q.14) Answer: C
Zang took the land route to China through  Pair 1 is correct: During the Sangam
north-west and Central Asia. He carried with period, war heroes occupied a special position
him statues of the Buddha made of gold, silver in society, and memorial stones called nadukal
and sandalwood, and over 600 or virukkal were raised in honour of those who
manuscripts loaded on the backs of 20 horses. died in fighting and they were worshipped as
Over 50 manuscripts were lost when the boat godlings.
on which he was crossing the Indus capsized.  Pair 2 is incorrect: Muziris was the foremost
He spent the rest of his life translating the port in the Cheras kingdom.
remaining manuscripts from Sanskrit into The Periplus speaks of cargo-laden ships
Chinese. landing here from Arabia and Egypt, and gives
long lists of imports and exports. Tondi was
Q.13) Answer: D another important port of Cheras.
 Pair 1 is correct: Rudradaman-I (130-150AD)  Pair 3 is correct: Kumaramatyas were high
was the most famous Shaka ruler in India. He is level central officials under the Gupta kings.
famous in history because of the repair work One of the Vaishali seals refers to a
he undertook to improve the Sudarshana kumaramatya who seems to have been in
lake in the semi-arid zone of Kathiawar. He charge of the maintenance of the sacred
issued the first-ever long inscription in chaste coronation tank of the Lichchhavis. Individuals
Sanskrit, called Junagarh inscription. All the of the rank of kumaramatya sometimes
earlier longer inscriptions that we have in this had additional designations as well, and such
country were composed in Prakrit. The ranks could be hereditary.
Junagadh rock also contains inscriptions of
Ashoka and Skandagupta. PW OnlyIAS Extra Edge:
 Pair 2 is incorrect: Udayagiri caves are famous  Kaveripattinam (also known as Pumpuhar
for the Hathigumpha inscription which is or Puhar) was the premier Chola port in
presented in Kavya style, and in Brahmi. The early historical times. Classical accounts
inscription starts out with “Jain Namokar refer to it as Khaberis or Camara. An entire
Mantra” and highlights various military Sangam collection—the Pattinappalai—is
campaigns undertaken by the King Kharavela. devoted to a description of this place.
45

 Korkai was an important Pandya port, Mahamantri, Senapati, and Yuvraj. Ashoka had
celebrated for its pearls in Sangam poems appointed Dhamma Mahamantras who
and Greek accounts. The Arthashastra also helped in the enforcement of Dhamma. The
refers to the pearl fisheries of Pandya Gupta kings assumed titles like
country. Paramabhattaraka, Maharajadhiraja,
Parameswara, Samrat and Chakravartin. The
king was assisted in his administration by a
council consisting of a chief minister, a
Q.15) Answer: B Senapati or commander in-chief of the army
 Statement 1 is incorrect: The succession and other important officials. The king
during the Gupta rule was maintained a close contact with the provincial
mainly hereditary. However, the practice administration through a class of officials
of primogeniture (transfer of throne to the called Kumaramatyas and Ayuktas.
eldest son) was not fully developed and this led
to uncertainties and chaos which was misused Q.16) Answer: B
by the chiefs and other officials. There were  Statement 1 is incorrect: The system of
numerous gifts made to the Brahmanas during granting land tax free to the priests was started
the Gupta times who compared the Kings to in the Deccan by the Satavahanas. This system
God. was fully developed during the Gupta times
 Statement 2 is correct: The Guptas had a very especially in Madhya Pradesh. Priests were
well developed judicial system where several given tax free land and were even empowered
books on law were compiled. Civil and criminal to collect taxes from the peasants. The priests
law was clearly distinguished for the first time were also given powers to punish the criminals.
during the Gupta rule. The King was regarded  Statement 2 is correct: The participation of
as the ultimate upholder of the law. Crimes artisans, merchants, elders, and even priests in
like adultery, theft, etc were put under criminal the administration greatly reduced the need to
laws while those relating to property appoint administrators during the Gupta
inheritance were put under the civil laws. times. Hence, the major chunk of the Gupta
However, the guilds of artisans, merchants, Empire was administered by the feudal chiefs
traders, etc. were governed by their own set of and the beneficiaries of land grants. The areas
laws. in the Northern parts of Bengal, Bihar, Uttar
 Statement 3 is correct: The Gupta rulers were Pradesh, and some areas of Madhya
themselves Vaishyas and hence made Pradesh were administered by the officials
recruitment from the lower varnas as appointed by the Gupta rulers whereas
well. The Kumaramatyas were the important other areas near the fringe of the empire were
officials during the Gupta rule and were administered by the feudal vassals. These
directly appointed by the king and paid in cash. feudal vassals received the King’s charters for
A single person held several posts under the ruling the areas under their control in return
Gupta rule and posts were for tribute to the King. Samudragupta greatly
generally hereditary which loosened the exercised control over these feudal vassals.
control of the King over his officials.
 Statement 4 is incorrect: The bureaucracy of Q.17) Answer: C
Guptas was not as organized as the Mauryas.  Statement 1 is incorrect: Gold coins during the
The civil servants were known Gupta times were called Dinars and were
as Amatyas during the Mauryan issued in large numbers. They portrayed the
times. Mauryans had a council of ministers conquests of the Gupta rulers. However,
called the Mantriparishad who helped the they were not as pure as that of the Kushan
king in the administration. It included Purohita, gold coins. Silver coins during the Gupta times
46

were mainly used for the purpose of local Gupta empire from the Huns. Ultimately, the
exchange. The copper coins were very few in Gupta power totally disappeared due to the
number and hence indicate that the common Hun invasions and later by the rise of
citizens did not make much use of money Yasodharman in Malwa. Banabhatta in his
unlike the Kushan period. Harshcharitra refers to the Huns invasion.
 Statement 2 is correct: Land grants given by  Statement 4 is incorrect: Bhitari monolithic
the Gupta rulers to the Brahmin priests pillar inscription deals with Skandgupta. It is
brought huge expanses of land under located in Gazipur, Uttar Pradesh. It provides
cultivation with better knowledge of chronology of the various Gupta rulers. It also
cultivation imparted by the Brahmanas. They mentions the conflict between Skandagupta
emerged as landlords while the local peasantry and the Pushyamitras as well as the Hunas.
was subjugated.
 Statement 3 is correct: The economic status of
Q.19) Answer: C
Shudras improved during the Gupta times due
 Statement 1 is incorrect: Sanskrit was
to their employment in the agricultural
the court language of Guptas as a result of
activities while in the earlier periods, Shudras which Sanskrit literature flourished under
were mostly servants and slaves of the higher them. Kalidasa, the great poet, and playwright
varnas. They were permitted to listen to the was in the court of Chandragupta
Mahabharata, Ramayana, Puranas, and other Vikramaditya.
religious scriptures. They could even conduct  Statement 2 is correct: Kalidas composed
domestic rites that brought income to the great epics such as
priests. Abhijnanashaakuntalam, Kumarasambhavam,
 Statement 4 is incorrect: The women of lower Malavikagnimitram, Ritusamhara,
varnas were given the freedom to work in the Meghadootam, Vikramorvashiyam, and
agricultural fields and not depend fully on their Raghuvamsham. The celebrated Sanskrit
drama Mṛichchhakatika was composed during
husbands. On the other hand, the women of
this time. It is attributed to Shudraka.
the higher varnas were still deprived of the
o Poet Harisena also adorned the court of
freedom of livelihood. Men of higher varnas Samudragupta. He wrote the Allahabad
acquired huge land and property. They were Prashasti (inscription).
mostly polygamous and looked at women as o Vishnu Sharma of Panchatantra fame
property. lived during this era.
o Amarasimha (grammarian and poet)
Q.18) Answer: C
composed a lexicon of
 Statement 1 is correct: Mehrauli Iron Pillar Sanskrit, Amarakosha.
inscriptions talk of the conquests o Vishakhadatta composed
of Chandragupta II. It is famous for the rust- Mudrarakshasa. Other grammarians who
resistant composition of the metals used in its contributed to the Sanskrit language
construction. include Vararuchi and Bhartrihari.
 Statement 2 is incorrect: Chandragupta II ● Statement 3 is correct: Bhasa wrote 13 plays
patronized the Navratnas or the nine gems, which echo the lifestyle of the Gupta Era along
the most important among them with its prevalent beliefs and culture. The
was Kalidasa. Others were: Amarsimha, unique features of the plays of this time is
Dhanvantri, Harisena, Kahapanaka, Sanku, that. They are all comedies. Characters of
higher and lower classes do not speak the
Varahamihira, Vararuchi, Vetala Bhatta.
same language, Women and Sudras
 Statement 3 is correct: The Hun invasion was featured in these plays speak Prakrit.
faced mainly by Skangupta who protected the
empire from the Huns. However, weak
successors of Skandagupta could not save the
47

Q.20) Answer: A was the first international residential


About Gupta Empire university in the world. It was patronized by
 On the ruins of the Kushan empire arose a new his successors and later by Harsha. The
empire, which established its sway over a good professors of the University were called
part of the former dominions of both panditas. Some of its renowned professors
the Kushans and Satavahanas. This was the were Dingnaga, Dharmapala, Sthiramati and
empire of the Guptas. Although the Gupta Silabadhra. Dharmapala was a native of
empire was not as large as the Maurya Kanchipuram and he became the head of
empire, it kept north India politically Nalanda University. Though it was a Mahayana
united for more than a century from AD. 335 University, different religious subjects like
to 455. The Guptas were the Vedas, Hinayana doctrine, Sankhya and
possibly the feudatories of the Kushans in Yoga philosophies were also taught.
Uttar Pradesh, and seem to have succeeded
them without any wide time-lag. Factors that
led to the rise and growth of the Gupta
Empire:
 Statement 1 is correct: In the Kushan scheme
of things, horse-chariots and elephants had
ceased to be important. Horsemen played the
main part in the case with the Guptas.
Although some Gupta kings are described
as excellent and unrivaled chariot warriors,  Statement 2 is incorrect: The top ten eminent
their basic strength lay In the use of horses. All rulers of the Gupta empire in India are:
these gave them mobility and made them o Chandra Gupta I (320-335 or 340 A.D.)
excellent horsemen. o Samudra Gupta (Nearly 340-380 A.D.)
 Statement 3 is correct: The Guptas enjoyed o Rama Gupta
certain material advantages. The center of o Chandra Gupta II (Nearly 380-413 or 415
their operations lay in the fertile land A.D)
of Madhyadesha covering Bihar and Uttar o Kumara Gupta (Near about 415-455 A.D.)
Pradesh. They could exploit the iron ores o Skanda Gupta (455-467 A.D)
of central India and south Bihar. o Puru Gupta (467-469 A.D.)
 Statement 2 is correct: Further, they took o Budha Gupta (477 to nearly 500 A.D.).
advantage of their proximity to the areas in
north India which carried on silk trade with
Students Note:
the Eastern Roman empire also known as
the Byzantine empire.

Q.21) Answer: D
 Statement 1 is incorrect: Nalanda University is
located in Rajgir, near Nalanda, Bihar, India. It
stands for a university that attracts students
and scholars from across Asia and even farther
away. It was developed under the patronage
of the Kumaragupta of the Gupta dynasty and
48

Pallavas & Chalukyas in South, Chola Empire (9th -12th Century)


 Statement 2 is Incorrect- Shalabhoga was a
Q.1) Answer: B type of land for the maintenance of a school.
 Statement 1 is Incorrect - The core area of According to Chola inscriptions, there were five
rule of the cholas was the Kaveri delta. The types of 'land gifts' that Chola kings gave to
Cholas controlled the central and northern their people:
parts of Tamil Nadu.The Chola Empire o Vellanvagai was land for non-Brahmana,
consisted of the current day territories of peasant proprietors.
Tiruchirapalli, Tiruvarur, Perambalur, o Brahmadeya was land gifted to
Ariyalur, Nagapattinam, Pudukkottai, Brahmanas.
Vridhachalam, Pichvaram and Thanjavur
districts of Tamil Nadu. o Shalabhoga was land given for the
 Statement 2 is Correct - King Karikala, was maintenance of a school.
a famous ruler of the chola kingdom. He o devadana / tirunamattukkani was land
founded the port city of Puhar (identical gifted to temples.
with Kaveripattinam) and constructed 160 o Pallichchhandam land donated to Jaina
km of embankment along the Kaveri River. institutions.
He was the greatest among the early Chola
kings of the Sangam age in South India. Q.3) Answer: C
The Sangam literature discusses the 3 main
Kingdoms- Chola. Pandya & Chera.
 Statement 1 is correct - Chera kings adopted
the “Bow and arrow” as a crest/identity of
their dynasty. Cheras controlled the central
and northern parts of Kerala and the Kongu
region of Tamil Nadu. Vanji was their capital.
The Pugalur inscription of the first century A.D
refers to three generations of Chera rulers.
Senguttuvan (Red Chera) was one of the
 Statement 3 is Correct - In the Battle of prominent Chera Kings. He established
Venni King Karikala defeated the the Kannagi or Pattini Cult.
confederacy of Cheras, Pandyas and  Statement 2 is Incorrect - The emblem of
eleven minor chieftains leading to the the Cholas was the Tiger. They controlled the
unifying of the three kingdoms into one central and northern parts of Tamil Nadu. Elara
under Karikala's rule. The Battle of Venni was the earliest known king. He conquered Sri
was the defining event in the career of Lanka & ruled over it for 50 yrs. Rajaraja chola,
Karikala, which established a virtual Rajendra Chola etc were some prominent
hegemony among the three crowned chola
monarchs. rulers. Kaverippattanam, Uraiyur and Arikam
edu (Puducherry) became famous centers of
Q.2) Answer: D trade and industry under Cholas.
 Statement 1 is Incorrect- Musiri and Tondi  Statement 3 is Incorrect - The emblem of
were the important seaports of the the Pandya kings was Fish. They ruled from
Chera dynasty Kaveripattinam was the port Madurai and Korkai was their main port.
associated with Chola dynasty. The oldest and Rajasimha I , Nandivarman II were some
most famous port in the Chola Empire prominent Pandyan rulers. Pandyas had trade
was Poompuhar. Other naval ports of relations with Romans. They were first
cholas were located at Arikamedu, mentioned by Megasthenes. They also find
Kancheepuram, Nagapattinam, Kulachal, mention in the Ramayana &
Korkai, Kadalur, and Thoothukudi. Mahabharata. Maduraikkanji authored
by Mangudi Maruthanar describes the socio-
49

economic condition of the Pandya country.


Extra Edge by PW Only IAS:
Q.4) Answer: C
 Ibn Batuta a Moroccan traveller visited India
 Statement 1 is correct : The following are
in 1334. He was contemporary of Muhammad
some of the sources where Pandyas are
Tughlaq and was his envoy to China.
mentioned-
 During the journey he visited Malabar Coast,
o Pandyas are mentioned in Kautilya’s
Bengal, Assam, Sri Lanka, Maldive, Sumatra
Arthashastra and Megasthenes’ Indica.
etc. He went to China through the sea route
o They are also frequently mentioned in
but returned by land route through central
the Sangam Literature.
Asia.
o They also find mentions in the Ramayana
 His book Rihla, written in Arabic, provides
and Mahabharata.
extremely rich and interesting details about
o The Hathigumpha inscriptions of the
the social and cultural life in the subcontinent
Kalinga King Kharavela, refers to the arrival
in the fourteenth century.
of tributes from the Pandyan king.
 Statement 2 is incorrect - In the later part of
the 13th century (in 1288 and 1293 CE)
Q.5) Answer: B
Venetian traveller Marco Polo visited the
 Statement 1 is Incorrect- The Pallava state was
Pandya kingdom and left a vivid description of
divided into Kottams. The Kottams were
the land and its people.In his travel account, he
administered by officers appointed by the king.
also records the incidents of sati and the
The king was at the centre of administration in
polygamy practiced by the kings. He also
which he was assisted by able ministers.
visited the Kakatiya kingdom.
 Statement 2 is Correct- The king provided
land-grants to the temples known as
Devadhana and also to the Brahmans known as
Brahmadeya. These lands were exempted by
the taxes of the government.
 Statement 3 is Correct- The Pallava
inscriptions depict on the village assemblies
called sabhas as well as their committees.
Sabhas maintained records of all village lands,
looked after local affairs and managed
temples. In Pallava polity, there are three
important territorial assemblies- Ur, Sabha
and Nagaram. The Ur was a non-Brahmanical
assembly .The Nagara was an assembly of
 Statement 3 is correct - The Pandyan dynasty mercantile groups. All these local assemblies or
power declined with the invasion of a tribe bodies used to meet regularly every year while
called the Kalabhras. For a long time, this the day-to-day tasks were taken care of by a
period of the Kalabhras was described as the small executive body. However, the king was at
‘Dark Ages’ in the history of Tamilakam. There the centre of administration in which he was
was political chaos during the Kalabhra supported by ministers.
rule. The secular outlook of the Sangam
period gave way to the religious outlook of
the Kalabhras. Q.6) Answer: C
 Statement 1 is correct : The Mamandur
inscription contains a note on the notation of
vocal music. Mahendravarman Pallava’s
achievements and titles were also inscribed
here. The Kudumianmalai inscription referred
50

to musical notes and instruments.It is an chieftains and is said to have ensured the
important musical Inscription. safety of the great port Musiri by putting down
 Statement 2 is correct : The Sittannavasal piracy.
paintings belonged to this period. They are a o But the great north Indian expedition of
2nd-century Jain complex of caves in Tamil Chenguttuvan mentioned in
Nadu. It has beautiful Fresco Paintings. They Silappathikaram is however not
have a close form of Ajanta and Bagh mentioned in the Sangam poems.
caves. Most paintings date to the Pandyan o He is said to have ruled for fifty-six years,
period i.e. 9th century AD. The notable patronising the orthodox and heterodox
drawings include a pond with lotuses. This religions. Some Cheras issued copper and
scene shows Samava-sarana –a unique, chosen lead coins, with Tamil-Brahmi legends,
and elegant audience hall where Tirthankaras imitating Roman coins. There are many
gave sermons after they reached realisation other Chera coins with their bow and
(kevala-jnana). arrow emblem but without any writing on
them.

Q.8) Ans: A
Exp:
● Statement 1 is correct: The Pandya dynasty,
also known as the Pandyas of Madurai, was a
dynasty of south India, one of the three famous
Tamil lineages, the other two being the Chola
and the Chera. The Pandyas ruled from
Madurai. Korkai was their main port, located
near the confluence of Thampraparani with
 Statement 3 is correct: Pallavas were
the Bay of Bengal. It was famous for pearl
the contemporaries of
Chalukyas. Mahendravarman II was also
fishery and chank diving. Korkai is referred to
a contemporary of Harshavardhan.
in the Periplus as Kolkoi. Fish was the emblem
of the Pandyas. Their coins have elephants on
Q.7) Ans: B
one side and a stylised image of fish on the
Exp:
other. They invaded Southern Kerala and
 Statement 1 is correct: The Cheras controlled controlled the port of Nelkynda, near
the central and northern parts of Kerala and Kottayam. According to tradition, they
the Kongu region of Tamil Nadu. Vanji was patronized the Tamil Sangams and facilitated
their capital and the ports of the west coast, the compilation of the Sangam poems. The
Musiri and Tondi, were under their control. Sangam poems mention the names of several
o Vanji is identified with Karur, while some kings, but their succession and regional years
scholars identify it with Tiruvanchaikkalam are not clear.
in Kerala. ● Statement 2 is incorrect: From the 6th century
o Now it is accepted by most scholars that
to the 9th century CE, the Chalukyas of Badami
there were two main branches of the or Rashtrakutas of the Deccan, the Pallavas of
Chera family and the Poraiya branch ruled Kanchi, and Pandyas of Madurai dominated
from Karur of present-day Tamil Nadu. the politics of south India. The Pandyas at one
● Statement 2 is incorrect: The Patitrupathu time or another ruled or invaded the fertile
speaks of eight Chera kings, their territory and estuary of Kaveri (the Chola country), the
fame. The inscriptions of Pugalur near Karur ancient Chera country (Kongu and central
mention Chera kings of three generations. Kerala) and Venadu (south Kerala), the Pallava
Chellirumporai issued coins in his name. country and Sri Lanka. The Pandyas went into
Imayavaramban Nedun-cheralathan and decline with the rise of the Cholas of Tanjore
Chenguttuvan are some of the prominent in the 9th century and were in constant
Chera kings. Chenguttuvan defeated many
51

conflict with them. The Pandyas allied


themselves with the Sinhalese (Sri Lanka) and
the Cheras in harassing the Chola Empire until
it found an opportunity for reviving its
fortunes during the late 13th century.

Q.9) Ans: D
Exp:
 Statement 1 is correct: The Sangam Age
constitutes an important chapter in the history
of South India. According to Tamil legends,
there existed three Sangams (Academy of
Tamil poets) in ancient Tamil Nadu popularly
called Muchchangam. These Sangams
flourished under the royal patronage of the
Pandyas. The first Sangam, held at Then
Madurai, was attended by gods and legendary
sages but no literary work of this Sangam was
available. The second Sangam was held at  Statement 3 is correct: There is evidence of
Kapadapuram but all the literary works had craft production such as bronze vessels, beads
perished except Tolkappiyam. The third and gold works, textiles, shell bangles and
Sangam at Madurai. In the Sangam Age the ornaments, glass, iron smithy, pottery making.
wars waged by the Vendar were involved in Craft production was common in the major
expanding their territorial base by annexing urban centres such as Arikamedu, Uraiyur,
the enemy’s territories. Endemic warfare Kanchipuram, Kaveripattinam, Madurai,
presumably created conditions for social Korkai, and Pattanam in Kerala.
disparities. War captives serving in some cult Maduraikanchi speaks about day markets as
centres are mentioned. Some references to well as night markets selling several craft
slaves are also found there. goods.
 Statement 2 is correct: The corpus of Sangam
literature includes Tolkappiyam, Ettutogai, Q.10) Ans: C
Pattuppattu, Pathinenkilkanakku, and the Exp:
two epics - Silappathigaram and Manimegalai. ● The Sangam corpus includes six of the eight
Tolkappiyam authored by Tolkappiyar is the anthologies of poems included in the Ettutokai
earliest of Tamil literature. Women were (The Eight Collections), and nine of the ten
actively engaged in economic production and pattus (songs) of the Pattuppattu(The Ten
there were a significant number of women Songs). The style and certain historical
poets in the Sangam Age. Women poets like references in the poems suggest that they
Avvaiyar, Nachchellaiyar, and Kakkaipadiniyar were composed between the 3rd century BCE
flourished in this period and contributed to and the 3rd century CE. They were compiled
Tamil literature. into anthologies in about the mid-8th century.
● A few centuries later, these anthologies were
collected into the super-anthologies (i.e.,
anthologies of anthologies) called the
Ettuthokai and the Pattuppattu. The earliest
parts of the first two books of the Tolkappiyam
can also be included in Sangam literature.
● The Tolkappiyam is essentially a work on
grammar, but it also includes a discussion of
phonology, semantics, syntax, and literary
conventions. A number of Tamil didactic works
52

were written in the post-5th century period. ● Thirukkural is concerned with an ethical
The most famous of these is Thiruvalluvar's lifestyle and is very practical in its approach
Thirukkural, a work on ethics, polity, and love and focus materials. It has as its foremost
(5th–6th centuries). moral imperatives the avoidance of killing and
● Of the several Tamil epics, two of the best the avoidance of falsehood. It also commends
known are the Silappadikaramand to the reader a feeling of compassion for all
Manimekalai. The former is a little earlier than individuals, regardless of caste or creed.
the latter, but both were composed in about ● The poems are concerned with two main
the 5th–6th centuries CE. topics: those of the first five collections are on
love (akam), and those of the next two are on
Q.11) Ans: A heroism (puram), including the praise of kings
Exp: and their deeds.
 The first three centuries of the Common Era ● Thirukkural is widely regarded as a secular
are widely accepted as the Sangam period, as work which despite some references to God,
the information for this period is mainly does not venture to propound the tenets of
derived from the Sangam literature. More any particular religion or godhead. Its 133
correctly this has to be called the early sections of 10 couplets each are divided into
historical period and starts one or two three books: aram (virtue), porul
centuries earlier, from the second century (government and society), and kamam (love).
BCE, as we have clear epigraphical and
archaeological evidence, in addition to literary Q.13) Answer: A
evidence.  Statement 1 is correct: Cholas had built more
 The names of persons mentioned in than two hundred temples which seems to be
inscriptions on pottery reveal the presence of a continuation of previous Pallava
non-Tamil speakers, mostly traders, in certain architecture with some variations. These
craft centres and towns. Traders from faraway temples amply illustrate the style of
regions were present in the Tamil country. architecture between the 8th to 12th
 Manimegalai refers to Magadha artisans, century and its influence may also be seen on
Maratha mechanics, Malva smiths and Yavana the architecture of temples of Ceylon and
carpenters working in cooperation with Tamil those of South-East Asian kingdoms like Sri
craftsmen. Vijaya (Sumatra) and Chavakam (Java). The
 Trade-related terms such as vanikan, important example of the early group is
chattanand nigama appear in the Tamil-Brahmi the Vijayalaya temple while the latter group
inscriptions. Salt merchants called represents the Brihadeeswara temple of
umanar,traveled in bullock carts along with Thanjavur. Brihadeeswara Temple is a Hindu
their families for trade activities. Chattu temple dedicated to Shiva, located in
referred to the itinerary or mobile merchants. Thanjavur in the Indian state of Tamil Nadu. It
is also known as Raja Rajeswara Temple. It is
one of the largest temples in India and is the
Q.12) Ans: B
finest example of Dravidian architecture
Exp:
during the Chola period. It was built by
About the Thirukkural:
emperor Raja Raja Chola I. The temple is part
● Thirukkural (Tamil: “Sacred Couplets''), also
of the UNESCO World Heritage Site known as
called Kural, is the most celebrated of the
the “Great Living Chola Temples”.
Patiren-kirkkanakku (''Eighteen Ethical
Works”) in Tamil literature and work that has  Statement 2 is correct: The Airavatesvara
had an immense influence on Tamil culture temple at Darasuram was built by the
and life. It was authored by the poet great Chola king Rajaraja II. Despite being a
Tiruvalluvar who lived in Tamil Nadu in the smaller temple, the Airavatesvara temple
6th century, though some scholars assign an holds sculptural masterpieces that make it
earlier date (1st-century bc). stand separately, exemplifying the great
heights achieved in the field of art,
53

sculpture, and architecture, during the Chola  Statement 1 is correct- Two inscriptions
reign. belonging to the period of Parantaka I found
 Statement 3 is incorrect: The Papanatha at Uthiramerur provide details of the
Temple was built during the 8th Century in the formation and functions of village councils.
Early Chalukya period. It is part of Pattadakal That village was divided into thirty wards and
group of monuments which is a UNESCO world each was to nominate its members to the
heritage site. This temple has features of both village council.
Dravida and Nagara styles. Pattadakal  Statement 2 is incorrect- The system of village
monuments in Karnataka, represent a high autonomy with sabhas and their
point of an eclectic art which, in the 7th and committees developed through the ages and
8th centuries under the Chalukya reached its culmination during the Chola
dynasty, achieved a harmonious blend of rule. Decentralisation of administration was an
architectural forms from northern and important feature of Chola rule.
southern India. An impressive series of  Statement 3 is incorrect- The Cholas paid
nine Hindu temples, as well as a special attention to their navy. The naval
Jain sanctuary, can be seen there. achievements of the Tamils reached its climax
under the Cholas. They controlled the Malabar
Q.14) Answer: B and Coromandel coasts. In fact, the Bay of
 Statement 1 is incorrect: Sangam literature is Bengal became a Chola lake for some time.
the compilation of the earliest available Tamil  Statement 4 is correct- Uttaramerur
literature. It implies an association of Tamil inscriptions talk about the Kudavolai system.
poets that flourished in ancient southern India. This system was a very notable and unique
The three chief Tamil kingdoms of this period feature of the village administration of the
were the Cheras, the Cholas, and the Cholas. The representative for each ward in a
Pandyas. The Sangam period roughly extends village was elected through this system.
between 300 BC and 300 AD, although most of
the work is believed to have been composed Q.16) Answer: D
between 100 CE and 250 CE. The Sangam texts  Statement 1 is correct - The Pallavas
are different from the Vedic texts, particularly introduced the art of excavating temples from
the Rig Vedic texts. They do not constitute the rock. And the Dravidian style of temple
religious literature. The short and long architecture also began with the Pallava rule.
poems were composed by numerous poets in  Statement 2 is correct - The second stage of
praise of numerous heroes and heroines. Thus Pallava architecture is represented by the
they are secular in nature. monolithic rathas and Mandapas found at
 Statement 2 is correct: The Sangam texts refer Mamallapuram. In the subsequent
to many settlements including phases/stages of the pallava architecture there
Kaveripattanam whose flourishing existence is was a gradual evolution starting from the cave
now attested archaeologically. They also temples to monolithic rathas and subsequently
speak of the Yavanas (foreigners) coming in in the structural temples.
their own vessels purchasing pepper with gold  Statement 3 is correct- Descent of the Ganga’
and supplying wine and women slaves to the or Arjuna’s Penance at Mahabalipuram is one
natives. This trade is known not only from of the best examples of the art and
Latin and Greek writings but also from architecture of the Pallavas. The Kailasanatha
archaeological records. The Sangam literature temple at Kanchi and the Shore temple at
is a very major source of our information for Mamallapuram are also one of the finest
the social, economic and political life of examples of the early structural temples of the
the people living in deltaic Tamil Nadu in the Pallavas.
early Christian centuries.
Q.17) Answer: C
Option C is correct: India under 900 AD consisted
Q.15) Answer: A of several dynasties ruling it. The Karkota
54

Dynasty established their rule in Kashmir. It o Karpuramanjari: A famous play written in


was founded by the Durlabh Vardhana during the Sauraseni Prakrit to please his wife,
times of Harshavardhana. The Pratihara Avantisundari, a woman of taste and
Dynasty also known as the Gurjara-Pratihara was accomplishment. He is perhaps the only
founded by the Harishchandra (a Brahmana) in and ancient Indian poet to acknowledge a
around the areas near Jodhpur, in South west woman for her contributions to his literary
Rajasthan. The Palas extended their rule in areas
career.
of Bihar and Bengal. The Pala dynasty was founded
o Kavya Mimansa (around c.880−920 CE): A
by Gopala in the 8th century AD. These rulers were
practical guide for poets that explains the
mainly the followers of the Mahayana and the
elements and composition of a good
Tantric school of Buddhism. The Rashtrakuta
Kingdom was founded by Dantidurga in 753 AD poem.
who was the feudatory of the Chalukya
kingdom. They ruled over the areas of the Deccan. Q.19) Ans: A
Hence, the correct geographical sequence of the Exp:
above kingdoms in the south to north direction  Dharmapala (c.770−810 CE):
is Rashtrakutas, Palas, Pratiharas, and o Though he initially suffered defeats at the
Karkota. Therefore, option 4-3-2-1 is correct. hands of the Pratiharas and the
Rashtrakutas (he was defeated by the
Rashtrakuta king, Dhruva, who had earlier
Q.18) Ans. B
defeated the Pratihara ruler too), but later
 Statement 1 is correct: The Pratiharas were he conquered large parts of northern
also known as the Gurjara−Pratiharas, as they
India and raised the Pala empire to great
originated from the Gurjaras who were
primarily pastoralists and fighters. Since the heights.
early kings of this dynasty considered the epic o He founded the Vikramshila monastery
hero Lakshmana as their hero, who served as near Bhagalpur in Bihar, and like Nalanda
the door-keeper to his brother Rama, the university, it also attracted students from
Pratiharas took on their title which literally all parts of India and from Tibet. Many
means ‘door-keeper’. The Gurjara−Pratiharas Sanskrit texts were translated into Tibetan
are known for their sculpture, their carved
language at this monastery. The most
panels and the open pavilion style temples.
The dynasty was founded by a Brahmana celebrated name associated with the
named Harichandra in and around Jodhpur, Vikramshila University was that of the
south western Rajasthan. Buddhist scholar Dipankara (called Atisa),
 Statement 2 is incorrect: Under Bhoja and his who was greatly respected in Tibet.
successor Mahendrapala I, the Pratihara o He also founded the Somapuri monastery
empire reached its peak of prosperity and (near Paharpur, Bihar) and is also credited
power. The extent of its territory was almost with the grant of 200 villages to Nalanda
similar that of the Guptas and by the time of University.
Mahendrapala, the empire reached west to o The renowned 8th century Buddhist
the border of Sindh, east to Bengal, north to scholar, Santarakshita, who is also
the Himalayas, and south past the Narmada. considered as an abbot of Nalanda,
 Statement 3 is correct: Mahendrapala took the belonged to his reign. Santarakshita
title Maharajadhiraja of Aryavarta (Great King founded the philosophical school known as
of Kings of Northern India). His court was Yogacara- Svatantrika-Madhyamika, which
adorned by Rajashekhar, who was an eminent united the Madhyamaka tradition of
Sanskrit poet, dramatist and critic who wrote: Nagarjuna, the Yogacara tradition of
55

Asanga and the logical and epistemological Q.21) Answer: C


thought of Dharmakirti.  Statement 1 is correct: The Satavahanas
retained some administrative structures
Q.20) Ans: C of Ashokan times. Their district was
Exp: called ahara, as it was known in the time of
Ashoka, and their officials were known
 Statement 1 is correct: The Pala empire's lands
as amatyas and mahamatras, as was the case
that were managed directly and were divided
in Maurya times. However,
into many provinces known as Bhuktis and their administrative divisions were also
governed by officials known as Uparika. called rashtra, and their high officials were
o Uparika collected the levy and preserved styled maharashtrikas. The lowest level of
the province's law and order. administration was a grama (village), which
o These Bhuktis (Provinces) were further was under the charge of a gaulmika.
segmented into Vishaya (Divisions) and Statement 2 is correct: There was remarkable
Mandala (Districts). progress in the fields of trade and
o Further, they were divided into smaller industry during the Satavahana rule.
units like Khandala, Bhaga, Avritti, Merchants organized guilds to increase their
Chaturaka, and Pattaka. activities. The Satavahana period also
witnessed overseas commercial
 Statement 2 is correct: Pala's administrative
activity. Ptolemy mentions many ports in the
system was based on Gupta's administrative Deccan. The greatest port of the Satavahanas
system, however more efficiently managed. was Kalyani on the west
The Pala Dynasty had a monarchical Deccan. Gandakasela and Ganjam on the east
administrative structure. All power was coast were the other important seaports.
concentrated in the hands of the king or
monarch. Q.22) Answer: C
 Option A is correct: The best account of Indo-
Roman trade is given in the book called
the Periplus of the Erythrean Sea. Major items
Extra info: of export to the Romans were the Indian
The Pala dynasty ruled from 8th century to 12th products. such
century CE in the regions comprising Bihar and as spices (included yavanapriya or pepper), p
Bengal. erfumes, jewels, ivory and fine
 Gopala founded the dynasty in 750 AD. textiles (muslin especially), several precious
o Rulers of Pala dynasty also built and semi-precious stones
magnificent monasteries and temples: like diamond, carnelian, turquoise, agate, sap
Somapura Mahavihara (in Bangladesh), phire, and so on, besides
Odantapuri Monastery. pearls, indigo, sandalwood, iron and steel
They also patronized Buddhist centers of cutlery. Against this import, Romans
learning like Nalanda University and the exported gold and silver to India.
Vikramshila University. Important rulers of Pala  Option B is correct: Though the concept of
Dynasty were: varna was known, social classes in the Sangam
 Gopala(Reign: 750-770AD), Dharmapala period were not marked by higher or lower
(Reign: 770 – 810 AD), rankings as in north India. For example,
Brahmans were present in the society and they
 Devapala (Reign: 810 – 850 AD)
performed Vedic ceremonies and sacrifices
 Madanapala (Reign: 1144 – 1162 AD) and also acted as advisers to the chief but they
After him, the Sena dynasty replaced the Palas. enjoyed no special privileges.
 Option C is incorrect: Women in the Sangam
period were educated and engaged in
various economic activities such as paddy
56

plantation, cattle rearing, basket-making, local cesses, the latter was collected by the
spinning etc. Child marriage was absent but local authorities with the central authorities
the cruel practice of Sati was also prevalent in providing them full assistance in enforcing
Tamil society, and it was known as tippayadal. their demands.
But it was not obligatory as there are  Statement 3 is correct: Analysis through
references to widows present in society. statistics and the chronology and spatial
However, their position was miserable as they distribution of tax terminology and the context
were prohibited to decorate themselves or of their occurence have revealed that the
participate in any form of amusement. major land tax ( kadamai) was standardized,
along with a number of small ones related to
Q.23) Answer: C land.
 Statement 1 is incorrect - The Pandya rulers
created a number of irrigation sources and as a Q.25) Answer: A
result agriculture flourished during the  Pulakesin I (543-566 AD) was the founder of
Pandyan times. The trade activities also the Chalukya dynasty. He established a small
flourished immensely. They exempted traders kingdom Badami (Vatapi) as its capital.
from various kinds of port dues and tolls. Kayal  Kirtivarman I (566-597 AD) was the son of
was their great port. Marco Polo, the famous Pulakesin I, and expanded the kingdom
traveller from Venice, visited Kayal twice, in by wars against the Kadambas of Banavasi and
1288 and in 1293. He tells us that this port the Nalas of Bastar.
town was full of ships from Arabia and China  Vikramaditya I (644-681 AD) drove out the
and bustling with business activities. Pallava forces, and consolidated the kingdom.
 Statement 2 is correct - The Delhi Sultanate Later he plundered Kanchi, the capital of the
ruler Allaudin Khilji, sent his general Malik Pallavas, thus avenging his father’s (Pulakesin
Kafur to the Southernmost Kingdoms. He II) defeat in the hands of Pallava ruler
invaded the Pandyan Kingdom. He plundered Narasimhavarman I.
the city of Madurai. The Madurai temple was  Vikramaditya II (733-744 AD) invaded
desecrated and an enormous amount of Pallava kingdom three times, and repelled the
wealth was looted. After Malik Kafur’s Arab invasion of South Gujarat.
invasion, the Pandyan kingdom came to be  Kirtivarman II ( 744-755 AD) was the last
divided among a number of the main rulers in Chalukyan ruler of Badami and was defeated
the Pandya’s family. by Dantidurga, the founder of the Rashtrakuta
 Statement 3 is incorrect - Cholas undertook dynasty, marking the end of the Chalukyan
measures to improve the irrigation system. For dynasty.
the periodical or seasonal maintenance and
repair of irrigation works, conscripted labour
was used. Village assemblies under the Cholas
collected a tax called eriayam, which was Students Note:
utilised for repairing irrigation tanks.

Q.24) Answer: D
 Statement 1 is correct: There was a
regular land revenue department ( Puravu
Vari tinaikkalam) during the Chola period and
a well maintained record of land rights,
surveys and keeping of fresh entries
(Varippottagam) highlighting a well organized
tax assessment and tax collection mechanism.
 Statement 2 is correct: There
was differentiation in the central taxes and the
57

Ghaznavid & Turkish Invasions, Delhi Sultanate (1200 - 1400 CE)


all the merchants.
Q.1) Answer: B
 Pair 1 is Correct - Chahalgani is a term used for Q.2) Answer: A
“the 40” Turkish chiefs. They were noblemen  Statement 1 is incorrect: Bulban was himself
who exerted great influence on the one of the Turkish chiefs and had arrogated all
kingdom. Iltutmish had shown great deference power to himself. He refused to entertain
to them. But when he chose Razia as his important government posts for anyone who
successor, these noblemen were not very did not belong to a noble family. This virtually
pleased and started making trouble for her. meant exclusion of even Indian muslims from
positions of power and authority.
 Statement 2 is correct: Although he wanted to
break the power of the chahalgani (the 40) and
didn’t like the non-turks, but to win the
public's confidence, he administered justice
with extreme impartiality. Not even the
highest in the land were spared if they
transgressed his authority.
 Statement 3 is correct: He was not prepared
to share power with anyone, even though he
considered himself a champion of the Turkish
nobility. He excluded non-Turks from the
administration. He dissatisfied many by basing
his rule over a very narrow racial group. This
led to much disturbance and troubles after his
 Pair 2 is Incorrect - Chaugan was a polo like death.
sport played from horseback. It was popular
among the aristocrats and used specially Q.3) Answer: D
trained horses, a ball and a stick. It originated  Statement 1 is correct: By the end of the 9th
in Iran and was played even in Central Asia and century, the Abbasid Caliphate was in
India. Qutubuddin Aibak had fallen from his decline and replaced by a series of states
horse and suffered injuries and ultimately died. ruled by Islamized Turks. After entering the
 Pair 3 is Correct - Paibos was a Persian empire as palace guards and mercenary
tradition made compulsory by the slave ruler soldiers, they emerged as king makers. As the
Balban/Ulugh Khan. The visitors were to kiss power of the central government declined,
the feet of the Sultan to show respect and provincial governors started assuming
deference. Another such tradition also made independent status and these new rulers
compulsory by Balban was Sajdah which assumed the title of ‘amir’ at first and of
involved a person touching the floor with his ‘sultan’ later on.
face while sitting in front of the ruler. He  Statement 2 is correct: The Turks brought with
himself was one of the slaves and had risen to them the habit of ruthless plunder, their main
become a Turkish chief. By introducing these, mode of warfare consisted of rapid advance
he wanted to show that other chiefs were not and retreat, lightning raids, and any loose body
his equals. of stragglers. They could do this because of the
 Pair 4 is Incorrect - Allaudin Khalji set out to fix excellent quality of their horses as also their
the price of all commodities from foodgrains to hardihood so that they could cover incredible
slaves and horses etc. He set up three markets distances on horseback.
for foodgrains, for clothes and for slaves,  Statement 3 is correct: Break up of Gurjara-
horses etc. Each of these was under a Pratihara empire led to a phase of political
“Shahna” who was an official to control prices uncertainty in north India, and a new phase of
and shopkeepers. He maintained a register of struggle for domination. As a result, little
58

attention was paid to the emergence of assessment of land revenue. He was the
aggressive, expansionist Turkish states on the first sultan to realize land revenue after the
northwestern border of India and in West Asia. assessment of land and measurement of
land under cultivation. The post of a
Q.4) Answer: D special officer called “Mustakhraj” was
 Kabul, Kandahar and its neighbouring area to created to collect land revenue from
the south were known as Zabul peasants in doab area and khalisa lands.
or Zamindawar, they were considered parts of  To keep the army satisfied with their salary, he
al-Hind or India till the end of the ninth started a strict price control mechanism which
century. came to be known as the market reform
 There were many Buddhist and Hindu shrines system. He fixed the prices of cattles, horses,
in the area, the most important being the 53.5 slaves. He strictly kept harsh punishments
m colossal statue of Buddha at Bamiyan, with against any kind of profiteering and this kept
caves for the residence of a thousand monks. his soldiers satisfied with their salaries.
 The area up to the river Oxus was ruled by
many dynasties, some of them claiming Q.6) Ans: C
descent from Kanishka. These kingdoms, Exp:
backed by a mixed population of local tribes,  Statement 1 is correct: Jalaluddin Khalji ruled
Hunas, Turks,exiled Iranians and Indians only for a brief period of six years. He tried to
offered stiff resistance to the Arab effort to mitigate some of the harsh aspects of Balban’s
enter the area for tribute , plunder and slaves. rule. He was the first ruler of the Delhi
Consequently there were skirmishes on both Sultanate to clearly put forward the view that
sides of the river. the state should be based on the willing
support of the governed, and that since the
Q.5) Ans: C large majority of the people in India were
Exp: Hindus, the state in India could not be a truly
 Alauddin Khilji (1296–1316) was a strong and Islamic state. He also tried to gain the goodwill
efficient ruler of Khalji (Khilji) dynasty. He set of the nobility by a policy of tolerance and
up a strong central government. He was the avoiding harsh punishments.
highest authority of the state and he believed  Statement 2 is correct: Ala-ud-din was the first
in the history of divine right. The Sultan started Sultan to pay his soldiers in cash rather than
considering himself a representative of God or give them a share of booty. As the soldiers
“Shadow of God''. Some of the reforms were paid less, the prices had to be monitored
introduced by him were: and controlled. Moreover, Ala-ud-din had to
o Alauddin Khilji maintained a strong and maintain a huge standing army. In order to
huge standing army to safeguard his restrict prices of essential commodities, Ala-
empire. ud-din set up an elaborate intelligence
o He introduced the system of branding of network to collect information on black-
horses (dagh) and maintenance of the marketing and hoarding. The transactions in
descriptive register of soldiers to prevent the bazaars, the buying and selling and the
false musters and corrupt practices. Hence bargains made were all reported to the Sultan
option (A) is correct. by his spies.
o Alauddin abolished the Jagir system and  Market superintendents, reporters and spies
paid the salaries in cash to soldiers. had to send daily reports on the prices of
o He fixed the pay of soldiers at 234 tankas a essential commodities. Violators of the price
year, with an additional 78 tankas for a regulations were severely punished. If any
soldier maintaining two horses. deficiency in weight was found, an equal
o Ariz-i-Mumalik was in charge of the weight of flesh was cut from the seller’s body
appointment of soldiers. and thrown down before his eyes!
o Alauddin Khilji introduced the scientific
method of measurement of land for the
59

Q.7) Ans: D  Statement 1 is correct: The iqta system was


Exp: not only revived but also it was made
 Statement 1 is incorrect: Alauddin Khilji took hereditary. As per the Islamic law he levied the
important steps in the land revenue taxes.
administration. He was the first Sultan of Delhi  Statement 2 is correct: Jiziya was strictly
who ordered for the measurement of land. imposed on non-Muslims. He was the first
Even the big landlords could not escape from Sultan to impose an irrigation tax. But at the
paying land tax. Ala-ud-din collected land taxes same time he dug irrigation canals and wells.
directly from the cultivators. The village The longest canal was about 200 kilometers
headman who traditionally enjoyed the right from Sutlej to Hansi. Another canal was
to collect them was now deprived of it. between Yamuna and Hissar.
 The tax pressure of Ala-ud-din was on the rich  Statement 3 is correct: A new department
and not on the poor. Ala-ud-din set up the called Diwan-i-Khairat was created to take
postal system to keep in touch with all parts of care of orphans and widows. Free hospitals
his sprawling empire. and marriage bureaus for poor Muslims were
 Statement 2 is incorrect: The reign of Firoz also established. Firoz patronized scholars like
Tughlaq was more notable for his Barani. As he was guided by the ulemas, he
administration. He strictly followed the advice was intolerant towards Shia Muslims and Sufis.
of the ulemas in running the administration. He treated Hindus as second grade citizens
He pleased the nobles and assured hereditary and imposed Jizya.
succession to their properties. Thus the iqta
system was not only revived but also it was Q.9) Ans: B
made hereditary. As per the Islamic law he Exp:
levied the taxes. Jiziya was strictly imposed on Muhammad bin Tughlaq introduced many liberal
non-Muslims. He was the first Sultan to and beneficial reforms. But all his reforms failed.
impose an irrigation tax. But at the same time Contemporary writers like Isami, Barani and Ibn
he dug irrigation canals and wells. Battutah were unable to give a correct picture
about his personality. But, Muhammad bin Tughlaq
Q.8) Ans: A was the only Delhi Sultan who had received a
Exp: comprehensive literary, religious and
 The reign of Firoz Tughlaq was more notable philosophical education.
for his administration. He strictly followed the ● Statement 1 is correct: Muhammad Tughlaq’s
advice of the ulemas in running the attempt to shiftthe capital from Delhi to
administration. He pleased the nobles and Devagiri in Maharashtra, which he named
assured hereditary succession to their Daulatabad, was a bold initiative. This was
properties. after his realization that it was difficult to rule
south India from Delhi. Centrally located,
Devagiri also had the advantage of possessing
a strong fort atop a rocky hill. Counting on the
military and political advantages, the Sultan
ordered important officers and leading men
including many Sufi saints to shift to Devagiri.
However, the plan failed, and soon
Muhammad realised that it was difficult to
rule North India from Daulatabad. He again
ordered transfer of capital back to Delhi.
● Statement 2 is correct: Equally innovative was
Muhammad Tughlaq's scheme to expand
cultivation. But it also failed miserably. It
coincided with a prolonged and severe famine
in the Doab. The peasants who rebelled were
60

harshly dealt with. The famine was linked to


the oppressive and arbitrary collection of land
revenue. The Sultan established a separate
department (Diwan-i-Amir Kohi) to take care
of agriculture. Loans were advanced to farmers
for purchase of cattle, seeds and digging of
wells but to no avail. Officers appointed to
monitor the crops were not efficient; the
nobility and important officials were of diverse
backgrounds.

Q.10) Ans: B
Exp:
 The sultans of Delhi enforced the Shariah and
also appointed the ulema to profitable offices,
they also formulated their own state laws  Statement 2 is correct: Indo-Greek rule is
called zawabit to bring in a practicality in important in the history of India because of
the large number of coins that the Greeks
solving disputes involving the people
issued. The Indo-Greeks were the first rulers in
 Aurangzeb was not interested in philosophical India to issue coins that can be definitively
debates or in mysticism—though he did attributed to particular kings. This is not
occasionally visit Sufi saints for their blessings, possible in the case of the early punch-marked
and did not debar his sons from dabbling in coins, which cannot be assigned with certainty
Sufism. While taking his stand on the Hanafi to any particular dynasty. The Indo- Greeks
were also the first to issue gold coins in India,
school of Muslim law which had been
and these increased in number under the
traditionally followed in India, Aurangzeb did Kushans.
not hesitate in issuing secular decrees, called  Statement 3 is correct: Kushanas coined the
zawabit. A compendium of his decrees, and precious metals of gold and copper into
government rules and regulations had been circular shapes and design, by die-strike
collected in a work called Zawabit-i-Alamgiri. technique, but in their own names, effigy and
Theoretically, the zawabits supplemented the style. Nevertheless, like the Indo-Greeks coins,
their original coins too had the king figuring on
sharia. In practice, however, they sometimes
the obverse and some deity, preferably Greek
modified the sharia, in view of the conditions God, on the reverse.
obtained in India which were not provided for
in the sharia. Q.12) Answer: B
 Statement 1 is correct : The Delhi Sultanate
Q.11) Answer: D had its origins in the victories of Muhammad
 Statement 1 is correct: Early Kushan kings Ghori, he defeated Prithviraj Chauhan in 1192
issued numerous gold coins with a higher AD thus establishing a firm control over north
degree of metallic purity than is found in India. When Muhammad Ghori died in 1206
the Gupta gold coins. Although the gold coins AD, his trusted general Qutubuddin
of the Kushans are found mainly west of the Aibak declared an independent dynasty in
Indus, their inscriptions are distributed not Delhi which was known a Mamluk dynasty and
only in north-western India and Sindh but also was the first in a series that became
in Mathura, Shravasti, Kaushambi, and collectively known as Delhi Sultanate.
Varanasi.  Statement 2 is incorrect : The Delhi Sultanate
consisted of five short lived dynasties of the
Sultanate based in Delhi. The first three (
Mameluk, Khalji and Tughlaq) of which were of
61

Turkish origin, the fourth was the Sayyid Q.14) Answer: B


dynasty ( Multan), and the last was the Lodhi  Ibn Battuta’s book of travels,
dynasty, which was of Afghan origin. called Rihla, written in Arabic, provides
extremely rich and interesting details about
Q.13) Answer: D the social and cultural life in the subcontinent
 Statement 1 is incorrect: Firoz Shah in the fourteenth century. Travelling overland
Tughlaq made the iqtadari system through Central Asia, Ibn Battuta reached Sind
hereditary. He made a rule that whenever a in 1333. during the reign of the Tughlaq
noble a died, his son should be allowed to dynasty. Ibn Battuta has travelled extensively
succeed to his position, including this iqta, and and has even mentioned about-the coconut
if he had no son, his son in law and in his and paan(betel leaf).
absence, his slave. He followed a lenient
approach towards the nobles in case of
revenue collection mismatch unlike his
predecessors. These steps pleased the nobles
and were a major factor for the absence of
rebellions by the nobles during his time.
 The Iqtadari system was a unique type of land
distribution and administrative system that
evolved during the reign of Iltutmish. Empire
was evenly distributed into large and small  Nicolo Conti Was from Italy who visited
tracts of land called iqtas to various nobles and Vijayanagar Kingdom during the reign
officers for better administration and revenue of Devraya I (1406-1422). Conti refers to the
collection. prevalence of slavery in society. He also
 Statement 2 is incorrect: It was established inferred that dancing, music, wrestling,
by Muhammad Bin Tughlaq to extend and gambling and cock-fighting were some of the
improve the cultivation in the doab. Under this amusements of the society.
the area was divided into development blocks  Domingo Paes was from Portugal. He visited
headed by an official whose job was to extend the Vijayanagar Kingdom during the rule
cultivation by giving loans to the cultivators of Krishnadevaraya (1509-1529). Paes
and to induce them to cultivate superior crops mentioned the beautiful houses of the rich and
–wheat in place of barley, grapes and dates in the large number of Their household servants.
place of sugarcane etc. Policy of extending and Paes also refers to the flourishing devadasi
improving cultivation was taken up by Firoz system in the kingdom.
shah Tughlak and later on to a larger scale by  Fernao Nuniz was from Portugal. He visited
Akbar. Vijayanagar Kingdom during the rule of Achyut
 Statement 3 is correct: After his accession, Devraya (1529–1542). According to Nuniz, a
Firoz Shah Tughlaq was faced with the problem large number of women were employed in
of preventing the imminent break up of royal palaces as dancers, domestic servants
theDelhi Sultanate. He adopted a policy of and palanquin bearers. Nuniz also provided a
trying to appease the nobles, the army and the description of the Sati system in society. He
theologians and asserted his authority only also mentioned excavation of canals and
over such areas which could be easily irrigation facilities.
administered from Delhi. He therefore
made no attempt to reassert his authority over Q.15) Ans: A
South India and Deccan. He led two campaigns Exp:
into Bengal, but was unsuccessful in both and Administration of Delhi Sultanate:
resulted in the loss of Bengal. ● In the Delhi Sultanate, the village was the
smallest unit of administration. The
functioning and administration of the village
remained more or less the same as it had
62

existed in pre-Turkish times. The main village ● Control of prices of horses was very important
functionaries were khut, Muqaddam and for the Sultan because without the supply of
Patwari. They worked in close coordination good horses at reasonable prices to the army,
with the muqti in the collection of revenue and the efficiency of the army could not be
in maintaining law and order etc. ensured. Low prices in the horse market were
● A number of villages formed the Pargana. The ensured by putting a stop to the purchase of
important Pargana officials were Chaudhary, horses by horse dealers and brokers (dalals) in
Amil (revenue collector) and Karkun the Delhi market.
(accountant). Villages and pargana were
independent units of administration, and yet
there were interrelated areas. Q.17) Ans: C
● In certain cases the province had a local ruler Exp:
(Rai, Rana, Rawat, Raja) who helped the ● Statement 1 is correct: Delhi Sultanate was
governor in his duties. In such cases the local formally an Islamic State. Qazis headed
rulers were recognised as subordinates of the various posts in the Justice Department under
Sultan. the Sultanate. The head Qazi headed the
Department of Justice. The qazis dispensed
Q.16) Ans: D civil law based on Muslim Law (Sharia). The
Exp: Hindus were governed by their own personal
● Alauddin’s measures to control the markets laws which were dispensed by the Panchayats
were one of the most important policy and Guilds in the villages and by leaders of the
initiatives. Ala-ud-din was the first Sultan to various castes.
pay his soldiers in cash rather than give them a ● Statement 2 is correct: Criminal Law was based
share of booty. As the soldiers were paid less, on regulations framed for the purpose by the
the prices had to be monitored and controlled. rulers. The sultans had to supplement the
● Since Alauddin wanted to maintain a large Muslim Law by framing their own regulations
army, he, therefore, lowered and fixed the (zawabit).
price of the commodities of daily use. To
control the prices, Alauddin set up three
different markets for different commodities in
Students Note:
Delhi.

● These markets were the grain market (Mandi),


cloth market (Sarai Adl), and the market for
horses, slaves, cattle, etc. To ensure
implementation, Alauddin appointed a
superintendent (Shahna-i-Mandi) who was
assisted by an intelligence officer.
63

Vijayanagara & Bahmini kingdom

Q.1) Answer: B Dynasty. He was a person of high intellectual


 Statement 1 is Incorrect - The Deccan states quality who ruled from 1509 to 1529 A.D.
(Bijapur, Ahmadnagar, Golkonda and Bidar, Babur defeated Ibrahim Lodhi in the first battle
except Berar) formed a confederacy and of Panipat and established Mughal dynasty in
inflicted a crushing blow on the Vijayanagar 1526 A.D. It lasted till the establishment of
armies at Bannihatti in the battle of Talaikotta British rule in India. Therefore Babur was the
in c.1565 CE. This battle is also known as contemporary of Krishna Deva Raya.
Rakshasa Thangadi. Rama Raya was
imprisoned and executed and the city of Q.3) Answer: B
Vijayanagar was destroyed.  Option B is correct - According to to the
 Statement 2 is Incorrect - Muskets, a superior accounts of foreign Travellers the Vijayanagar
military technology, were used for the first Empire was one of the wealthiest parts of the
time by Portuguese and they were able to
world at a time there were numerous
expand their territory in India in the reign of
Vijayanagara Kingdom. industries and they were organised into guilds
Vijayanagar was also a great centre of trade
and there were a number of sea ports on the
Malabar Coast the chief being Cannanore. The
chief items of exports were cotton and silk
clothes, spices,rice, iron, saltpeter and sugar.
The imports consisted of horses, pearls,
copper, Coral, Mercury, China silk and velvet
clothes.

 Statement 3 is correct. Foreign traveller Abdur Q.4) Answer: D


Razaqq noted that Vijayanagara Kingdom  Pair 1 is correct: Madhura Vijayam meaning
fortified not only the city but also the "The Victory of Madurai", is a 14th-century C.E
agricultural hinterland and forests for Sanskrit poem written by the poet Gangadevi.
protecting the agricultural belts. It chronicles the life of Kumara Kampana. The
poem describes Kumara Kampana’s invasion
Q.2) Answer: C and conquest of the Madurai Sultanate.Ganga
 Statement 1 is correct: Vijayanagar Kingdom Devi was the wife of Kumar kampana.
was founded by Harihara and Bukka In the year  Pair 2 is correct: Allasani Peddana was a
1336 A.D. Harihara and Bukka were famous poet and was ranked as the foremost
feudatories under the Kakatiya of of the Ashtadiggajas in the court of Krishna
Warangal.Later they became ministers in the Deva Raya. He has authored Manucharitra. The
court of Kampili in modern day theme for his Manu Charitra is a short story
karnataka. They found a new city on the South from Markandeya Purana.
Bank of Tungabhadra river which was called  Pair 3 is correct: Krishna Deva Raya was a great
Vijayanagar thus laying the foundation of the scholar of Sanskrit and Telugu language. He has
new empire. The Tughlaq dynasty was founded written a book Amuktamalyada in Telugu. This
by Ghiyasuddin Tughlaq who ruled over the
is a work on Polity.In his work Krishnadevaraya
Delhi sultanate in medieval India.His reign
had mentioned that a King should rule with
started in 1320 in Delhi when Ghiyasuddin
Tughlaq overthrew Khilji dynasty. great care and should attend to the work of
 Statement 2 is correct: The greatest of the protecting the good and punishing the wicked.
Vijayanagara rulers, He also suggested the king to levy taxes from
Krishnadevaraya belonged to the Tuluva
64

his people moderately.  Statement 3 is incorrect: The Vijayanagar


kingdom was successively ruled by four
Q.5) Answer: A dynasties over a period of more than three
 Statement 1 is correct: The rulers of the hundred years: the Sangama dynasty (1336–
Vijayanagara Empire were great patrons of art 1485), the Saluva dynasty (1485–1505), the
and architecture. Under them the architectural Tuluva dynasty (1505–1570) and the Aravidu
style started to be influenced by Indo-Islamic dynasty (1570–1650).
style of Bijapur which was reflected in temples
of that period.The walls of the temples were Q.7) Ans: A
highly decorated with geometrical patterns. Exp:
Beautiful carvings were also made on them.  Among those who exercised power in the
 Statement 2 is correct: Gopuram is a empire were military chiefs who usually
monumental entrance tower, usually ornate, controlled forts and had armed supporters.
at the entrance of a temple in the Dravidian These chiefs often moved from one area to
architecture.Gopurams were previously another, and in many cases were accompanied
present on the front side. The Vijaynagar by peasants looking for fertile land on which to
architecture had a characteristic feature of settle. These chiefs were known as Nayaks
having Gopurams on all sides of the Temple. and they usually spoke Telugu or Kannada.
 Statement 3 is incorrect: Shrines in the shape  Statement 1 is correct: Nayakas collected
of intricately shaped designs were a feature of taxes and other dues from peasants,
Hoysala art. In hoysala art multiple shrines craftspersons, and traders in the area. They
were built around the central pillared hall. This retained part of the revenue for personal use
was called stellate plan.The chief and for maintaining a stipulated contingent of
characteristics of the Vijayanagara horses and elephants.
architecture were the construction of tall Raya  These contingents provided the Vijayanagara
Gopurams or gateways and the kings with an effective fighting force with
Kalyanamandapam with carved pillars in the which they brought the entire southern
temple premises. peninsula under their control. Some of the
revenue was also used for the maintenance of
Q.6) Ans: A temples and irrigation works.
Exp:  Statement 2 is incorrect: The amara-nayakas
 Statement 1 and 2 are correct: The two sent tribute to the king annually and
brothers Harihara and Bukka laid the personally, appeared in the royal court with
foundation of Vijayanagara kingdom in about gifts to express their loyalty. Kings occasionally
1336. Initially the capital was in or near about asserted their control over them by
Anegondi on the north bank of the transferring them from one place to another.
Tungabhadra river. But later it was shifted to However, during the course of the
the Hoysala town Hosapattana (near Hampi) seventeenth century, many of these nayakas
on the south bank. The capital was expanded established independent kingdoms.
and renamed Vijayanagara, the city of Victory.  This hastened the collapse of the central
Vijayanagara rulers adopted the emblem of imperial structure. The Chola traditions of
the Chalukyas, the boar, or varahaas their village self-government were considerably
royal insignia. weakened under the Vijayanagara ruler. The
growth of hereditary nayakships tended to
curb their freedom and initiatives.

Q.8) Ans: A
Exp:
 Krishnadevaraya is considered the greatest of
the Vijayanagar kings. He built upon the strong
military base laid by his father and elder
65

brother. He tried to keep the greatness of the  Statement 3 is Incorrect - Moroccon


kingdom intact, by undertaking many military traveller Ibn Batuta visited India during the
expeditions during much of his reign. reign of Mohammed Bin Tughlaq and not
Iltumish. He wrote a book on accounts of his
visit named - “Rihla ''

Q.10) Answer: C
 Option 1 is correct: Nicolo Conti Was from Italy
who visited Vijayanagar Kingdom during the
reign of Devraya I. Conti refers to the
prevalence of slavery in society. He also
 There are some other reasons for the inferred that dancing, music, wrestling,
celebration of Krishnadevaraya as the greatest gambling and cock-fighting were some of the
ruler of Vijayanagar. He made very large amusements of the society.
donations to many of the greatest Siva and  Option 2 is correct: Paes was from Portugal. He
Vishnu temples of the day- Srisailam, Tirupati, visited the Vijayanagar Kingdom during the
Kalahasti, Kanchipuram, Tiruvannamalai, rule of Krishnadevaraya. Paes mentioned the
Chidambaram, etc. He added towering beautiful houses of the rich and the large
gopuras to many of those temples, which number of Their household servants. Paes also
survive to this day. refers to the flourishing devadasi system in the
 Contemporary foreign visitors like Paes and kingdom.
Nuniz, who visited Vijayanagar left glowing  Option 3 is correct: Fernao Nuniz was also
tributes to his personality, and the grandeur from Portugal. He also visited Vijayanagar
and opulence of the city. His court was also Kingdom during the rule of Achyut Devraya.
adorned by some great poets like Allasani According to Nuniz, a large number of women
Peddana. He himself is considered a great were employed in royal palaces as dancers,
scholar and is author of the famous poem domestic servants and palanquin bearers.
Amuktamalyada (the story of Andal). Nuniz also provided a description of the Sati
 But his crowning achievement, as a clever system in society. He also mentioned
administrator, was the reorganization of the excavation of canals and irrigation facilities.
Nayak or nayankara system and giving legal  Option 4 is correct: Abdur Razzaq was from
recognition to the system. Persia. He visited Vijayanagar Kingdom during
the reign of Devraya II. He had elaborated
about the magnificence of cities. He also
Q.9) Answer: A mentioned that the country was fairly
 Statement 1 is correct - Duarte Barbosa along cultivated and very fertile. He also mentioned
with Domingos Paes was a Portuguese that there were a heavy number of soldiers in
traveller who visited Krishnadeva Raya's the empire.
court in the 16th century. Both recorded the
development of city Hampi along with socio
economic strategies of development by Q.11) Ans: A
comparing it with Rome. Exp:
 Statement 2 is Incorrect - Francois Bernier was ● Rama Raya (died 23 January 1565 CE), known
a contemporary of Mughal Empire Shah as "Aliya" (son-in-law in Kannada) was a
Jahan. He was physician to Prince Dara Shikoh statesman of the Vijayanagara Empire, the
and later was attached to the court of son-in-law of Emperor Krishna Deva Raya and
Aurangzeb . ‘Travels in the Mughal the progenitor of the Aravidu dynasty of
Empire’ was written by Francois Bernier which Vijayanagar Empire, the fourth and last
talks about the rules of Dara Shikoh and dynasty of the empire.
Aurangzeb.
66

● As regent, he was the de facto ruler of the pulse legumes grew in semi-arid regions, while
empire from 1542 to 1565, although legally the sugarcane, rice, and wheat thrived in rainy
emperor during this period was Sadasiva Raya, areas. Betel leaves, areca (for chewing), and
who was merely a puppet ruler. coconut were the principal cash crops, and
large-scale cotton production supplied the
weaving centers of the empire's vibrant textile
industry.
● According to Abdur Razzak, much of the
empire was fertile and well cultivated. Most of
the growers were tenant farmers and were
given the right of part ownership of the land
over time. Tax policies encouraging needed
● Rama Raya was killed at the Battle of Talikota, produce made distinctions between land use
after which the Vijayanagara Empire got to determine tax levies.
fragmented into several semi-independent ● For example, the daily market availability of
principalities paying only nominal allegiance to rose petals was important for perfumers, so
the empire. cultivation of roses received a lower tax
assessment. Salt production and the
Q.12) Ans: B manufacture of salt pans were controlled by
Exp: similar means. The making of ghee (clarified
● The chronological order of the major Dynasties butter), which was sold as an oil for human
which ruled the Vijayanagara Empire is consumption and as a fuel for lighting lamps,
Sangma, Saluva, Tuluva, Aravidu. The was profitable.
Vijayanagar Empire was established in 1336 AD ● Exports to China intensified and included
was founded by the rulers namely Harihara I cotton, spices, jewels, semi-precious stones,
and his brother Bukka Raya I. ivory, rhino horn, ebony, amber, coral, and
● The Sangma dynasty was succeeded by the aromatic products such as perfumes.
Saluva dynasty which was the second dynasty
to rule the Vijaynagar Empire as well as almost
the whole of South India from 1485 AD to Students Note:
1505 AD.
● The Saluva dynasty was succeeded by the
Tuluva Dynasty which was the third dynasty
that ruled the Vijayanagar Empire from 1491
AD to 1570 AD and the most famous king of
the Vijayanagar empire, Krishna Deva Raya
belonged to this dynasty.
● The Aravidu Dynasty was the fourth and last
Hindu dynasty to rule the Vijayanagar kingdom
in South India due to the reason that although
the empire continued to exist till 1646 AD, it
lost its importance in 1565 AD after a key
military defeat (The battle of Talikota) by the
Deccan Sultanates.

Q.13) Ans: D
Exp:
● The Vijayanagara Empire was based in the
Deccan Plateau region in South India. The
economy of the empire was largely dependent
on agriculture. Sorghum (jowar), cotton, and
67

Mughal Empire, Rise of Marathas

Q.1) Answer: C  Statement 2 is incorrect: Risal -i-usaz was the


 Akbar made some experiments in the land bunch of letters written by Babur. He also
revenue administration with the help of Raja wrote his autobiography Tuzuk -i- Baburi in
Todar Mal. The land revenue system of Akbar turki. Tuzuk-i-Baburi was also known as
was called Zabti or Bandobast system. Baburnama.
 It was further improved by Raja Todar Mal and
came to be known as the Dahsala System. Q.4) Ans: B
Under it revenue was fixed on the average Exp:
yield of land assessed on the basis of the past  Statement 1 is correct: Sher shah was the
ten years. Payment of revenue was made founder of the Sur dynasty. He developed the
generally in cash. Brilliant administrative system. Sher Shah’s
The land was also divided into four categories empire was divided into forty seven sarkars.
– Polaj (cultivated every year), Parauti (once in Chief Shiqdar (law and order) and Chief Munsif
two years), Chachar (once in three or four (judge) were the two officers in charge of the
administration in each sarkar. Each sarkar was
years) and Banjar (once in five or more years).
divided into several parganas. Shiqdar
(military officer), Amin (land revenue), Fotedar
(treasurer) Karkuns (accountants) were in
Q.2) Answer: A charge of the administration of each pargana.
 Statement 1 is Correct : The word ‘Peshwa’ There were also many administrative units
probably originated in Persian, meaning called iqtas.
‘foremost’, and was introduced in Deccan by
the Muslim rulers. Duties of a Peshwa
were equal to that of a Prime Minister. The
office of Peshwa existed from 1640 - 1818 CE.
Sonopant Dabir (c.1640–1652 CE) was the First
unofficial Peshwa.
 Statement 2 is Incorrect : Ramchandra Pant
Amatya (c.1689–1708 CE) was a Peshwa under
Rajaram. He wrote Adnyapatra, in which he
explained different techniques of war,
maintenance of forts, and administration.
 Statement 2 is correct: The military
administration was also efficiently reorganized
Q.3) Ans: D and Sher Shah borrowed many ideas like the
Exp: branding of horses from Alauddin Khalji.
 Statement 1 is incorrect: Alam khan was the  Statement 3 is correct: The land revenue
uncle of Ibrahim lodi who invited Babur to administration was well organized under Sher
invade India.He was pretender of the throne. Shah. Land survey was carefully done. All
The first battle of Panipat took place on April cultivable lands were classified into three
21,1526. It was fought between Babur and classes – good, middle and bad. The state’s
Ibrahim Lodi. In north India Babur had specific share was one third of the average produce
Military tactics including use of gunpowder, and it was paid in cash or crop. His revenue
firearms and field artillery. Though , Gun reforms increased the revenue of the state.
Powder was not used for the first time by Sher Shah introduced new silver coins called
Babur- Gunpowder and gunpowder weapons “Dam” and they were in circulation till 1835.
were transmitted to India through the
Mongol invasions of India.
68

Q.5) Ans: C the destruction of the Deccan kingdoms was a


Exp: political blunder as it removed the barrier
 Statement 1 is correct: In 1738-1739, Nadir between Mughals and Marathas and so there
Shah descended upon the plains of northern was a direct confrontation between them.
India. Nadir Shah was attracted to India by the Deccan campaigns exhausted the Mughal
fabulous wealth for which India was always treasury.
famous. Disunity, poor leadership, and mutual
jealousy and distrust resulted in defeat. The Q.7) Ans: B
invader inflicted a crushing defeat on the Exp:
Mughal army. Inflicted immense damage on  Statement 1 is incorrect: Mughal painting is a
the Mughal Empire.Caused an irreparable loss particular style of South Asian painting, which
of prestige and exposed the hidden weakness is generally confined to miniatures, and
of the Empire to the Maratha sardars and the emerged from Persian painting. Jahagir
foreign trading companies. The Emperor encouraged paintings depicting events of his
Muhammad Shah was taken prisoner. own life, individual portraits, and studies of
birds, flowers and animals, scenes of hunting,
battles and royal courts. He was deeply
influenced by European paintings. He
employed a number of painters like Abul
Hasan, Bishan Das, Madhu, Anant, Manohar,
Govardhan and Ustad Mansur. The
Jahangirnama, written during his lifetime,
which is an autobiographical account of
Jahangir's reign, has several paintings,
including some unusual subjects such as the
 Statement 2 is correct: Due to successful union of a saint with a tigress, and fights
invasion by Nadir shah ,Invader took between spiders.
possession of the royal treasury and other  Statement 2 is correct: Several new forms of
royal property of the Mughal Empire and music such as Tarana, Dadra and Ghazal came
plundered the rich of Delhi. India lost its into existence during mughal era.Tabla and
famous Koh-i-Noor diamond and the jewel- sitar became popular during this period. Akbar
studded Peacock Throne of Shahjahan. patronized Tansen of Gwalior. Tansen
composed many ragas. Tansen was a
Q.6) Ans: C prominent Hindustani classical music
Exp: composer, musician and vocalist. He was a
 Statement 1 is correct: Aurangazeb was a disciple of Swami Haridas.
staunch and orthodox Muslim in his personal
life. He created a separate department to Q.8) Answer: C
enforce moral codes under a high-powered
 Pair 1 is Incorrect - Al- Beruni was a Persian
officer called Muhtasib. Drinking was
Scholar and polymath of the 11th century.
prohibited by him. Cultivation and use of
bhang and other drugs were banned.
Aurangazeb forbade music in the Mughal
court.
 Statement 2 is correct: Aurangzeb (during
Shah Jahan’s reign), as governor of Deccan,
followed an aggressive Deccan policy. He
invaded Bijapur and Golconda to contain the
spread of the Marathas. In Bijapur he defeated He accompanied Mahmud in his conquests to
Sikandar Shah and annexed Bijapur. In the South Asia. His book Tahqiq-i-hind is a work
case of Golconda, he eliminated the Kutb Shahi related to Indian Philosophy and religion. Fa-
dynasty and annexed the Golconda also. But
69

Hien wrote a book called Records of Buddhist 7. Sardeshmukhi was an additional levy of ten
Kingdoms. percent on those lands which the Marathas
claimed hereditary rights.
 Pair 2 is correct - Huen Tsang or
Xuanzang was a Chinese Buddhist Traveler. Q.10) Ans: A
The title of his work is “Journey to the Exp:
West” or Si-yu-ki or ‘The Records of the ● Pair 1 is correct: Ibrahim Lodi (1517 -
Western World’ 1526) succeeded Sikandar Lodi. He was
 Pair 3 is correct - Francois Bernier was a arrogant. He insulted his nobles openly in court
French physician who visited India during the and humiliated them and those who revolted
reign of Shah Jahan. He was a physician to were put to death. Daulat Khan Lodi, the
Prince Dara Shikoh and later was attached to governor of Lahore was insulted and
the court of Aurangzeb. Francois Bernier wrote discontent emerged between them. Greatly
“Travels in the Mughal Empire’ describing the displeased by the arrogance of Ibrahim, Daulat
rules of Dara Shikoh and Aurangzeb. Khan Lodi along with Ibrahim’s uncle, Alam
 Pair 4 is Incorrect - Al- masudi was Arab khan, plotted with Babur, the king of Kabul to
historian sometimes referred to invade India. Babur marched against Delhi and
as “Herodotus of Arabs” who wrote several ● Pair 2 is incorrect: Akbar One of the greatest
texts on history and philosophy such as Muruj- monarchs of India Succeeded the throne after
al-Zahab explaining his journey. his father Humayun’s death. Afghan’s
commander-in-Chief, Hemu, had sieged Delhi
and so Akbar’s position was dangerous. In the
Q.9) Ans: A 2nd Battle of Panipat in 1556, Hemu was
Exp: almost on the point of victory. But an arrow
Chauth and sardeshmukhi were the taxes pierced his eye and he became unconscious.
collected in the Maratha kingdom: His army fled thinking that the king had died.
Thus, the battle was won by Akbar.
● Pair 3 is incorrect: Major conflict had started
between Ahmad Shah Abdali and Maratha for
control of North India in 1761. Najib-ud-
daulah of Rohilkhand and Shuja-ud-daulah of
Awadh had suffered at the hands of the
Maratha sardars. Both had made alliance with
Ahmad Shah Abdali Peshwa had dispatched a
1. The revenue system of Shivaji was based on
powerful army under the nominal command of
that of Malik Amber of Ahmednagar. his son Vishwas Rao, and actual command in
2. Lands were measured by using the measuring hands of his cousin Sadashiv Rao Bhau to fight
rod called kathi. with Ahmad Shah. European style infantry and
3. Lands were also classified into three categories artillery under command of Ibrahim khan Gardi
– paddy fields, garden lands and hilly tracks. was part of the force. In battle, Marathas were
4. He reduced the powers of the existing defeated . This battle proved very costly to
Marathas as they lost the cream of their army.
deshmukh and kulkarnis.
It gave an opportunity to English to consolidate
5. He appointed his own revenue officials called itself in Bengal and South India . The way was
karkuns. cleared for the rise of British power.
6. Chauth was one fourth of the land revenue
paid to the Marathas in order to avoid the
Maratha raids. Students Note:
70

Religious & Cultural Development in India Bhakti Sufi movements etc

Q.1) Answer: C
 Statement 1 is correct - In the later Q.2) Answer: C
period, Akbar, the Mughal Tevaram
emperor, appreciated Sufi doctrines which  It is a collection of the first seven volumes of
shaped his religious outlook and religious Tirumurai - a saiva narrative of epic and
policies. Akbar’s Sulh-i kul or “universal peace’’ puranic heroes, as well as a religious
idea of tolerance and religion called Din Ilahi or biographies of early Saiva saints set in
Divine Faith were influenced by Sufism. devotional poetry.
 It contains the works of three Tamil Shaiva
poets (Nayanars) - Appar, Sambandar, and
Sundarar. Appar and Sambandar lived around
the 7th century and Sundarar lived in the 8th
century.
 Information about these trio comes mainly
from the Periya Puranam, the 11th century
Tamil book on the Nayanars that is the last
volume (12th) of the Tirumurai.
 Statement 2 is incorrect - Sufism was a liberal  The first two poets are also mentioned in the
reform movement within Islam. Sufism third poet Sundarar's Tiruttondartokai.
stressed the elements of love and devotion as  Along with the Tevaram, Tiruvacakam is an
effective means of the realisation of God. Love important source for understanding the Saiva
of God meant love of humanity and so the Sufis Bhakti movement in early medieval South
believed service to humanity was tantamount India.
to service to God. Subramaniya Bharathiyar
 Statement 3 is correct - According to them one  Subramaniya Bharathiyar was a poet, freedom
must have the guidance of a pir or guru, fighter and social reformer from Tamil Nadu.
without which spiritual development is He was born 11th December 1882 and died on
impossible. Other ideas emphasised by Sufism 11th September 1921.
are meditation, good actions, repentance for
sins, performance of prayers and pilgrimages,
fasting, charity and suppression of passions by
ascetic practices.
Extra Edge by PWOnlyIAS

 Sufism sprang from the doctrine of Wahadut-


ul-wajud (unity of being) propounded by Ibn-
ul-Arabi [1165-1240AD]. It had its origin in
Persia and spread into India in the eleventh  He began his career as a journalist and as
century. a sub-editor in “Swadesamitran” in
 Sufi orders are broadly divided in two : Ba-sara November 1904. His songs on nationalism and
– Those followed Islamic law & Be-shara- who freedom of India appealed to the masses to
did follow Islamic law. support the Indian Independence Movement
 Sufism was divided into 12 orders (Silsila) and in Tamil Nadu.
each under a mystic Sufi saint of which 4 most  He also outlined his vision for a free India. He
popular ones were Chistis, Suhrawardis, published the sensational “Sudesa
Qadiriyahs and Naqshbandis. Geethangal” in 1908.
 His important poems - Kannan Pattu, Nilavum
Vanminum Katrum, Panchali Sabatam and
71

Kuyil Pattu etc. Padmavat was written around 1540 and


AlauddinKhilji died in 1316. According to the
Q.3) Answer: B poem, Padmavati was born in 1540 which is
 Pair 1 is incorrectly matched: Nalayira 224 years after Khalji’s death as per historical
Divyaprabandham is a composition by Alvar records.
saints of Tamil Nadu and is described as Tamil  Khalji defeated the Rana of Chittor in 1303 and
Veda. The Alvar saints are known for their died in 1316, but no one by the name Padmini
affiliation to the Sri Vaishnava tradition of or Padmavati existed then. There are just two
Hinduism. historical facts relevant to the story - i) Khalji’s
 Pair 2 is correctly matched: Andal was a attack on Chittor, ii) Ratan Sen’s defeat.
woman Alvar (devotee of Vishnu) whose
compositions were widely sung in Tamil Nadu. Q.5) Answer: B
Andal, Nachiyar and Kothai are the only female  The term Bhakti literally means devotion or
Alvar among the 12 Alvar saints of South India. passionate love for God. It was a religious
Andal is credited with the great Tamil works, reform movement during medieval times
Thiruppavai and Nachiar Tirumozhi, which are which emphasized single-minded intense
still recited by devotees during the winter devotion to God. It was initiated by Shaiva
festival season of Margazhi. Nayanars & Vaishnavite Alwars in south India,
 Pair 3 is correctly matched: Tevaram is a later spread to all regions. It was based on the
collection of the first seven volumes of following features:
Tirumurai - a saiva narrative of epic and o Supported monotheism.
puranic heroes, as well as a religious o Guru as preceptors and guides.
biographies of early Saiva saints set in
o Unity of God or one God though
devotional poetry. It contains the works
known by different names.
of three Tamil Shaiva poets (Nayanars)
- Appar, Sambandar, and Sundarar. Appar and o Condemnation of rituals, ceremonies
Sambandar lived around the 7th century and and blind faith.
Sundarar lived in the 8th century. o Rejection of idol worship.
o Surrender of oneself to God.
Q.4) Answer: D o Emphasized both Nirguna and Saguna
 Statement 1 and 2 are correct: Malik bhakti.
Muhammad Jaisi was a Sufi poet during Bhakti o Salvation through Bhakti.
Movement. He wrote the famous epic o Open-mindedness about religious
Padmavat in Awadhi language. The poem matters.
narrates that a princess of unparalleled beauty
o Rejected castes distinctions & believed
called Padmini lived in the kingdom of
in equality of all humans.
Simhaladvipa, now Sri Lanka. Enamoured by
her beauty, King Ratansen of Chittor was o Rebelled against the upper caste’s
engulfed with the passion to acquire her and domination and the Sanskrit language.
overcame a large number of adventurous o Use of local or regional languages for
obstacles to make her his queen. Preaching.
o Creation of literature in the local
language.

Q.6) Answer: D
 Chaitanya (1436- 1533) was responsible for
the popularity of Vaishnavism in Bengal
through his kirtanas, he spread the message
that ragamarga or the path of spontaneous
 Statement 3 is incorrect: He was not a love was best for salvation. He believed in the
contemporary of AlauddinKhilji because bhedabheda, i.e. dualistic non- dualism.
72

evolved into a well developed movement by


the 11th century. The khanqah (the hospice)
was the centre of activities of the various sufis
orders. The khanqah was led by a shaikh, pir or
murshid (teacher) who lived with his murids
(disciples). In the 10th century Sufism spread
 Surdas (1483- 1533) was the disciple of
Vallabhacharya of south India, he popularised across important regions of the Islamic empire.
the Krishna cult in U.P by singing songs Iran, Khurasan, Transoxiana, Egypt, Syria and
glorifying Krishna’s childhood. He was the Baghdad were important Sufi centers.The Sufi
author of Sur Sagara, Sur Saraswati, etc. movement in India commenced in the 11th
 Mirabai (1498-1569) was a great devotee of century A.D.
Krishna and popularised his cult in Rajasthan  Statement 2 is correct: ‘Sufism’ is a term used
through her songs. to refer to mystical religious ideas in Islam.
 Sankaradeva (1463- 1568) a contemporary of Sufis, stress on the importance of traversing
Chaitanya, he spread Vaishnava bhakti in the path of the Sufi pir enabling one to
Assam. establish a direct communion with the
 Tulsidas (1532-1623) was a worshipper of divine.The Sufis were very liberal in their
Rama and composed the famous religious outlook. They preached spirituality
Ramcharitmanas in Hindi, explaining in detail
through music and doctrines that professed
the various facets of Hindu dharma.
union with God.
 Statement 3 is correct: The Sufis believed in
Q.7) Answer: A the essential unity of all religions.Their sense of
 Statements 1, 3 and 4 are correct - Nirguna piety, tolerance, sympathy, concept of equality
saints believed in the formless God. Kabir, and friendly attitude attracted many Hindus,
Nanak, Raidas, Dhanna, Sena, Pipa, Dadu were mostly from lower classes, to Islam. Sufi saints
nirguna saints. They were against the caste such as Moinuddin Chisti, Nizamuddin Auliya,
system and attacked the privileges of the Fariduddin Ganj-e-Shakar were the pioneer
brahmins and were liberal in social matters. sufïs who are still loved, respected and
 They called for pure and intense devotion to honoured in India.The Sufis emphasized
God for attaining salvation and spoke out respect for all human beings.
against the social evils and called for an end to
formalities of religion such as fasts, ceremonies
and rituals. Q.9) Ans: C
 Statement 2 is incorrect - It's poetry  Statement 1 is incorrect: The Bhakti
is Gyanshrayi which has its roots in knowledge. Movement was a reform movement in
Whereas, saguna poetry is known as Hinduism. It occupies a significant position in
premashrayi which has its roots in love. bringing about harmony and normal
relationships between the Hindus and
Muslims. The development of the Bhakti cult
Q.8) Ans: D
first began in South India in the 7th-8th century
Exp:
in order to bridge the gulf between the Shaivas
 Statement 1 is correct: Sufism(Socio-religious
and the Vaishnavas. It was initiated by Shaiva
movement) or mysticism emerged in the 8th
Nayanars & Vaishnavite Alwars in south India,
century. Fundamental to sufism is God, Man
later spread to all regions.
and the relation between them that is Love.
 Statement 2 is correct: The Bhakti Saints and
They believe that from man emerged the
teachers came from a variety of backgrounds,
theories of ruh (soul), qurbat (divine proximity)
but the majority were from the lower castes.
and hulul (infusion of the divine spirit). It had
They emphasize the importance of tolerance
73

among humanity and religions. It gained language. This led to the development of the
popularity solely because it attacked the caste local and vernacular languages.The Bhakti and
structure and the dominance of Brahamanas. the Sufi saints criticised the existence of rituals
The Bhakti Movement arose as a result of in Indian society. Both these movements
religious and worship object rigidity. rejected the rigid caste system.
 Statement 3 is correct: Bhakti movement
preached using the local languages so that the Q.11) Ans: B
message reached the masses.The Bhakti saints Exp:
did their entire teaching in the local vernacular  Statement 1 is correct: Ekasarana Dharma
language to make it comprehensible even to (Shelter-in-One religion) is a neo-Vaishnavite
simple minds.This led to the development of monolithic religion propagated by Sankardeva
the local and the vernacular language and in the 15th-16th century in the Indian state of
regional literature. Regional languages such as Assam. The Neo- Vaishnavism Philosophy was
Hindi, Bhojpuri, Maithili and Oriya became enlightenment of the human conscience
extremely popular among masses. through universal love. People of the so called
untouchables or lower caste also came in
Q.10) Ans: B contact with the highest castes in Assamese
Exp: villages and they could take part in all functions
 Statement 1 is correct: Both Sufi and Bhakti of the villages.He was a devotee of 'Ekasarana'
movements emphasised on the feeling of (One God) in the guise of Krishna.
universal brotherhood and religious tolerance.  Statement 2 is correct: He was well-known for
As a result, an environment of mutual love and establishing and reorganizing a variety of
respect was created among different sections traditional art forms such as Ankia Nat,
of society.The Bhakti and Sufi movements Bhaona, Borgeet, and Sattriya dance. He was
showed the people that the existence of God instrumental in popularizing Vaishnavism in
could be experienced under the guidance of a Assam.He also called Krishna by several names,
guru. Priests or ulemas are not required to feel including Hari, Narayan, and Rama.
the presence of God. Both the Bhakti and the  Statement 3 is incorrect: Shankaradeva
Sufi movement made their focus on the idea Spread the Bhakti cult in Assam.He wrote in
that God is one and one should do prayers and Assamese and Brajavali (mixture of Maithili
meditation to contact god.Both preached love and Assamese). His most famous work is
to God as their central principle. “Kirtana Ghosha”, written in a simple language
 Statement 2 is incorrect: Sufism movement understood by the masses.
was largely followed by Muslims while Bhakti
movement was mostly followed by Hindus.
Q.12) Ans: C
Sufism believe in the conversion of Hindu into
Islam.Bhakti movement didn't believe in Exp:
religion conversion. The Bhakti movement has  Madhvacharya was the third of the trinity of
originated in eighth-century South India. The philosophers who influenced Indian thoughts
Origins of Sufism can be traced to the early after the ages of the Vedas and Puranas (the
days of Islam in seventh-century Arabian other two being Shankaracharya and
Peninsula. The Sufism reached India in the 12th Ramanujacharya). He propounded the
century A.D. Its influence grew considerably philosophy of Dwaita or Dualism.The basic
during the thirteenth and fourteenth tenet of Dvaita philosophy is the refutation of
centuries. In India, Chisti and Suhrawardi Silsila the Mayavada of Sri Shankara.
were most prominent.
 Statement 3 is correct: The Bhakti and Sufi
saints preached their teachings in the local
74

 They consider Karma a fallacy and also rejected


the authority of Vedas like Buddhism
andJainism did. However, they believed in the
existence of soul (atman) in every living being
like Jainism. But they believed existence of soul
inmaterial form whereas Jainism propounds
formless soul.
 Statement 3 is incorrect: The ajnana sect
Dvaitha emphasizes that the world is real and believed in radical skepticism. The Ajnana
not just an illusion. He was a critic of Adi school believed that it is impossible to attain
Shankara's Advaita Vedanta and Ramanuja's knowledge about nature. Even if it is possible,
Vishishtadvaita Vedanta teachings. He wrote it is useless for attaining salvation and
various texts that detailed his philosophy emancipation of the soul. This school was a
which he called Tattvavada, or as it is more strong rival of Jainism and Buddhism school of
popularly known, Dvaita. Some of his works thoughts.They specialised in refutation or
were the Gita Bhashya, Brahma Sutra Bhashya, disproof and were considered ignorant.They
Anu Bhashya, Karma Nirnaya, and Vishnu believed that “Ignorance is Best ''.
Tattva Nirnaya.According to him, salvation is
only possible through God's grace. The Dvaita
School influenced Vaishnavism, the Bhakti Q.14) Answer: C
movement in mediaeval India, and has been  Option A is correct: The Harappan script
one of the three influential Vedānta was pictographic and logosyllabic (each
philosophies, along with Advaita Vedanta and symbol stood for a word/syllable). Harappan
Vishishtadvaita Vedanta. writing was boustrophedon, that is to
say, right to left and left to right in alternate
Q.13) Ans: A lines. The Harappan script has not been
Exp: deciphered so far.
 Option B is correct: According to Buddhist
 Statement 1 is correct: Ajivika school was
texts, Anguttara Nikaya (the land between
founded by Makkhali Gosala in 5th century BC.
Himalayas and Narmada) was divided into 16
The school is based on the Niyati (Fate)
independent states (Mahajanapadas)
doctrine of absolute determinism. It believes – Anga, Magadh, Kasi, Vatsa, Kosala,
that there is no free will of humans and Saurasena, Panchala, Kuru, Matsya, Chedi,
whatever has happened, is happening and will Avanti, Gandhar, Kamboj, Asmaka, Vajji and
happen is completely predetermined or pre- Malla.
decided and is based on cosmic principles or  Option C is incorrect:
Universal. Hence there was no use and impact  Visakhadatta’s Mudrarakshasha is a political
of Karma. It is rooted in the theory of atoms drama that narrates the ascent of king
and believes that everything is a component of Chandragupta Maurya to power.
atoms and the various qualities emerge from  Visakhadatta’s Devichandraguptam tells us
the aggregates of atoms which are about Ramagupta’s defeat by a Saka ruler
predetermined. (Basana), his accession to the throne and
his marriage to Dhruvadevi (his brother’s
 Statement 2 is correct: Ajivikas led a very
widow).
simple ascetic life even without clothes and
any material possession. They strongly
Q.15) Answer: C
opposed Buddhism and Jainism and were
 Statement 1 is correct: The Mathura school of
atheists. They did not believe in the philosophy
art was developed indigenously and was
of Karma unlike Jainism and Buddhism.
devoid of external influence. It was built
75

using red sandstone under the patronage  Pair 2 is incorrect: Koldihwa is an archeological
of Kushana rulers. It was influenced by all site located in Allahabad on the left bank of the
three religions, Buddhism, Jainism, and Belan River. The site contains excavations
Hinduism. It depicts the Buddha in a smiling of Neolithic, Chalcolithic and Iron age phases.
state. It is found in the areas around Mathura,  Pair 3 is incorrect: The Burzahom
Kankalitila, Sonak. archaeological site is located in the Kashmir
Valley of the Indian state of Jammu and
Kashmir. Archaeological excavations have
revealed four phases of cultural significance
between 3000 BC and 1000 BC. Periods I and II
represent the Neolithic era, Period IlI the
Megalithic era, and Period IV relates to the
early Historical Period. It is situated on a high
terrace, which is part of the flow of the river
 Statement 2 is correct: The Indo-greek or Jhelum.
the Gandhara school of art had indo-greek  Pair 4 is correct: Gufkral (literally Gulf – cave
influence and was not purely indigenous. It Kral – potter) is located in the south Kashmir
used grey sandstone and was also patronized district, considered as one of the sites in the
by the Kushana rulers. It was mainly Buddhist valley which has been dated by archaeologists
and shows the spiritual mood of Buddha. It to the Neolithic period. It is part of river Jhelum
developed in the areas around present day and falls between two nallahs (streams) on an
Kandhar. extensive deposit of Karewa (elevated table-
 Statement 3 is incorrect: The Amaravati land) where people used to live in ancient
school of art was indigenously developed and times.
mainly focused on Buddhism. It used white
marbles and was patronized by Q.17) Answer: A
the Satavahana rulers. It was developed in the  The Chahmanas of Shakambari, also known as
lower valleys of Krishna-Godavari around the Chauhans of Sambhar, were a dynasty that
Amravati and Nagarjunakonda. ruled the present day parts of Rajasthan during
the 6th to 12th century time period.
Q.16) Answer: B  Ajayameru was established by a Chahamana
 Pair 1 is incorrect: Chirand is an archaeological ruler Ajayaraja I and served as capital until the
site in Bihar. It is situated on the northern bank 12th century. Ajayameru is translated as
of the river Ganga. It is the first known site of invincible hills. It is the modern day Ajmer and
the Neolithic age. It has a large prehistoric surrounded by Aravalli hills.
mound containing various artifacts. The  Mankhet, modern day Malkhed is in
mound is known for its continuous Karnataka,situated on the southwest part of
archaeological record from the Neolithic age Hyderabad. The city was founded in the 9th
(about 2500–1345 BC) to the reign of the Pal century by the Rashtrakuta ruler
Dynasty, which ruled this place during the pre- Amoghavarsha I and became the capital of the
medieval period. dynasty.
 Kanyakubja is the classical name of the
modern day Kannauj city in Uttar Pradesh. It
was also known as Mahodaya during the time
of Gujarat samrat Mihir Bhoja(9th Century AD).
The kanyakubja brahmins who included
Shandilya were held as the three prominent
families originally from Kannauj. Kanyakubja
was the centre of the tripartite struggle
between the Pala, Rashtrakuta and Pratihara.
76

 The Kannadiga dynasty is known as grains and was the precursor of the modern
the Kalyani Chalukya after the name of its rupee. It remained largely unchanged till the
capital at Kalyani, it is today’s Basavakalyan in early 20thCentury. Together with the silver
the modern Bidar district of the Karnataka Rupiya were issued gold coins called
state. the Mohur weighing 169 grains and copper
coins called Dam.
Q.18) Answer: C  Statement 3 is Incorrect : Sher shah suri was a
In 1205 AD, Ghori while entering India came across patron of art and architecture. He built many
stiff resistance from the Khokar tribe of Punjab forts and mosques among other architecture.
region, but he defeated them. Khokhar tribes were o He built the Rohtas Fort (now a UNESCO
warrior tribes, good at fighting and in 1206 when World Heritage Site in Pakistan)
Ghori after his brief stay in India was returning to o He built Many structures in Rohtasgarh
Ghanzi was killed by the Khokhars in Dhamyak fort in Bihar.
district near Jhelum river (Modern day Pakistan).
Q.21) Answer: A
 Statement 1 is Correct : Under the
Q.19) Answer: C administration of Sher Shah Suri, the
 Firuz Shah Bahmani encouraged the pursuit of government was highly centralised and
astronomy and built an observatory near consisted of several departments. There were
Daulatabad. He paid attention to the ports of also many administrative units
his Kingdom, Chaul and Dabhol, which called iqtas. The king was assisted by four
attracted trading ships from the Persian Gulf important ministers:
and the Red Sea. o Diwan-i- Wizarat – Also called Wazir, in
 Firuz Shah Bahmani was determined to make charge of Revenue and Finance.
Deccan the Cultural centre of India and the o Diwan-i-Ariz – In charge of the Army.
decline of Delhi Sultanate helped him as many o Diwan-i-Rasalat – Foreign Minister
learned people migrated from Delhi to the o Diwan-i-Insha – Minister for
Deccan. Further he was also well acquainted Communications.
with natural sciences such as botany,  Statement 2 is Correct : He improved the land
geometry. revenue system by adopting Zabti-i-har-
 He was also well acquainted with religious sal (land assessment every year) and classified
Sciences like commentaries on the Quran. He all cultivable lands into three heads (good,
was a good calligraphist and a poet and often middle, bad).
composed extempore verses.  Statement 3 is Incorrect : The state’s share
was one third of the average produce and it
Q.20) Answer: B was paid in cash or crop. Land was measured
 Statement 1 is Incorrect : Sher Shah, even using Sikandari gaz (32 points). Sher Shah
though a pious Muslim, adopted introduced two documents:
a tolerant attitude towards other religions. He o Patta (amount each peasant had to pay)
employed Hindus in important offices. o Qabuliyat (Deed of agreement).
o His last campaign was against
Kalinjar (Bundelkhand), in which Sher Shah also introduced new copper coins
he succeeded but died from an accidental called Dam and it is interesting to note that they
explosion of gunpowder in c.1545 CE were in circulation till c.1835 CE. He was also the
 Statement 2 is Correct : The system of tri- first ruler to introduce silver Rupaiya (1 Rupayia =
metalism which came to characterise Mughal 64 dams) and gold coin (Ashrafi/Mohur)
coinage was largely the creation, not of the
Mughals but of Sher Shah Suri (1540 to 1545
AD), an Afghan, who ruled for a brief time in Q.22) Ans: C
Delhi. Sher Shah issued a coin of silver which Exp:
was termed the Rupiya. This weighed 178
77

 Statement 1 is correct: The Kanheri Caves are  Statement 3 is correct: Kanheri was
a group of caves and rock-cut monuments mentioned in the travelogues of foreign
separated into a massive basalt outcrop in the travellers. The earliest reference of Kanheri is
forests of the Sanjay Gandhi National Park, on ascribed to Fa-Hein who visited India during
the former island of Salsette in the western 399-411 CE.
outskirts of Mumbai. The Kanheri caves
comprise further than 110 different rock-cut
monolithic excavations and is one of the
largest single excavations in the country. Students Note:

 Statement 2 is incorrect: The Kanheri cave


excavations were primarily accepted during
the Hinayana phase of Buddhism but also has
several exemplification of the Mahayana
stylistic architecture and few printings of the
Vajrayana order.
 The name Kanheri is derived from ‘Kanhagiri’
in Prakrit and occurs in the Nasik inscription of
the Satavahana ruler Vasisthiputra Pulumavi.
They contain Buddhist sculptures and relief
carvings, paintings and eulogies, dating from
the first century CE to the 10th century CE.



You might also like